Сохранен 531
https://2ch.hk/sci/res/372443.html
Домены arhivach.top и arhivach.site временно не функционируют! Используйте домен ARHIVACH.XYZ.
24 декабря Архивач восстановлен после серьёзной аварии. К сожалению, значительная часть сохранённых изображений и видео была потеряна. Подробности случившегося. Мы призываем всех неравнодушных помочь нам с восстановлением утраченного контента!

Вопросы тупые, и еще тупее

 Аноним 30/06/16 Чтв 17:53:52 #1 №372443 
14672984328080.jpg
14672984328091.jpg
14672984328092.jpg
Куда девается солнце ночью?
Почему огонь обжигает?
Откуда берется ветер?
Почему волны могут только интерферировать, но не изменять друг друга?
Как с помощью звуковых колебаний измерить молекулярную структуру газа?
Почему фотон выбивает электрон из атома при фотоэффекте, если электрон в атоме представляется волной функцией, которая говорит что электрон размазан по орбитали атома и не имеет конкретного местопложения, а фотон мало того что волна, так еще и энергия в нем помещена в волновой пакет, который тоже может быть в любом месте фотона, если фотон представлять как колебания электромагнитного поля?
Все то о чем вы хотели спросить, но боялись спросить лень было создавать тред.

Предыдущий поглощается имматериумом тут
https://2ch.hk/sci/res/369551.html
Аноним 30/06/16 Чтв 17:59:46 #2 №372448 
Так все же пояснит кто-либо: как выглядит аннигиляция пары грамм антивещества?
Аноним 30/06/16 Чтв 18:01:23 #3 №372449 
Чем дирекционный угол отличается от азимута?
Аноним 30/06/16 Чтв 18:06:26 #4 №372452 
>>372439
Слишком мало мощности у сварки, тепло успевает рассеиваться. А тут у нас почти мгновенный разогрев окружающего воздуха(или зданий) до тысяч и миллионов градусов, всё испаряется, переходит в состояние плазмы, а обьём не успевает сильно увеличится-в итоге высокое давление и сам взрыв. Разницы между взрывом антиматерии и ядерным на самом деле почти нет, разве что в случае ядерного взрыва остаются продукты деления, и часть энергии идёт в тепло этих продуктов.
Аноним 30/06/16 Чтв 18:18:36 #5 №372456 
>>372452
Только что ты выше говорил что это просто фотоны. Какой взрыв? Вещества то нету.
Аноним 30/06/16 Чтв 18:30:23 #6 №372458 
>>372456
А у нас аннигиляция в вакууме или на Земле? Если в вакууме-будет просто вспышка(да и от ядерного взрыва тоже будет просто вспышка), которая максимум испарит что-нибуть неподалёку и наведёт радиоактивность на то, что подальше, если где-то на Земле-будет примерно как я и написал выше
Аноним 30/06/16 Чтв 18:38:36 #7 №372461 
>>372458
Почему? Как свет может вызвать взрыв такой мощи? Он же просто нагреет округу. Свет же не вещество.
Аноним 30/06/16 Чтв 18:51:57 #8 №372464 
>>372461
>просто нагреет
Да, он именно что просто нагреет воздух до миллионов градусов, а получившийся воздух просто будет расширяться, в итоге получим просто взрыв. Серьёзно, ядерные бомбы так и работают, они просто нагревают воздух.
Аноним 30/06/16 Чтв 18:59:46 #9 №372467 
>>372464
То есть вне атмосферы плевать на взрывы килотон антивещества? Просто свет и радиация?
Аноним 30/06/16 Чтв 19:24:59 #10 №372469 
>>372467
Да. Для взрыва 1 грамма антиматерии, если учитывать только тепловое действие фотонов, уже на расстоянии 100 км на 1 кв.метр попадёт 18 кДж (такая энергия нагреет литр воды на 4.3 градуса), то есть немного.
Ну а в случае аннигиляции тонны антиматерии тот же результат надо умножить в миллион раз. Если считать человека массой 100 кг полностью состоящим из воды, и с площадью сечения 1 кв.метр-на расстоянии вплоть до 12 тысяч км(диаметр Земли) его просто испарит излучением. А так да, плевать.
Аноним 30/06/16 Чтв 19:31:51 #11 №372470 
>>372469
Так, я немного(в 14 раз) ошибся, испарит человека во втором случае на расстоянии не выше 840 км
Аноним 30/06/16 Чтв 19:39:04 #12 №372473 
>>372467
Именно. Максимум остывающим газом забрызгает.
Собственно и в атмосфере примерно так же - 1кг С4 (оче дохуя мощной взрывчатки) смертелен только до 3-х метров, а ближе полуметра от тебя мало что останется.
Аноним 30/06/16 Чтв 19:52:07 #13 №372477 
>>372470
А плотность атмосферы ты учёл то?
>>372473
Там материя вещество из с4 разлетается и создаёт давление, а фотоны и прочие нейтрино не могут же так же на материю воздействовать?
Аноним 30/06/16 Чтв 19:55:39 #14 №372478 
>>372477
>плотность атмосферы
Речь про космос же
>>372467
>вне атмосферы
Аноним 30/06/16 Чтв 20:15:53 #15 №372480 
>>372478
Чет и впрямь не понял). Но почему радиация вызывает ударную волну так и не понял.
Аноним 30/06/16 Чтв 20:19:41 #16 №372481 
>>372480
В атмосфере: излучение нагревает воздух, тот расширяется, вот и ударная волна.
В космосе никакого воздуха нет, и ударных волн соответственно тоже, излучение нагревает первый попавшийся обьект
Аноним 30/06/16 Чтв 20:24:21 #17 №372482 
>>372481
Че за говно? Тогда как ядерная бомба взрывную волну создает или там теперь аннигиляция? Чет пиздец сложна
Аноним 30/06/16 Чтв 20:31:57 #18 №372484 
>>372482
Точно так же: нагревая окружающий воздух и увеличивая его давление. Правда тут к этому прибавляются продукты деления, также разогретые реакцией.
Аноним 30/06/16 Чтв 20:47:21 #19 №372489 
>>372484
Только сейчас понял, что проебал облако рад. пыли. Спасибо.
Аноним 30/06/16 Чтв 21:08:45 #20 №372498 
если e^iп = -1, то почему нельзя левую часть возвести в квадрат чтобы избавиться от i. т.е e^(-п^2) по идее должно получиться. но это же не будет правой стороне равно.
Аноним 30/06/16 Чтв 21:10:22 #21 №372499 
>>372477
Могут, если их много.
Гугли световые крутильные весы.

Но обычная взрывчатка не может породить много фотонов, а ядерная, термоядерная и АМ2 - может. Вот и вся разница.
Аноним 30/06/16 Чтв 21:13:19 #22 №372500 
>>372498
(a^m)^n=a^(mn), при возведении левой части в квадрат у тебя выйдет степень 2iп
Аноним 30/06/16 Чтв 21:14:07 #23 №372501 
>>372498
Потому что степени умножаются. Степень степени равно нижняя степень умножить на верхнюю степень
Аноним 30/06/16 Чтв 21:14:09 #24 №372502 
>>372500
а да, это я хуйню какую то написал ебта, сенкс.
Аноним 30/06/16 Чтв 21:18:15 #25 №372503 
это майндфак какой-то. только подумал что логарифм должен помочь с двух сторон от этой ебаной i избавиться. ан нет, справа же -1. издевательство.
Аноним 30/06/16 Чтв 21:19:32 #26 №372504 
а, ну наверное сперва в квадрат с двух сторон а потом ln?
Аноним 30/06/16 Чтв 21:22:22 #27 №372505 
пиздец, получается -2п = 0. охуенно
Аноним 30/06/16 Чтв 21:32:37 #28 №372506 
>>372503
>>372504
Ты поосторожней, комплексный логарифм-вещь опасная. Он многозначный, как например арксинус.
>>372505
exp(2ip)=1
2ip=0+2kip, всё сходится, 2kip-слагаемое, аналогичное 2kp в тригонометрии
Аноним 30/06/16 Чтв 21:41:15 #29 №372510 
>>372506
ну да, я понял, по кругу крутимся, понятно что -2pi = 0. просто интересно можно ли в формулу эйлера подставить с двух сторон какие-нибудь числа кроме удобных типа pi и получить численное равенство.
Аноним 30/06/16 Чтв 21:56:47 #30 №372517 
>>372510
> -2pi = 0.
Эй-эй, это так не работает!
>просто интересно можно ли в формулу эйлера подставить с двух сторон какие-нибудь числа кроме удобных типа pi и получить численное равенство.
Не пойму, что ты хочешь. Бери и подставляй любое число, и получай равенство. Вот например exp(2i)=cos 2 + isin 2. Кстати через формулу Эйлера легко получать формулы удвоенных углов и другие похожие формулы геометрии. Вот пример:
exp(ix)=cos(x)+isin(x)
Re[exp(i(x-y))]=cos(x-y)
Re[exp(i(x-y))]=Re[exp(ix)exp(-iy)]=Re[(cos(x)+isin(x))(cos(y)-isin(y))]=Re[cos(x)cos(y)+sin(x)sin(y)-icos(x)sin(y)+icos(y)sin(x)]= cos(x)cos(y)+sin(x)sin(y)
Аноним 30/06/16 Чтв 22:04:16 #31 №372519 
>>372517
>Не пойму, что ты хочешь. Бери и подставляй любое число, и получай равенство. Вот например exp(2i)=cos 2 + isin 2

Я не понимаю как численно проверить что левая часть равна правой. И что это может быть за "число". Справ я могу хотя бы модуль и угол вычислить. Слев же только мнимая часть. Экспонента от мнимой части может пониматься как движение по кругу. Как яблоки с апельсинами сравнивать?
Аноним 30/06/16 Чтв 22:07:50 #32 №372520 
>>372517
>Эй-эй, это так не работает!
чому не работает. про e^ix можно думать как движение по кругу при изменении х. 0 и -2pi совпадают.
Аноним 30/06/16 Чтв 22:18:30 #33 №372524 
>>372519
Ну да, по другому никак не проверить. Разве что разложить левую часть в ряд(что вобще-то не совсем законно), и то же с правой частью. Ну или просто можешь считать формулу Эйлера определением возведения в степень
>>372520
Из того, что exp(2ip)=exp(0) не следует, что 2ip=0, точно так же как из того, что cos(x)=cos(-x) не следует, что x=-x. Просто комплексная экспонента периодическая по мнимой оси, вот и всё. А если принять, что 2ip=0, тогда либо 2=0, либо i=0, либо p=0, и любой из этих исходов противоречит всей известной математике
Аноним 30/06/16 Чтв 22:24:49 #34 №372527 
>>372524
>считать формулу Эйлера определением
Да, пожалуй нравится мне такой подход. Тут вон некоторые N определить пытаются, и от этого плохо спят.

>Из того, что exp(2ip)=exp(0) не следует, что 2ip=0, точно так же как из того, что cos(x)=cos(-x) не следует, что x=-x.

Ну да, это очевдино, просто я заколупался с с преобразованиями справа-слева особо не обдумывая последствия
Аноним 30/06/16 Чтв 22:31:27 #35 №372530 
интересно помогли ли человечеству компьютеры найти какие-нибудь интересные мат. совпадения типа аппроксимации пи или e или вообще любых иррациональных выражений рациональными или совпадения типа 2^5 * 9^2 = 2592 и тп. похоже все интересное уже давно нашли без всяких комплюктеров.
Аноним 30/06/16 Чтв 23:50:32 #36 №372554 
Может ли наука быть объективной или все зависит от определения?
Аноним 30/06/16 Чтв 23:51:27 #37 №372555 
>>372530
Помогли, но хули толку?
Иррациональное рациональным при всем желании не заменишь - только приблизительно.
Но приблизительные числа с приемлемой точностью нашли давно, вручную.
А то, что выдает компьютер - с высокой точностью, и там такие числа, что хер запомнишь. Так что нафиг не нужно.
Аноним 01/07/16 Птн 00:03:16 #38 №372557 
>>372554
Нет, не может. Наука строго субьективна.
Это прямо прописано в основных научных принципах.

От этого ученым частенько печет. Наука отрицает то, что уже работает, но пока не введено в официальную научную картину мира. Это крайняя степень субьективности - наука видит мир строго в черно-белом цвете, причем белого там намного меньше, чем видят сами ученые, от этого и печет.
Аноним 01/07/16 Птн 01:12:02 #39 №372567 
>>372443 (OP)
Модератор, поясни какое лебедев отношение к наукачу имеет? Ты хочешь чтоб его отравили или он такой глупый что тебе денег за это заплатил?
Аноним 01/07/16 Птн 01:13:07 #40 №372568 
Что значит двойное слепое исследование?
Раньше думал что это сдача анализов в двух независимых лабораториях, но википедия сделала мне когнитивный диссонанс.
Аноним 01/07/16 Птн 01:45:35 #41 №372571 
>>372568
Испытуемые делятся на две группы, одной даётся исследуемый препарат, второй - физраствор/сахар/что угодно (плацебо), причем ни принимающие, ни дающие не занают, что кому дают (врачам это дело даётся в виде таблеток под номером, что именно значит номер, они не знают), потом смотрят на рвзницу между группой, принимавшей плацебо, и группой, принимавшей препарат. Таким образом учитывается эффект поацебо - условно говоря, если человек принимает сахар и ВЕРИТ, что эт эффективное лекарство, оно может оказать какое-то ненулевое действие, если препарат реально эффективен, он должен действовать лучше полацебо, а если действует не лучше - такой препарат на рынок выпускать мысла никакого нет. Ну, это для исследований лекарств, но логика такая же и для всего остального.
Аноним 01/07/16 Птн 02:41:25 #42 №372576 
>>372567
кстати подгорает у меня, что тупая моча лебедева ебальник вывесила и уже несколько месяцев этот уебан - лицо саентача. все таки каким нужно быть дебилом, чтобы такое вывесить.
Аноним 01/07/16 Птн 03:07:59 #43 №372577 
14673316797880.jpg
>>372576
>>372567
Согласен. Пикрелейтеду в подмётки не годится
Аноним 01/07/16 Птн 03:10:38 #44 №372578 
>>372571
А тот метод исследований про который я думал как называется? Просто проверочное тестирование или есть более чёткое определение именно для того случая?
Аноним 01/07/16 Птн 03:14:05 #45 №372579 
На сколько нужно подняться вверх над земной поверхностью жителю столицы Габона, чтобы в ясную погоду увидеть устье амазонки?
Аноним 01/07/16 Птн 03:25:58 #46 №372580 
>>372579
Переправил твой вопрос в тред поехавших сторонников плоской земли.
Аноним 01/07/16 Птн 03:34:50 #47 №372581 
14673332905180.png
>>372580
А вот хуй. Ответ: примерно на высоту радиуса земли - около 6400 км. Вот только почему, хуй знает. (Из книги 1000 вопросов и ответов по географии, авторы Болысов и Гладкевич - преподы МГУ). Я ответил, что это невозможно.
Аноним 01/07/16 Птн 04:06:04 #48 №372583 
Задачка уровня седьмого класса. Для простоты можно считать, что обе точки на экватторе (это близко к истине), поле этого можно решать всё на плоскости, проходящей через экватор. Тогда положение наблюдателя - это такая точка на продолжении радиуса из центра Земл к Габону, что касательная из неё к экватору коснётся его в устье Амазонки. Получаем прямоугольный треугольник, где гипотенуза - радиус земли плюс искомая высота, один из углов (с вершиной в центре Земли) - 58 градусов (устье - 59 градусов западной долготы, Габон - 9 градусов восточной долготы), а прилежащий к этому углу катет - радиус Земли. То есть гипотенуза - R/cos 58, а высота - R/cos 58-R = R(1/cos 58-1) = R*0.887
Аноним 01/07/16 Птн 04:06:59 #49 №372584 
>>372581
>>372583
Устье 49, конечно.
Аноним 01/07/16 Птн 04:10:29 #50 №372585 
>>372583
Собственно, по-хорошему решается вообще без калькулятора, cos 60 = 0.5, имеющиеся 58 градусов достаточно близки. То есть расстояние от центра до наблюдателя примерно в два раза превосходит радиус Земли, а высота, соответственно, примерно равна радиусу.
Аноним 01/07/16 Птн 04:20:38 #51 №372586 
>>372581
я думал ты рашку габоном назвал
Аноним 01/07/16 Птн 05:50:07 #52 №372590 
Чем отличается волна света от волны вероятности электрона?
Аноним 01/07/16 Птн 07:02:10 #53 №372592 
>>372590
Восприятием.
Аноним 01/07/16 Птн 08:49:10 #54 №372600 
>Почему фотон выбивает электрон из атома при фотоэффекте, если электрон в атоме представляется волной функцией, которая говорит что электрон размазан по орбитали атома и не имеет конкретного местопложения, а фотон мало того что волна, так еще и энергия в нем помещена в волновой пакет, который тоже может быть в любом месте фотона, если фотон представлять как колебания электромагнитного поля?

Нет, а и правда, почему так?
Аноним 01/07/16 Птн 09:12:05 #55 №372602 
>>372600
>Почему фотон выбивает электрон из атома при фотоэффекте
Потому что он передаёт атому достаточно энергии для отрыва электрона.
Аноним 01/07/16 Птн 09:27:28 #56 №372603 
>>372602
Пиздоглазое мудило, ты вторую часть прочитай
Аноним 01/07/16 Птн 09:36:09 #57 №372604 
>>372603
Попрошу без оскорблений. Весь этот текст про размазанность никакого отношения к фотоэффекту не имеет.
Аноним 01/07/16 Птн 09:45:02 #58 №372605 
>>372600
Потому что может, ёпта!

Нельзя, и ты об этом сам пишешь, воспринимать фотон и электрон как волну или как частицу, можно только как частицу-волну, к тому же, находящуюся да-да, в единственном числе не только во времени наблюдателя.
Аноним 01/07/16 Птн 10:26:55 #59 №372610 
Есть ли какие-то экспериментальные подтверждения расширения Вселенной, не связанные с красным смещением?
Аноним 01/07/16 Птн 10:26:55 #60 №372611 
>>372605
>электрон и фотон это одна волна
Как там с бранами, все уже нашли?
Аноним 01/07/16 Птн 10:51:30 #61 №372612 
>>372590
Первое материя, второе математика.
Аноним 01/07/16 Птн 12:05:58 #62 №372621 
>>372611
Нет. Его девчушка всё ещё тащит в Винтерфелл. Прекрати.
Аноним 01/07/16 Птн 12:19:04 #63 №372627 
>>372612
А физический смысл второе имеет?
Аноним 01/07/16 Птн 13:10:51 #64 №372634 
>>372443 (OP)
>Почему волны могут только интерферировать, но не изменять друг друга?
Волны не взаимодействуют, только интерферируют.
Все частицы - волны.
...
Частицы не взаимодействуют.
Аноним 01/07/16 Птн 14:38:40 #65 №372648 
>>372627
Имеет
Аноним 01/07/16 Птн 14:43:53 #66 №372649 
>>372648
Чем эти волны отличаются в с точки зрения их физического смысла?
Аноним 01/07/16 Птн 15:05:48 #67 №372652 
>>372649
Да ничем.
И электрон и фотон - кванты энергии.
Во время движения они не имеют четкой позиции. Так что их положение описывают полем вероятностей.
Аноним 01/07/16 Птн 17:15:47 #68 №372682 
про иррациональное число на прямой.

есть у меня точка на прямой с иррациональной координатой. можно записать координату как десятичное приближение, например для, пи: 3.14
и указать эту приближенную точку на прямой. а потом уточнить приближение, теперь пи = 3.145
на прямой, новая точка пи сдвинулась правее, на 3.145 - 3.14.
если и дальше уточнять, то точка будет двигаться в право. а так-как уточнять можно до бесконечности, то точка будет все время двигаться в право. тогда, что значит такая точка на прямой? на точку можно указать пальцем. а у меня точка движется.
Аноним 01/07/16 Птн 17:19:41 #69 №372683 
>>372682
В пределе после бесконечного числа уточнений ты попадёшь как раз на точку, соответствующую пи.
Ну или можно скрутить круг единичного диаметра из бумаги, развернуть его и приложить одним концом к точке 0, а пальцем указать на другой конец, вот тебе и пи
Аноним 01/07/16 Птн 22:32:01 #70 №372750 
>>372443 (OP)
Чудо, с научной точки зрения?
Аноним 02/07/16 Суб 02:14:56 #71 №372814 
>>372750
Явление, которое пока что не объяснила наука
Аноним 02/07/16 Суб 02:35:16 #72 №372815 
>>372448
2 грамма антивещества + 2 грамма вещества.
E=mc^2. m=0.004. 0.004910^16=36*10^13 Дж.
В тротиловом эквиваленте: 90 килотонн. Соответствует средней (на границе с большой) по мощности ядерной бомбой. Высота гриба ~6.5 км, диаметр облака ~3.5 км.
Аноним 02/07/16 Суб 02:40:18 #73 №372817 
>>372815
Откуда 0.004910^16 ?
Аноним 02/07/16 Суб 07:38:49 #74 №372833 
14674343291980.jpg
Почему спин существует у фотонов? Если это вращение, то его ведь невозможно увидеть?
Аноним 02/07/16 Суб 08:10:20 #75 №372834 
Вопрос - возможно ли полностью представить четырехмерное тело с помощью трехмерных проекций?
Аноним 02/07/16 Суб 08:20:59 #76 №372835 
>>372834
Если взять достаточное кол-во проекций, то да. Одной не хватит, двух хватит, но если известны параметры проекций.
Аноним 02/07/16 Суб 08:49:02 #77 №372836 
>>372833
Достаточно открыть википедию. Это не вращение, это вибрация в пространстве-времени, в нескольких его точках.
Аноним 02/07/16 Суб 08:59:59 #78 №372838 
>>372833
Поляризация
Аноним 02/07/16 Суб 09:25:29 #79 №372842 
>>372817
Я не он, но судя по наклону девятки имелось в виду 0.004х9х10^16.
Аноним 02/07/16 Суб 10:24:54 #80 №372848 
>>372836
Какая ещё вибрация??? Момент импульса всю жизнь был про вращение, википедия пишет также.

Почему возможные спины так ограничены?
Аноним 02/07/16 Суб 10:30:12 #81 №372849 
>>372848
Это абстракция в среде квантов.
Там намеряли магнитный момент больший чем дает теория и для спасения теории прикрутили спин, который увеличивает магнитный момент и представляет из себя хуй знает что
Аноним 02/07/16 Суб 10:37:29 #82 №372850 
>>372848
А в квантмехе понятие "вращение" немного теряет смысл, остаётся только момент импульса, орбитальный или собственный. Спин-собственный момент импульса.
Аноним 02/07/16 Суб 10:58:35 #83 №372854 
>>372849
магнитный момент фотонов?
Аноним 02/07/16 Суб 11:38:11 #84 №372858 
>>372854
Ну так это же костыль, как темная энергия.
Просто теория в этом месте расходится с реальностью, поэтому получается такая хуйня.
Ученые новую теорию искать не хотят - им и со старой норм.
Аноним 02/07/16 Суб 11:54:35 #85 №372859 
>>372858
>>372854
>флогистон объясняет горение
>все кивнули в знак одобрение
>все видали горение дерева
>почернеет оно и скрючится
>флогистон из него улетучится
>флогистон объясняет горение
>все кивнули в знак одобрения
Аноним 02/07/16 Суб 12:09:54 #86 №372865 
Что почитать по методологии науки? Ну там как ставят эксперименты в разных областях, как интерпретируют результаты, работают со статистикой, как публикуют какие подводные камни, как выборки составляют, как наёбывают?


Позитивные результаты обнародуют охотнее, чем негативные. Допустим, вы только что завершили строго научное повторное исследование, которое показало, что увлечение видеоиграми не препятствует развитию рака толстой кишки. На протяжении двадцати лет вы обследовали репрезентативную выборку из 100 000 американцев и выяснили, что среди фанатов видеоигр и тех, кто совершенно ими не интересуется, заболеваемость раком толстой кишки находится примерно на одном уровне. Предположим, ваша методология безупречна. Какой из престижных медицинских журналов опубликует результаты данного исследования?

Думаю, никакой. И тому есть две причины. Во-первых, нет ни одной научно обоснованной причины полагать, что увлечение видеоиграми способствует развитию рака толстой кишки, а потому не совсем понятно, зачем вы проводили свое исследование. Во-вторых, тот факт, что некий фактор не препятствует заболеванию раком, не представляет научной ценности. В конце концов, многие вещи этому не препятствует. Негативные результаты не производят особого впечатления — ни в медицине, ни где бы то ни было.

А теперь допустим, что один из ваших приятелей, вместе с которым вы оканчивали магистратуру, провел другое повторное исследование и выяснил, что среди тех, кто много играет в видеоигры, заболевание раком толстой кишки встречается реже. А вот это уже гораздо интереснее! Результаты именно такого рода привлекают повышенное внимание медицинских журналов, популярной прессы, блогеров и разработчиков компьютерных игр (которые не преминут их снабдить надписью о пользе игр для здоровья). Пройдет совсем немного времени, и заботливые мамаши по всей стране бросятся спасать своих детей от рака, выхватывая у них книги из рук и заставляя играть в видеоигры.

Разумеется, согласно важному положению в статистике, необычные явления происходят довольно редко и, как правило, в результате случайного стечения обстоятельств. В одном из 100 аналогичных исследований наверняка обнаружатся нелепые результаты типа взаимозависимости между увлечением видеоиграми и меньшей заболеваемостью раком толстой кишки. Проблема в том, что результаты 99 исследований, которые не выявили такую связь, опубликованы не будут, поскольку малоинтересны. А вот единственное исследование, которое ее обнаружит, попадет в печать и привлечет к себе повышенное внимание. Источником данной систематической ошибки является не исследование как таковое, а сомнительная информация, которая фактически становится достоянием широкого круга читателей. Тот, кого интересует литература о видеоиграх и заболеваемости раком толстой кишки, найдет публикацию лишь о единственном исследовании, и в ней будет утверждаться, что увлечение видеоиграми предотвращает заболевание раком. Хотя на самом деле результаты 99 исследований из 100 свидетельствуют об отсутствии какой-либо связи.

Да, мой пример несколько абсурден, однако данная проблема вполне реальна и довольно серьезна. Вот первое предложение из статьи в The New York Times, в которой говорится о систематической ошибке публикации, касающейся лекарств от депрессии: «Производители антидепрессантов, таких как Prozak и Paxil, никогда не публиковали результаты примерно трети испытаний своих лекарств, проводившихся, чтобы получить одобрение государственных контролирующих органов. Таким образом производители антидепрессантов вводили в заблуждение врачей и пациентов относительно подлинной эффективности этих препаратов»19. Оказывается, были обнародованы данные о 94% исследований с положительными результатами, касающимися эффективности этих лекарств, и лишь о 14% исследований с отрицательными результатами. Для пациентов, страдающих депрессией, это крайне важно. Если бы были оглашены результаты всех исследований, то оказалось бы, что в действительности антидепрессанты лишь немногим лучше любого плацебо.

Чтобы справиться с данной проблемой, теперь медицинские журналы, как правило, требуют зарегистрировать любое исследование в самом начале проекта, если предполагается последующая публикация его результатов. Это предоставляет редакторам определенные свидетельства о соотношении позитивных и негативных исходов. Если, например, зарегистрировано 100 исследований по анализу влияния катания на роликовой доске (скейтборде) на развитие сердечно-сосудистых заболеваний, и лишь одно из них будет в конечном счете представлено для публикации с положительными результатами, то редакторы могут заключить, что в ходе других исследований получены отрицательные результаты (или по крайней мере проверить такую вероятность).
Аноним 02/07/16 Суб 12:18:17 #87 №372867 
>>372865
Можешь прочитать про мегавесёлую историю с TGN1412, про то, как исчезают исследования (не публикуются) и прочее веселье.

>В 2008 году группа исследователей решила проверить, сколько результатов было опубликовано после проведения исследований антидепрессантов, попавших на рынок в период между 1987 и 2004 гг.
>Исследователи нашли в общей сложности 74 эксперимента, заслуживающих внимания, где принимали участие 12500 пациентов. 38 из этих исследований дали положительные результаты, которые подтверждали эффективность препарата, а результат 36 был отрицательным. В итоге данные по негативным и положительным эффектам распределились поровну.

>Затем исследователи занялись поиском информации об этих клинических экспериментах в научной литературе, опубликованных материалах, открытых для врачей и пациентов. Картина получилась совсем другой. По 37 исследованиям, принесшим положительные результаты, то есть по всем, кроме одного, результаты были опубликованы полностью, часто с хвалебными отзывами. Однако у исследований, показавших отрицательные данные, была другая судьба: было опубликовано только три из них. 22 эксперимента были просто потеряны в анналах истории и не появлялись больше нигде, кроме как в тех пыльных, неряшливо организованных папках, собранных в архиве Управления по контролю за качеством пищевых продуктов и лекарственных веществ.

>Информация об остальных 11 исследованиях, принесших негативные результаты и хранившихся в сводках ведомства, тоже появилась в научных изданиях, но статьи были написаны так, как будто исследования лекарства закончились успешно.

Отрывок полного текста "bad science" Бега Голдакра. Тут подробнее http://www.socialcompas.com/2016/06/30/kak-farmkompanii-i-kapitalizm-voobshhe-korrumpiruyut-nauku/
Аноним 02/07/16 Суб 12:20:05 #88 №372868 
>>372867
*Бена
Аноним 02/07/16 Суб 13:35:04 #89 №372878 
>>372867
Ну так наука давно коммерческая - кто платит тот и заказывает музыку.
В результате вместо стройной картины имеем кривую косоебину, где сам черт ногу сломит. А иное и быть не могло: в науку нельзя привносить ни капли лжи.
Аноним 02/07/16 Суб 13:37:41 #90 №372880 
>>372878
С публикации ОТО наука уже не про правду, а про влияние и авторитетность. сейчас еще и $$$ прибавились.
Аноним 02/07/16 Суб 13:41:42 #91 №372882 
>>372878
А где тут >>372867
вскрывается ложь? Про качество самих исследований не слова.
Аноним 02/07/16 Суб 13:54:22 #92 №372886 
>>372882
Так ложь в том, что половина данных скрывается.
Данных, неугодных тому, кто проплатил исследования.
А в результате выстраивается неверная картина - будущие исследователи, если попытаются основать свои суждения на этих исследованиях (а иных они и не найдут, лол), однозначно придут к неверным выводам. Исследования то скомпрометированы изначально.
Аноним 02/07/16 Суб 13:59:49 #93 №372887 
>>372886
При чём тут сами исследования?Каким образом сокрытие исследований показавших отрицательный результат говорит об ошибочности или намеренной фальсификации исследований показавших положительный результат?
Аноним 02/07/16 Суб 14:33:05 #94 №372894 
>>372887
Давай просто уберем из законов требование об обязательном указании противопоказаний, и посмотрим что получится.

А получится вот что: большинство лекарств тут же начнут нагло врать про отсутствие побочных эффектов, т.к. продавца совершенно не ебет судьба покупателя.

И не будет никакого способа узнать, безопасно ли то или иное лекарство для тебя.
А потому ты не веришь никакому лекарству, но ведешься на рекламу (чисто психологически).
Таким образом на спрос влияет только маркетинг - для продавца это просто идеально. Для тебя - адъ.

С исследованиями и информацией та же хуйня: часть информации и исследований скрывается, и у тебя уже нет никакого способа узнать достоверность тех или иных публикаций.
Ты точно также вынужден не верить никому, и ведешься на рекламу. А рекламы в этих публикациях выше крыши - все эти хвалебные отзывы.

Короче говоря тобой таким образом манипулируют, тебе нагло лгут.
Сокрытие части правды - это именно ложь.

Как тебе уже и сказали, сокрытие негативных результатов выставляет препарат эффективным, но это ложь.
Если принять во внимание все результаты, выходит что препарат бесполезен.

Ошибки тут нет: если во внимание принимать только исследования, подтверждающие эффективность, то статистика будет говорить об аномальной эффективности. Но статистика ведь не лжет.
Но в реале препарат эффективен в 1% чрезвычайно редких случаев, а в 99% вообще бесполезен, а то и вреден - вот это правда, и ее от тебя скрыли. Тебя поимели, поимели мир, поимели науку.

Так вот наука превратилась в продажную шлюху.

И это разрушает не только саму науку - это еще и уносит жизни людей. Разрешения то основаны на этой лживой статистике, и суды тоже. А причина всему - продажные исследователи.
Аноним 02/07/16 Суб 14:47:33 #95 №372904 
>>372887
Что касается фальсификации ради положительного рез-та, вопрос хороший и годный. Читал на этот счёт пост: http://tanchik.livejournal.com/560377.html

Помимо самого рассказа, интересна часть как раз про то, каким образом получаются очень оптимистичные положительные рез-ты у лекарств, которые не лучше плацебо.
Аноним 02/07/16 Суб 17:20:51 #96 №372925 

>>372894
> А причина всему - продажные исследователи.
Я так и не понял как из того, что отрицательные результаты не публикуют следует продажность исследователей. У исследователя наверное не так много способов повлиять на вероятность публикации отрицательного результата.
Аноним 02/07/16 Суб 17:31:53 #97 №372926 
>>372925
Т.е.по твоему причина в журналах?

Но ведь публикация возможна и в сети - выложи исследование в сеть, кому нужно - найдут. Но нет же - все похоронено в бумажных архивах.
Так что тут исследователь налажал.
Аноним 02/07/16 Суб 17:38:57 #98 №372930 
>>372887
>Каким образом сокрытие исследований показавших отрицательный результат говорит об ошибочности или намеренной фальсификации исследований показавших положительный результат?
Потому что тысячу положительных результатов опровергается одним отрицательным. Само по себе сокрытие информации о негативных экспериментах никак не показывает ошибочности положительных исследований - оно лишь говорит о неполноте имеющейся информации.
Например, где-то в африке провели исследование, и получили результаты, говорящие о том, что лекарство эфективно. Это было с лекарством от ВИЧ, я не так давно постил историю в подробностях. Но беда в том, что куча людей поумирало после исследований, однако, этот факт никак не был зафиксирован в отчетах. Более того, документация велась настолько ужасно, что из нее вообще нельзя было получить никаких достоверных сведений.
Вот тебе и пример положительного опыта. Таких отчетов можно сделать десятки, и, замолчав отрицательные результаты, показать неполную картину. Но не факт, что в отрицательных результатах все прямо так и плохо - ты просто не знаешь, что там было в отрицательных результатах, ты знаешь лишь часть правды, а, как сказал аноньчик выше, частичная правда - это ложь.
>>372894
>А причина всему - продажные исследователи.
Можно найти тетю Дусю, которая метет двор, и дать ей должность исследователя - ты думаешь она откажется? Да ей похую, она подпишет все, что нужно, скажет то, что ей сказали. Дальше результаты исследований тети Дуси опубликуют в Science, все их прочитают, но кому жаловаться? Тете Дусе? Отгадай, насколько она переживает про свой имидж в академической среде?
Настоящая причина - это что есть люди, которым нужна ложь, и которые готовы за эту ложь платить. Люди, которые хотят невозможного - таблеток и оперативного вмешательства, которые бы даровали им здоровье при обжорстве и недостатке двигательной активности, при этом от самого человека требовалось бы не более чем движения пальцем, все будут делать за него, и они будет здоров, в отличной физической форме, с большими мышцами, подтянутой фигурой, безупречной эрекцией, ну или большими сиськами и узкой талией для баб.
Это невозможно, но за это платят деньги. Однако, есть заноза, под названием "ученые", которые будут публиковать правду, правду не хотят слышать ни корпорации, ни потребители.
Аноним 02/07/16 Суб 17:48:19 #99 №372933 
>>372925
>У исследователя наверное не так много способов повлиять на вероятность публикации отрицательного результата.
Есть большая корпорация и гос органы, которые на самом деле представляют собой единое целое. В FDA присылают статью, но она рассматривается так, что в итоге про нее вроде как никто и не слышал, и не знал.
Так было с тиомерсалом, который был многократно проверен, и была хорошо известная его крайне высокая токсичность при неэфективности в качестве стерилизующего агента. Тиомерсал используется в производстве вакицин до сих пор, потому что есть лоббисты, потому что есть заинтересованные люди в гос службах, которые будут в один голос твердить "никаких сведений об опасности тиомерсала нет", хотя они есть.
Ты можешь опубликовать чтото в сети - но кто тебя будет читать? Ты? Ты много читал исследований о том, что химиотерапия имеет практичеки нулевую эфективность? Что препараты для лечения ВИЧ - неизбирательно токсичны? Что многие вакцины имеют серьезные побочные эфекты, вплоть до летального исхода, которые ставят под сомнение необходимость их применения?
Еще раз: в сети все есть. В сети более чем достаточно статей выложено. Но всем похуй. Те же исследования по тиомерсалу есть в интернете - вы можете их скачать. Но по центральным телеканалам никто о них не скажет, и ни в одном популярном журнале их не опубликуют, и даже не сошлются на эти исследования.
Аноним 02/07/16 Суб 18:06:01 #100 №372944 
>>372602
Фотоны не взаимодействуют друг с другом потому, что не передают друг другу достаточное кол-во энергии?
Аноним 02/07/16 Суб 18:29:14 #101 №372954 
>>372944
Фотоны в принципе могут взаимодействовать только в электромагнитном и гравитационном взаимодействиях, но заряд и магнитный момент фотона-0, поэтому электромагнитно они не взаимодействуют друг с другом. Гравитационно-в принципе должны, но это взаимодействие пока что необнаружимо.
Аноним 02/07/16 Суб 18:36:14 #102 №372955 
>>372954
Почему же тогда фотоны взаимодействуют с электронами, если заряд и момент фотона - нулевые?
Аноним 02/07/16 Суб 18:51:05 #103 №372967 
>>372955
Вот не могу уверенно ответить на этот вопрос. Квантовая электродинамика описывает фотон как переносчик электромагнитного взаимодействия, но по какому механизму происходит испускание/поглощение я не знаю. Зато я узнал, что через рождение виртуальных пар фотоны всё-таки могут взаимодействовать друг с другом(хоть и очень слабо), а также участвовать в сильном взаимодействии.
Аноним 02/07/16 Суб 22:25:14 #104 №373000 
>>372814
>>372814
А как же первопричина?
Аноним 03/07/16 Вск 04:03:13 #105 №373041 
На какую высоту можно поднять дирижабль\шар, который будет нести на себе груз (груз обязательно сверху), на текущем уровне развития техники? Из условий, конструкция должна быть практически неподвижна относительно поверхности и быть достаточно высоко (выше плотных слоев атмосферы).
Аноним 03/07/16 Вск 04:27:56 #106 №373042 
>>373041
Стратостаты поднимаются до 30-40 км. Если пораскинуть мозгами, то можно соединить несколько и поднять груз над ними по принципу катамарана. Но конструкция будет неустойчива, а вес груза очень невелик.
Аноним 03/07/16 Вск 07:53:59 #107 №373054 
Объясните, какой физический смысл имеет момент импульса и магнитный момент?
Аноним 03/07/16 Вск 07:57:31 #108 №373055 
>>373054
Это все воображаемые величины, которые используются для оценки вполне стабильных воспроизводимых макросвойств объектов. Температура - еще один красивый пример, ведь в теле некоторые молекулы имеют энергию больше, некоторые - меньше, но температура оценивает общую энергию системы, поскольку у нас в мире энергия сохраняется.
Аноним 03/07/16 Вск 08:01:05 #109 №373056 
>>373054
Ядрёность вращения и магнетизма.
Аноним 03/07/16 Вск 08:13:28 #110 №373057 
Если взять два запутанных фотона , то какие параметры не измеряемого фотона будут определятся в момент измерения измеряемого фотона?
Аноним 03/07/16 Вск 09:26:11 #111 №373061 
Могут ли существовать планеты, на которых возможна жизнь, размером с Юпитер и более, и чтобы на них ускорение свободного падение было как на Земле?
Аноним 03/07/16 Вск 09:53:32 #112 №373064 
>>373057
Все
Аноним 03/07/16 Вск 09:59:45 #113 №373067 
>>373061
Нет.
Аноним 03/07/16 Вск 10:05:27 #114 №373069 
>>373054
Момент импульса-величина, которая:
1. Вектор
2. Сохраняется(причём её сохранение вытекает из изотропности пространства)
3. Аддитивна
>>373061
Для того, чтобы на Юпитере было ускорение свободного падения как на Земле, надо, чтобы величина M/R^2 у Земли и Юпитера совпадали. Тогда плотности Земли и Юпитера будут относится, как Rю/Rз, что даёт плотность такого Юпитера примерно 500 кг/м^3, или очень мало. Планета с твёрдой поверхностью не может иметь такую плотность.
Аноним 03/07/16 Вск 10:25:45 #115 №373071 
>>373057
Можешь перечислить?
Аноним 03/07/16 Вск 10:28:28 #116 №373072 
>>373071
Не тот пост выбрал
>>373064
Можешь перечислить параметры?
Аноним 03/07/16 Вск 10:30:43 #117 №373073 
>>373069
>физический смысл имеет магнитный момент?
?
Аноним 03/07/16 Вск 10:53:31 #118 №373075 
>>373073
Магнитный момент-параметр системы, который показывает, какой момент сил будет действовать на эту систему во внешнем магнитном поле.
Аноним 03/07/16 Вск 11:05:10 #119 №373076 
>>373072
Пардон, уточню: все запутанные. Что запутаешь то и опредлелится.
Ну а что именно - на выбор. Координаты, импульсы, моменты, энергии, комбинаций дохуищща.
Аноним 03/07/16 Вск 12:09:32 #120 №373086 
>>373076
А как понять что координаты запутаны?
Аноним 03/07/16 Вск 12:47:58 #121 №373091 
>>373086
Как и в любом другом случае - провести соответствующий тест. Самый строгий для самой сильной запутанности - Белл (который неравенства) без лазеек.
Аноним 03/07/16 Вск 13:16:41 #122 №373097 
>>373091
>Белл
>найди то, не знаю что, так, не знаю как
Аноним 03/07/16 Вск 13:49:01 #123 №373100 
>>372926
Так в сеть выкладываю или в какие нибудь индуские журналы. Но кто будет читать не прошедшие рецензирование статейки или статейки из журнала издаваемого институтом Панаджи? А как потом тому кто финансирует исследование рассказывать, что тебе нужно 2 миллиона долларов и 1 года что бы проверить результаты, ну или начать новое исследование основываясь на результатах статьи которую ты нашёл на интернет помойке?
Аноним 03/07/16 Вск 14:07:28 #124 №373105 
14675440486900.jpg
Что официальная наука думает о шаровых молниях? Каковы общепринятые гипотезы и мнение анона в частности?
Аноним 03/07/16 Вск 14:14:18 #125 №373108 
>>373105
Есть несколько гипотез разной степени правдоподобности, но вроде ни одна из них не общепринята. Экспериментальных данных слишко мало.
Аноним 03/07/16 Вск 14:29:06 #126 №373111 
>>373105
Я читалд книжку о них.
Вкратце все очень плохо.
Еще там задвигалось что энергия шаровых молний берется от озона, мол химическая она.
Аноним 03/07/16 Вск 14:45:38 #127 №373114 
>>373105
https://geektimes.ru/post/210620/
Аноним 03/07/16 Вск 16:23:57 #128 №373151 
>>373097
С чего бы это?
С лазейками - стандартная техника, прекрасное тренировочное задание для старшекурсника. Без лазеек - чуть сложнее, но уже получилось.
Если не параноик и о системе много чего заранее известно - Белл-тест не обязателен, есть "свидетели" попроще.
Аноним 03/07/16 Вск 16:37:20 #129 №373159 
-0/0 = -1?
Аноним 03/07/16 Вск 17:10:52 #130 №373183 
>>373159
0/0 не имеет определённого значения.
Аноним 03/07/16 Вск 18:41:38 #131 №373215 
14675604989840.jpg
Подскажите что изображено на этой пикче?
Аноним 03/07/16 Вск 18:49:41 #132 №373219 
Охота заиметь химическую и физическую лабораторию, чтобы от души поделать разных прикольных опытов. Проблема только в том, что на своей жилплощади я этого делать не могу в принципе. Где я могу этим заняться? Мне снять гараж, купить там мебель, вентиляцию и обустроить там лабораторию?
Аноним 03/07/16 Вск 19:05:17 #133 №373225 
>>373159
0 не имеет обратного элемента по умножению.
Аноним 03/07/16 Вск 19:07:10 #134 №373227 
>>373219
Смотря что конкретно ты хочешь делать, лол. Дохуя химических опытов требуют только нужных веществ и возможности разогрева.

Физическая лаборатория тоже лол. Ты же понимаешь, насколько разные требования к физ. лаборатории в зависимости от направления?
Аноним 03/07/16 Вск 19:17:46 #135 №373231 
>>373227
> мотря что конкретно ты хочешь делать, лол. Дохуя химических опытов требуют только нужных веществ и возможности разогрева.
Ну в любом случае нужно какое-то помещение, вентиляция и реактивы
> Физическая лаборатория тоже лол. Ты же понимаешь, насколько разные требования к физ. лаборатории в зависимости от направления?
Да мне школьный набор нужен - калориметры, электрометры, электроскопы, статические генераторы электричества, и всё в таком роде.
Аноним 03/07/16 Вск 19:22:57 #136 №373233 
> какое-то помещение, вентиляция
Не особо. Если только ты не лютый педант какой-то.
Аноним 03/07/16 Вск 19:29:10 #137 №373234 
>>373231
И какие эксперименты ты собираешься делать? Пример какой-нибудь
Аноним 03/07/16 Вск 19:32:31 #138 №373238 
>>373234
Да любые из книг для школьников по занимательной физике и химии.
Аноним 03/07/16 Вск 19:33:27 #139 №373239 
>>373238
Что, ты сходу даже пример не назовёшь?
Аноним 03/07/16 Вск 19:35:27 #140 №373240 
>>373239
Ну хотя бы превращать бумагу в вискозу, покрывать металлы другими металлами, измерять теплоту плавления льда и заряд статического электричества.
Аноним 03/07/16 Вск 19:53:55 #141 №373244 
>>373239
Амф, меф, ГМТД, стифнат свинца.
Аноним 03/07/16 Вск 23:16:13 #142 №373279 
Виртуальная частица это именно частица, или просто описание взаимодействия? Если это частица, то в чем ее отличае от реальной?
Аноним 03/07/16 Вск 23:26:23 #143 №373280 
>>373279
Ни то ни другое.
Виртуальноая частица - костыль, призванный обьяснить некоторые парадоксы.
Это как сокращения/замещения в математике.

Т.к. энергия вакуума равна нулю, но ничто не мешает представить вакуум как сумму энергий постоянно рождающихся и аннигилирующих пар частиц и античастиц, где каждая частичка имеет ненулевую собственную энергию, но она уравновешивается энергией античастички, и суммарная их энергия равна нулю.
А раз ноль равен нулю, то разницы как бы и нет.
Аноним 03/07/16 Вск 23:29:33 #144 №373282 
>>373279
Это именно частица. Отличие в том, что своим существованием она почти нарушает закон сохранения энергии, а потому может существовать лишь ограниченное время, по принципу неопределённости. У таких частиц взаимосвязь импульса и энергии, или импульса и скорости, не обязана работать, то есть например частица может лететьив одну сторону, а её импульс булет направлен в другую.
Аноним 04/07/16 Пнд 01:53:08 #145 №373311 
14675863885400.jpg
Меня терзает такой вот вопрос по поводу виртуальной реальности. Думаю, уже никто не сомневается, что до конца этого века(а то и до середины) появится техническая возможность создавать ВР практически неотличимую от реальности.
Т.е. надеваешь очки или втыкаешь себе нейроинтерфейс куданибудь , и видишь мир неотличимый от реального.

Так вот, а не является ли сам факт возможности создания реалистичной ВР доказательством того, что наш мир скорее всего и есть ВР? Как это было показано в фильме Экзистенция.
По закону больших чисел.

Т.е. вот если мы получаем возможность создать мир-матрешку копирующую наш мир с некоторыми упрощениями, то из самого факта создания такой матрешки не следует ли , что наш мир с большой долей вероятности в свою очередь тоже является упрощенной копией некого мира? И тогда ведь хер знает какая мы матрешка-копия по счету.
Аноним 04/07/16 Пнд 01:58:47 #146 №373312 
А значит вероятность того, что наша реальность - изначальная оригинальная Матрешка - стремится к нулю, так ведь? Где я не прав?
Аноним 04/07/16 Пнд 02:12:20 #147 №373319 
>>373311
Хуйня и бред эта твоя цитата из фильма.
Аноним 04/07/16 Пнд 02:14:38 #148 №373320 
14675876785400.jpg
>>373311
>Думаю, уже никто не сомневается, что до конца этого века(а то и до середины) появится техническая возможность создавать ВР практически неотличимую от реальности.
Ты еблан и несёшь ебанину.
Сложность воссоздания мира должна меняться экспоненциально от времени пребывания человека в ВР. Смотреть - значит изучать. И изучать живому разуму в ВР будет нечего, как и сейчас. Шаг в сторону - и полигоны торчат.
Аноним 04/07/16 Пнд 02:17:37 #149 №373321 
>>373312
Везде ты не прав. А что происходит внутри черных дыр, единственного кандидата на матрёшки, никто нихуя не знает. Предположение, что там происходит что-то вроде нашего мира -- не более чем очень мощное фантазёрство.

А вообще страшно бля, где-то ведь у всех этой хуеты есть ключик и начало и фундамент. А в то же время и нету. Где эти счетчики, что проверяют законы сохранения? Где эти счетчики, что считают волновые функции? А ещё в квантмехе полная хуета с понятием времени творится, спутанности...

Всё очень плохо в общем, но матрёшка это самое тупое, о чем можно было задуматься в этой ситуации.
Аноним 04/07/16 Пнд 02:17:41 #150 №373322 
>>373319
Ну я тоже думаю что херня. Врядли доказательство того, что матрица имеет нас, такое очевидное. Но где именно ошибка в моих рассуждениях?
Аноним 04/07/16 Пнд 02:20:14 #151 №373323 
>>373322
1. В том, что мы можем сделать замкнутое подобие нашего мира.
2. Квантмех ВЕСЬ ржот над каждой мыслью о такой хуете.
3. Вероятность к применении к этим вопросам -- это вообще ржач.
Аноним 04/07/16 Пнд 02:21:23 #152 №373324 
>>373320
>Сложность воссоздания мира должна меняться экспоненциально от времени пребывания человека в ВР. Смотреть - значит изучать. И изучать живому разуму в ВР будет нечего, как и сейчас. Шаг в сторону - и полигоны торчат.

Ну, понятно, что созданная нами ВР, будет проще нашего мира, сколько бы мощностей мы в неё не вложили. Ну, допустим, в созданной нами ВР не будет воще иных галактик кроме нашей, и все мощности симуляции будут направлены на симулирование жизни только в нашей галактике...А что бы обьяснить допытливому пользователю несуразсности в окружающей физике мира ему будет подкинута идея существования некой гипотетической Темной Энергии\Материи.
Аноним 04/07/16 Пнд 02:23:01 #153 №373325 
Сосоны, я правильно пощитал? Прост что-то не верится.

По расчетам выходит, что для того, чтобы подбросить 2-х тонный автомобиль на высоту трех метров (ну типо для облета препятствия) нужно всего 2гр топлива.
Энергоемкость топлива стандартная - 40-45МДж

2гр - это же пиздец, крохи. На 3 метра 2-х тонную железку? Пиздец же выходит. Не верится.
Аноним 04/07/16 Пнд 02:25:07 #154 №373326 
>>373324
Если не обращать внимания на твой очевидный жир, то воссоздай-ка мне вместо галактики простую, обычную дверную ручку.
Ты охуеешь, узнав, как много информации тебе понадобится только для имитации её внешнего вида и только для взгляда издалека. А как только наш поциент приблизится, тебе понадобится вчетверо больше информации. Потом ещё больше. Потом ещё.
Аноним 04/07/16 Пнд 02:28:02 #155 №373327 
>>373159
На нуль делить нельзя
Аноним 04/07/16 Пнд 02:32:47 #156 №373329 
>>373326

Т.е. создание Матрицы технически невозможно впринципе даже с самым крутым квантовым суперкомпьютером размером с Юпитер? Но почему? Что мешает такому суперкомпьютеру симулировать мир вплоть до квантового уровня? Где квантовым уровнем будет генератор случайных чисел. Вроде как ни один ученый не может доказать что наш мир НЕ симуляция?
Аноним 04/07/16 Пнд 02:42:27 #157 №373331 
>>373329
>Что мешает такому суперкомпьютеру симулировать мир вплоть до квантового уровня?
Отсутствие исходных данных и принципиальная невозможность их получения, например.
Аноним 04/07/16 Пнд 02:46:51 #158 №373332 
>>373331
А что мешает создателю Матрицы вбить исходные данные и на их основе сделать процедурную генерацию окружающего мира, подобно современным компьютерным играм но гораздо сложней и подробней?
Аноним 04/07/16 Пнд 02:51:28 #159 №373333 
>>373332
Ты "Создатель" с маленькой буквы не пиши, грешно это.
Аноним 04/07/16 Пнд 02:57:20 #160 №373334 
Мне одному думать о естественной смерти далеко не так страшно, как о гравитационных волнах и особенно структуре всей вселенной?
Аноним 04/07/16 Пнд 03:28:15 #161 №373339 
>>373325
Да, все правильно, 2-х тонный автомобиль расходует 20 кДж на каждый метр. Это к вопросу о том, почему 5 г контактного взрыва отрывает руку.
Аноним 04/07/16 Пнд 04:07:15 #162 №373343 
>>373325
А в то, что для подъёма на 3 км (едем в горку) нужно 2 литра верится?
Попробуй как-нибудь развести огонь при помощи трения. Тогда прочувствуешь сколько энергии прячется вокруг в виде тепла.
Аноним 04/07/16 Пнд 04:57:30 #163 №373344 
>>373334
Лол, что в гравитационных волнах страшного?
Я бы понял если бы ты срался от ложного вакуума или тепловой смерти Вселенной. Или от того, что технологической сингулярности никогда не произойдёт потому что поток энергии от Солнца конечен и возникновение бесконечно сложных структур около него термодинамически невозможно.

Хотя мне лично более крипотными кажутся гипотезы о циклической Вселенной и о бесконечном количестве вселенных.

Представь, мир только коллапсировал, но тут же снова происходит Большой Взрыв. И всё точно так же как в прошлый раз. Все косяки и неудачи, которые ты стараешься забыть, тебе придётся не просто вспомнить, а пережить заново. Бесконечное количество раз. Без шанса что-либо изменить. Колесо баттхёрта, бессмысленное и беспощадное.

Но это детские шалости по сравнению с гипотезой о бесконечном количестве вселенных.
Если всё может произойти, то где-то происходят очень хорошие вещи. Где-то ты всю жизнь проводишь в эйфории, как будто упорот героином 24/7.
Но это никак не перевешивает тотального пиздеца, который происходит во вселенных где всё плохо. Там происходит такая бесчеловечная чернуха, до которой и Паланик не додумался.
На первый взгляд хорошего должно быть столько же, сколько и плохого, но нет. Асимметрия заложена в нашем восприятии. Вещей которые мы избегаем больше чем тех к которым мы стремимся. Верхний предел у счастья ниже чем у боли. Так уж хуёво устроен человек.
Аноним 04/07/16 Пнд 09:01:17 #164 №373354 
>>373334
Но ведь в них нет ничего страшного. Это где-то очень далеко столкновения чёрных дыр приводят к тому, что за доли секунды энергия, равная примерно 3 массы Солнца, целиком выделяется на гравитационные волны, на Земле от этого остаётся едва различимое сжатие/растяжение километровых стержней.
Аноним 04/07/16 Пнд 13:52:51 #165 №373450 
>>373280
>энергия вакуума равна нулю
Какое же ты тупое, безмозглое создание. Я аж порвался.
Аноним 04/07/16 Пнд 14:41:46 #166 №373469 
переход воды, к примеру, из одного физического состояние в другое. почему это обратимый процесс?

обратимый процесс с поршнем вроде бы понятен: увличим внешнее давление, P_ex в n этапов, а затем уменьшим P_ex в n этапов. при конечном n, интегральные суммы для процессов не будут равны. значит работа, q расширения не будет равна q сжатия. значит после цикла, энергия системы изменится. а так-как в поршне газ, то изменится его температура. значит часть энергии системы перейдет окружению. при n -> inf, энергия системы не изменится после цикла.

таяние льда. при 0C и давлении 1атм, три состояния воды находятся в равновесии. за любой промежуток времени, часть воды которая переходит из твердого состояния в жидкое равна части которая переходит из жидкого в твердое. при передаче тепла системе с льдом, равновесие нарушится в сторону: больше воды и газа. если система отдаст тепло, то равновесие сдвинется в сторону льда. при этом температура системы не меняется судя по эксперименту с каллориметром. так-как при передаче определенного количества тепла системе и отдаче системой того-же кол-ва тепла, температура системы не меняется, то системы не теряет энергию, значит система возвращается к прежнему состоянию. значит то процесс обратим.

короче, важно, что при цикле:
1) X энергии вошло
2) Х энергии вышло из системы
не меняется энергия системы.

я плохо понял?
Аноним 04/07/16 Пнд 14:46:46 #167 №373472 
>>373469
Я так и не увидел, как из всего этого текста должна следовать необратимость перехода воды из одного состояния в другое
Аноним 04/07/16 Пнд 15:07:06 #168 №373474 
>>373472
я и спрашивал, почему это _обратимый_ процесс. ну, как его понять.
Аноним 04/07/16 Пнд 15:20:05 #169 №373478 
>>373474
Потому что, строго говоря, необратимых в твоём смысле(можно вернуть систему в начальное состояние, забрав или добавив некоторое количество энергии, сравниваются состояния только самой системы) процессов вообще нет. Какой бы ни был процесс, определёнными внешними манипуляциями можно вернуть систему в начальное состояние.
Другое дело что в физике используется другое определение: процесс считается обратимым, если его можно обратить-провести по тем же состояниям в обратном порядке, в итоге получив исходное состояние без каких-либо внешних изменений. В таком смысле обратимых процессов в реальности вообще нет, можно лишь сколько угодно близко приближаться к теоретически обратимому процессу.
Аноним 04/07/16 Пнд 15:52:22 #170 №373483 
>>373450
>пок-пок
ась?
Аноним 04/07/16 Пнд 16:14:00 #171 №373496 
>>373483
>сидит на сцы, не знает кванты даже на поп. уровне.
Как там в начале 20 века? Не холодно?
Аноним 04/07/16 Пнд 17:43:18 #172 №373514 
Аноны, ЕЕ-дебил опять на связи. Так все-таки нахуя в EE формула эйлера? Про комплексные числы, фазоры, импедансы, активности, реактивности, и тд и тп. это я все знаю. Но все таки, зачем эйлер? Обычно же как, есть какой-то гармонический сигнал на входе и есть цепь из активностей и реактивностей. Требуется например найти напряжение на выходе. Ну и записываем по закону ома всякие jwL 1/jwC получаем импеданс, мб стоим фазор, короч получаем комплексное выражение для сигнала на выходе и из него при помощи пифагоров и тангенсов вышибаем реальное значение. И все. При чем тут ебаный эйлер?
Аноним 04/07/16 Пнд 17:57:42 #173 №373520 
>>373514
Что за ЕЕ?
Формула Эйлера нужна для того, чтобы из выражения типа R e^(iwt) получить действительное колебание, которое там происходит. Без неё ты не имеешь права вводить комплексные напряжения, токи и всё остальное
Аноним 04/07/16 Пнд 18:14:25 #174 №373530 
>>373469
>так-как при передаче определенного количества тепла системе и отдаче системой того-же кол-ва тепла, температура системы не меняется, то системы не теряет энергию
Совершенно неверно. Одинаковая температура ни о чём не говорит если материалы разные.
Аноним 04/07/16 Пнд 19:10:44 #175 №373553 
>>373520
Книжки по этой теме из раздела EE - electrical engineering.

>выражения типа R e^(iwt)
Не понимаю твоего примера. Ты имеешь ввиду запись типа jwL + 1/jwC возможна благодаря эйлеру, те складывать мы их можем? e^ там подразумевается но опускается?
Аноним 04/07/16 Пнд 19:39:58 #176 №373569 
>>373553
Я имею в виду, что без формулы Эйлера сама идея "записывать колебания комплексными числами" не работает. Ты можешь складывать что-то, умножать, делить, но это не имеет никакой связи с реальностью, пока ты не покажешь, что Acos(wt+a)-действительная часть комплексного числа Ae^(i(wt+a)).
Аноним 04/07/16 Пнд 20:20:43 #177 №373585 
>>373354
Страшно не это, а то, что ты увидишь перед смертью, если пойдёт мощная гравитационная волна.
Аноним 04/07/16 Пнд 20:23:07 #178 №373586 
>>373585
Хм... Ничего?
Аноним 04/07/16 Пнд 21:54:28 #179 №373611 
14676584691030.jpg
>>373244
Ну вот другое дело, можешь ведь когда захочешь!
Аноним 04/07/16 Пнд 21:58:35 #180 №373612 
>>373569
Почему не работает? Это же просто модель. Есть периодическое колебание от t, у него есть амплитуда и начальная фаза. Просто придумали такую запись - фазор, комплексное число в полярной форме. Можно перевести его в прямоугольную форму а+bi. Тогда по одно оси Re по другой Im. Можно делать с ними какие-то действия. Например складывать их по правилу параллелограма. А потом перевести обратно в полярную форму и оттуда взять амплитуду и фазу и записать в гармонической форме. Зачем для этого что-то знать про эйлера, про экспоненты какие-то?
Аноним 04/07/16 Пнд 22:30:22 #181 №373619 
>>373612
Смотри, как это работает.
У нас есть скажем переменное напряжение U=U0 cos(wt+a).
И это напряжение мы подаём на конденсатор ёмкостью C.
Посчитаем ток. I=dq/dt=-d(CU)/dt=-CdU/dt=Cw U0 sin(wt+a)=Cw U0 cos(wt+a+p/2) p-пи
А теперь введём комплексное напряжение и ток, так, чтобы их действительные части давали реальные напряжение и ток соответственно.
U=U0 e^(i(wt+a)); I=Cw U0 e^(i(wt+a+p/2))
Тогда можно сказать, что I=U/R, где R=1/iCw-комплексное сопротивление конденсатора, которое меняет не только модуль, но и фазу тока.
И далее оказывается, что с комплексными сопротивлениями можно работать так же, как с обычными, но вначале нужно записать входные напряжения/токи в комплексном виде(что делается через формулу Эйлера), а в конце полученные на выходе комплексные напряжения/токи нужно обратно преобразовать в действительные, по той же формуле Эйлера.

В принципе можно обойтись и без неё, и полностью работать в тригонометрической форме записи комплексных чисел, но видимо посчитали, что удобнее будет через экспоненту: она компактнее, из такого вида записи лучше видно, что фаза произведения=сумме фаз множителей, и никаких противоречий не происходит.
Аноним 04/07/16 Пнд 23:18:08 #182 №373628 
>>373569
>>373619
Я кажется понял аноны главную мысль, что без формула эйлера не было бы фазоров, так как фазор это и есть как бы экспонента. То есть входной гармонический сигнал - это как бы действительная часть эйлера (почти, только без wt). Просто мы по правилу сложения экспонент раскладываем на два множителя - частоту отбрасываем, оставляем только фазу, и это и есть фазор Aexp(jФ). Дальше мы делаем какие-то манипуляции а в конце берем Re часть, т.е. Acos(Ф) и впендюриваем туда каким-то волшебным образом обратно частоту и получаем искомый гармонический сигнал от времени Acos(wt+Ф). То есть я где-то почти понял, я и раньше как-то примерно понимал, но как часто бывает мое понимание редко достигает внутреннего чувства 100%, но процентов 90% тоже нормально.
Аноним 05/07/16 Втр 00:49:06 #183 №373640 
>>373628
хм. я так и не понял. пока думаю, что для удобства.
It turns out that the way the
angles of the polar forms of the complex numbers combine under the rules of complex
arithmetic models the effect of the components on relative phase. Because of this, in this
context, the complex numbers are sometimes called phasors .

см.: комплексное число в тригонометрической форме.

для того, что бы умножать и делить в полярной форме, не нужно доказывать через эекспоненциальную форму:
https://proofwiki.org/wiki/Division_of_Complex_Numbers_in_Polar_Form

Аноним 05/07/16 Втр 01:12:18 #184 №373649 
>>373640
Епт, ты опять засадил червь сомнения мне в голову. Я только только пришел к мысле, что мы не можем просто взять Acos(wt+ф) и размазать его по осям Х и Y без эйлера. В смысле схуяли у нас получится какая-то мнимая ось. С другой стороны, может это просто курица и яйцо, и эйлер выражает очевидное? Что если мы не будем про фазор думать как про exp(ф), допустим нет никаких экспонент. А просто есть вектор, палка, у него есть две компоненты, и две формы записи - прямоугольная a + bi и полярная. И вычисления типа сложения мы можем делать хоть в одной хоть в другой, как нам удобнее, лишь бы не в косинусах. хз, я опять в сомнении.
Аноним 05/07/16 Втр 08:11:27 #185 №373680 
>>373649
> не можем просто взять Acos(wt+ф) и размазать его по осям Х и Y без эйлера. В смысле схуяли у нас получится какая-то мнимая ось.
Мы всегда можем "угадать".
- Смотрите-ка, а ведь вот такая запись обладает теми же свойствами!
(звук математического доказательства)
Аноним 05/07/16 Втр 09:33:02 #186 №373687 
>>373649
Можешь думать про него как (r, ф), набор 2 чисел, который преобразуется при сложении и умножении определённым способом. Другое дело, что из записи экспонентой эти преобразования просто очевидны.
Аноним 05/07/16 Втр 10:02:58 #187 №373689 
>>373649
Ты просто дурачок который не может в абстракции. Сказали же тебе, одно число это координаты по иксу, другое по игреку.
Нет, хочу пощупать и потрогать
Аноним 05/07/16 Втр 14:42:47 #188 №373725 
>>372443 (OP)
Почему фотон выбивает электрон из атома при фотоэффекте, если электрон в атоме представляется волной функцией, которая говорит что электрон размазан по орбитали атома и не имеет конкретного местопложения, а фотон мало того что волна, так еще и энергия в нем помещена в волновой пакет, который тоже может быть в любом месте фотона, если фотон представлять как колебания электромагнитного поля?
Аноним 05/07/16 Втр 14:58:33 #189 №373727 
>>372443 (OP)
Сколько раз происходила конвергенция к многоклеточности?
Аноним 05/07/16 Втр 15:10:06 #190 №373730 
>>372443 (OP)
http://www.rifters.com/crawl/?p=6762

Вот тут мужик кратко пересказывает основную идею какой-то научной книги - предположение о том, что, на самом деле, это время течёт повсюду равномерно - это пространство относительно.

В общем, ваше мнение?
Аноним 05/07/16 Втр 15:34:56 #191 №373734 
>>373730
Хуйня. Пространство-время — единая конструкция, которая преобразуется как одно целое. Причём замедление времени как в движущихся обьектах, так и в гравитационном поле, были успешно подтверждены экспериментами.
Аноним 05/07/16 Втр 16:13:18 #192 №373741 
>>373734
>Пространство-время — единая конструкция
До эйнштейна тоже думали что времфя абсолютно.
Аноним 05/07/16 Втр 16:20:14 #193 №373745 
>>373741
Мало ли что до эйнштейна думали. Тогда приборов точных не было, вычислительных мощностей нормальных. Сейчас же есть точнейшие приборы, которые видят разницу в течении времени просто на разнице высот в метр. Да тот же gps был бы на больше десяти км менее точным, если бы не учитывал ОТО.
Аноним 05/07/16 Втр 16:21:17 #194 №373746 
>>373741
https://lenta.ru/news/2015/04/22/clock/
Вот
Аноним 05/07/16 Втр 16:22:37 #195 №373747 
>>373741
И что? Я не говорю, что теория относительности абсолютно правильная, но относительность времени подтверждена экспериментально, а эксперимент у нас на первом месте.
Аноним 05/07/16 Втр 16:44:50 #196 №373757 
>>373734
В гравитационном поле - согласен, в движущихся объектах - старая проблема точки отсчета, когда не ясно, что относительно чего движется со скоростью, близкой к световой.
Аноним 05/07/16 Втр 16:52:48 #197 №373761 
>>373757
>старая проблема точки отсчета, когда не ясно, что относительно чего движется со скоростью, близкой к световой.
Есть мюон, он живёт 2х10^-6 секунд в лаборатории, но, образовываясь под действием космических лучей в верхних(и не очень верхних) слоях атмосферы, долетает до земли, хотя с таким временем жизни он может пролететь не более 600 метров.
Где тут "проблема точки отсчёта"?
Аноним 05/07/16 Втр 17:23:48 #198 №373765 
>>373761
Земля движется со скоростью света - мюон покоится на месте.
Аноним 05/07/16 Втр 17:29:01 #199 №373767 
>>373765
А относительно Земли наоборот. Что теперь?
Аноним 05/07/16 Втр 17:29:35 #200 №373768 
>>373689
Ты просто петух, который не умеет читать, а кукарекает. Комплексные числа это и есть абстракция, тупой ты еблан. Вопрос был про обоснование их использования для представления гармонических колебаний. Анон выше сказал, что без эйлера эта абстракция не имеет связи с реальностью. Разобрались уже, что имеет. Про фазор можно думать просто как про полярную форму, без всяких экспонент. Проехали короч.
Аноним 05/07/16 Втр 17:31:50 #201 №373769 
>>373767
Значит, на земле время течет медленее, а у частицы - быстрее. По теории тогда частица должна была бы с одной стороны земли вообще не долетать. но мы не знаем таких подробностей, потому этот эксперимент показывает ничего.
Аноним 05/07/16 Втр 17:53:13 #202 №373773 
>>373769
Что ты вобще несёшь? Есть факт: мюоны долетают до Земли. ТО обьясняет это замедлением времени: мюоны летят с околосветовыми скоростями относительно нас, поэтому по нашим часам они живут дольше и успевают пролететь сквозь атмосферу. Теперь жду, как ты обьяснишь этот факт с позиции абсолютного времени.
Аноним 05/07/16 Втр 18:57:20 #203 №373791 
Где прочитать чисто про всю эту хуйню с корреляциями и статитсическим анализом экспериментов? ВСе эти ложно позитивные и прочее. Я всегда думал что корреляция 0.6 означает что в 60 проц случаях имеется совпадения, а оказалось нихуя.
Аноним 05/07/16 Втр 19:15:54 #204 №373800 
>>373773
ТО должна была бы также объяснять, что мюоны НЕ долетают до земли. Знаешь, что такое фальсифицируемость теории? Это когда результат опытов может опровергнуть теорию.
Аноним 05/07/16 Втр 19:28:09 #205 №373808 
>>373800
Ну вот если бы не долетали-это значило бы что замедления времени нет, или оно заметно ниже рассчитанного. Но они долетают. Последний раз спрашиваю: может ли теория абсолютного времени это обьяснить?
Аноним 05/07/16 Втр 20:26:24 #206 №373825 
>>373730
>The more dimensions the lattice has, the shorter the number of hops required to get between two points: Smolin invokes the analogy of a cell-phone network, which puts you just one step away from billions of “nearest neighbors”.
>you invoke an infinite number of parallel universes to increase the statistical odds that some of them would turn out the way ours did
По-моему мужик не очень хорошо разобрался в материале.

>It takes energy to keep those higher dimensions active, though.
>As the universe cooled, those higher dimensions collapsed; the cell network shut down, flattening reality into a low-energy mode where only those few locations adjacent in three dimensions could be considered “nearest”.
Он говорит про микроскопические измерения в духе теории струн? Почему для их существования нужна энергия?
Аноним 05/07/16 Втр 20:39:53 #207 №373833 
>>373825
Вобще было бы неплохо глянуть саму ту книгу, и проверить её выводы на соответствие реальности. А то обсуждать теорию по рецензии немного неудобно
Аноним 05/07/16 Втр 21:09:44 #208 №373844 
>>373808
Частица стоит на месте - земля летит со скоростью света. Частица мгновенно расыпается, не долетая до земли. Ферштейн, или еще раз другими словами пояснить?
Аноним 05/07/16 Втр 21:13:15 #209 №373847 
>>373844
>частица мгновенно рассыпается, не долетая до земли
А она долетает, вот в чём суть. Ну ладно, похоже никаких обьяснений я не услышу по этой теории.
Аноним 05/07/16 Втр 21:15:33 #210 №373849 
>>373844
Так Земля-то плоская, не слыхал чтоль?
Аноним 05/07/16 Втр 21:26:59 #211 №373853 
>>373847
А она долетает, что противоречит ТО, потому что на неподвижной частице время течет быстрее, чем на движещейся земле.
Аноним 05/07/16 Втр 21:33:34 #212 №373855 
>>373853
Тут работает сокращение длины. Относительно мюона отрезок "Земля-верхние слои атмосферы" движется с околосветовой скоростью, в итоге это расстояние сокращается до величины, меньшей чем 600 метров, и за 2х10^-6 секунд Земля успевает пролететь это расстояние и столкнуться с мюоном
Аноним 05/07/16 Втр 22:17:55 #213 №373875 
>>373833
>Вобще было бы неплохо глянуть саму ту книгу
http://gen.lib.rus.ec/book/index.php?md5=6F3D0EEEADFC450D1AE3BDE9C996BF07
Аноним 06/07/16 Срд 07:58:41 #214 №373956 
>>373311
>создать реальную вр
посмотрите на этого научпоп долбоеба!
люди бюджет и время на десятки лет планируют, эксперементы ставят, которые могут и ничего не дать, а манька с дивана уже приготовилась к ВР!
просто кекус-шмекус
Аноним 06/07/16 Срд 09:01:42 #215 №373957 
>>373808
Кароч, может когда ставили эксперимент, измеряемые мюоны летели почти противоположно направлению движения Земли и, значит точка соприкосновения с поверхностью переместилась ближе к мюону.
Аноним 06/07/16 Срд 09:46:41 #216 №373960 
Как наука объясняет гомофобию? Почему она только у людей? На вики какая то толстота про то что гомомфобы боятся оказаться принудительно сношенными в анус и тем самым пониженными в репутации и это выражается в агресси на всех от кого исходит такая угроза. Ну это что то совсем психоанализ. есть что то по проще?
Аноним 06/07/16 Срд 12:24:35 #217 №373972 
>>373960
Гомофобия только у тех, кто себя идентифицирует как гея - допускает возможность подобного или тайно мечтает.
У нормальных людей нет - они о тянках думают каждые 7 секунд, о геях думать некогда.
Аноним 06/07/16 Срд 12:39:30 #218 №373976 
>В 1957 году в Бостонском госпитале врачи Ян Брюэль и Джордж Олби успешно провели операцию по удалению опухоли мозга. Им пришлось удалить всю правую половину мозга пациента. К великому изумлению врачей, тот быстро поправился и не утратил своих умственных способностей. Казалось, операция никак на них не повлияла.
Почему я не могу найти никакой инфы из норм источника по этому случаю, в инглиш неете вообще ничего по этим именам нет, ну есть один но психолог и никаких операций не проводил? Это чистый рашкинский пиздёшь?
Аноним 06/07/16 Срд 12:45:39 #219 №373977 
Поясните, это правда возможно?
— Земля, как нам известно, имеет несколько одновременных вращений (вокруг своей оси, Солнца, в галактике и т. д.), и каждое из них оказывает за счет возникающей внутренней силы свое воздействие на все частицы, из которых состоит наша планета, — объясняет ученый. — При этом векторные составляющие, из которых складывается результирующая внутренняя сила частиц, непостоянны, так как изменяется расположение Земли в звездной системе, а плоскости орбит ее вращения пересекаются друг с другом. Все это, а также другие факторы вносят свои коррективы в изменения результирующих внутренних сил каждой частицы, а следовательно, и любого тела на Земле.

По словам Ерченко, периодически в определенных местах при стечении обстоятельств направление действия внутренней силы может не совпадать с привычным нам земным притяжением. Образуются так называемые «аномальные зоны», где людей (и не только) отрывает от земли, поднимает вверх, тянет в сторону какая-то «неведомая сила». Именно в такую зону и попали туристы Дятлова.
Аноним 06/07/16 Срд 13:47:06 #220 №373983 
>>373977
Первый абзац вполне логичен, второй-нет. Все те космические тела действуют на Землю и на людей на поверхности почти одинаково, отклонение для Луны и Солнца-порядка 10^-7 g, для остальных тел космоса-намного меньше, поднять с поверхности эти силы ничего не смогут
Аноним 06/07/16 Срд 13:51:24 #221 №373986 
>>373977
Это же всё рассчитать можно. Пиздец. Вот так учёный уровня /sci.
Аноним 06/07/16 Срд 14:18:38 #222 №373989 
Если бы не было гейзенберговской неопределённости, то демон Максвелла работал бы?
Аноним 06/07/16 Срд 15:41:19 #223 №373994 
>>373989
чё))) ЗА последние 7 лет 2 раза попадалась новость что реалищовали в реале димона максвела. то японцы то хохлы какие то.
Аноним 06/07/16 Срд 15:41:48 #224 №373995 
Если тупого качать ноотропами или метом, насколько умнее он будет? И действует ли лсд на клинических идиотов?
Аноним 06/07/16 Срд 15:48:40 #225 №374000 
14678093202350.jpg
Вот всплыл школьного уровня вопрос. К клеммам(U=const) подключен проводник с сопротивлением R, а к нему припаян ещё провод R1, так что выходит петля как на пикче. Будет ли по петле течь ток? Или, иначе говоря, можно ли это все заменить такой схемой?
Аноним 06/07/16 Срд 15:53:28 #226 №374002 
>>374000

1. r+r1=r общ
2. r\2 +r1= rобщ
3. это называетcя эквивалентная схема r и вытекает из из двух предыдущих примеров

ток будет течь
Аноним 06/07/16 Срд 15:55:21 #227 №374007 
>>374000
если задача олимпиадная, то да, если стандартная для даунов, то нет
Аноним 06/07/16 Срд 16:05:40 #228 №374011 
>>374002
То есть, сопротивление такой петли r+r1, да?
Аноним 06/07/16 Срд 16:05:49 #229 №374012 
>>374000
Нет, не будет. С обоих сторон петли одинаковый потенциал.
Аноним 06/07/16 Срд 16:11:24 #230 №374014 
>>374011

Да так как эта петля как ты видишь проходит в одной точке с сопротивлением r, мы получаем последовательную цепь. В петле возможно будет индуктироваться ток, если в условиях задачи у нас именно петля, в этом случае мы получаем RL но здесь уже другие рассчёты.
Аноним 06/07/16 Срд 16:27:17 #231 №374019 
>>374002
>>ток будет течь
>>374012
>>нет, не будет

Второй анон убедительнее объяснил. А вообще такие вопросы должны решаться практикой, имхо.
Аноним 06/07/16 Срд 16:30:00 #232 №374020 
14678118010740.png
>>373983
Ускорение Земли, придаваемое ей Солнцем, этой огромной ЙОБОЙ у нас под боком, уже считали недавно.
Вот уточняющая паста:

Давай посчитаем порядок величины: пик
Величина этого ускорения не более 0,00000000003 м/с^2 или 0,000000000003 g.
Не больше - посчитана только верхняя граница, при условии что вектор гравитации параллелен вектору скорости, чего на орбите никогда не бывает.
Реальная величина максимального ускорения на два-три порядка ниже, за счет того, что орбита Земли почти круговая, и вектор гравитации почти ортогонален вектору скорости, разница походу меньше градуса.





Да, Солнце на нас определенно влияет, но это влияние столь слабое, что проявляется только в макромасштабах - годовые смещения по орбите там всякие, и т.п.
Что-то более мелкое и легкое это влияние считай и не очучает вовсе.
Аноним 06/07/16 Срд 16:32:27 #233 №374021 
>>373989
Он и так работает. Неопределенность ему не мешает.
Аноним 06/07/16 Срд 16:33:23 #234 №374022 
А правда, что звёзд в нашей галактике больше чем атомов во вселенной?
Аноним 06/07/16 Срд 16:34:00 #235 №374023 
>>374022
Нет
Это платиновый вопрос?
Аноним 06/07/16 Срд 16:44:48 #236 №374027 
>>374021
Ок, переформулирую. Если бы не было неопределённости, можно было бы на его основе создать вечный двигатель второго рода?
Аноним 06/07/16 Срд 16:50:46 #237 №374031 
>>374000
Если петля именно в одной точке соединена с проводником - прямого тока не будет.
Максимум - индуктивный, если там переменка.

Если же точка не идеальная, и имеет какую-то протяженность, т.е. отрезок - тогда эта задача превращается в задачу двух параллельных сопротивлений.

Какой вариант твой - зависит от степени упоротости. Обычно для школодаунов все идеальное, так что и точка идеальная и ток постоянный - тогда тока не будет.
А кроме школодаунов тут никого и нет.
Аноним 06/07/16 Срд 17:01:10 #238 №374034 
>>374031
Петля подключается переключателем, так что точка одна => не потечет там ток.
Аноним 06/07/16 Срд 17:07:57 #239 №374036 
>>374031
На самом деле даже если там не точка, а небольшой отрезок-разницы будет немного.
Допустим, что между концами петли кусок проводника с сопротивлением r2, а вне петли остаётся 2 куска по (r-r2)/2, ну и сама петля r1.
Тогда сопротивление между концами петли r2'=r2r1/(r2+r1)≈r2(1-r2/r1), и полное сопротивление R=r-r2+r2(1-r2/r1)=r-(r2^2)/r1, то есть погрешность сопротивления имеет второй порядок малости по r2. Ну и, если полный ток Ι, то ток в петле будет Ir2'/r1≈Ir2/r1, то есть первый порядок малости.
Аноним 06/07/16 Срд 17:25:48 #240 №374041 
>>374036
>кроме школодаунов тут никого и нет
ты это дауну объясняешь, он же сам за себя пояснил
Аноним 06/07/16 Срд 17:34:34 #241 №374042 
14678156743420.png
>>374031
каких параллельных сопротивлений, ты ебанулся. основной провод - сплошной. точка его никак не разрывает. "протяженность" это точки - вертикальная а не горизонтальной. те при плохой пайке сопротивление будет в основании петли. что совершенно не влияет на тот факт, что тока в замкнутой петле тока не будет, даже если ты к ней резистор припаяешь. см пик
Аноним 06/07/16 Срд 17:36:18 #242 №374044 
>>372443 (OP)
Какой минимальной толщины должен быть медный провод чтобы я по нему при передаче 2 килоампер постоянного тока под напряжением 3 вольта на расстояние 10 метров не имел тепловых потерь?
Аноним 06/07/16 Срд 17:41:21 #243 №374048 
>>374036
Разница будет.
В одном случае тока нет, вообще.
Во втором пусть още малый, но таки есть.
Аноним 06/07/16 Срд 17:42:14 #244 №374049 
>>374044
>не имел тепловых потерь
Если у нас не сверхпроводимость-потери будут
Аноним 06/07/16 Срд 17:42:50 #245 №374050 
>>374044
Потери будут всегда.
Просто при большом диаметре площадь поверхности будет столь большая, что мощность потерь будет рассеиваться столь эффективно, что разница температур уйдет на сотые доли градуса.
Аноним 06/07/16 Срд 17:55:33 #246 №374053 
>>374050
Будут, но когда они станут КРАЙНЕ МАЛЫМИ?
Сопротивление же зависит от толщины проводника квадратично
Аноним 06/07/16 Срд 17:57:02 #247 №374055 
>>374053
5 см
Аноним 06/07/16 Срд 20:31:42 #248 №374092 
>>374055
короткоствол
Аноним 06/07/16 Срд 21:58:26 #249 №374106 
>>374055
Что то очень точное значение. С потолка взял?
Аноним 06/07/16 Срд 23:12:19 #250 №374125 
Как будет выглядеть сфера Дайсона изнутри? Есть пикчи какие или картинки? Круглый/цилиндрический хабитат будет так же нелепо выглядеть в реальности как и в играх и фильмах (например хало и интерстеллар)?
Аноним 07/07/16 Чтв 00:01:23 #251 №374132 
Что получится, если смешать калийную селитру и ацетон? Дохуя мощное топливо?
Аноним 07/07/16 Чтв 01:34:53 #252 №374143 
>>374125
Изнутри- никак, ее даже не видно изнутри.
Све Аноним 07/07/16 Чтв 02:29:50 #253 №374152 
Привет, саентач. Не захожу обычно сюда, поэтому платина, скорее всего:
Есть два стула фонарика, светят в противоположные стороны. Хоть убей, со школы не мог понять, почему скорость разбега света одного фонарика относительно света другого равна с, а не 2с. А заодно поведайте пожалуйста, почему с материальными предметами, например, с разъезжающимися поездами, такой способ нахождения относительной скорости работает, а со светом ЭМ волной - нет?
Только для совсем тупых, если можно.
Аноним 07/07/16 Чтв 02:33:14 #254 №374153 
>>374152
Я уже три раза анонам даю подобную задачу, и пока что ответа вразумительного не увидел.
Аноним 07/07/16 Чтв 02:38:39 #255 №374154 
>>374153
Бля, это ж успешных людей борда, умных, как же так?
Аноним 07/07/16 Чтв 02:50:13 #256 №374155 
>>374154
Это задача на противоречивость СТО. У меня есть серьезные подозрения, в что в рамках СТО непротиворечивого описания подобной системы нет.
Аноним 07/07/16 Чтв 04:21:56 #257 №374157 
Мясо рептилий чем-нибудь отличается от мяса млекопитающих? Почему бы не разводить рептилий для еды?
Аноним 07/07/16 Чтв 06:14:26 #258 №374161 
14678612661120.jpg
Пузыри на лужах. Почему?
Аноним 07/07/16 Чтв 07:10:21 #259 №374163 
>>374157
Черепашатина от курицы почти ничем не отличается. Разводят.
Аноним 07/07/16 Чтв 08:02:42 #260 №374164 
>>374152
Потому что формула сложения скоростей V=(v1+v2)/(1+v1v2/c^2)=(c+c)/(1+1)=c.
И это работает и для света, и для поездов. И никакой противоречивости тут нет.
Аноним 07/07/16 Чтв 08:21:10 #261 №374165 
А как работает давление воздуха? Атмосферное давление равно весу вышележащего столба воздуха. Молекулы же хаотично движутся во всех направлениях, как так получается что они давят вниз? Как так получается что они относительно стабильно давят между ними же дохуя расстояния. Чёто там с гравитацией
? Как эмитируют давление на мкс, гравитации почти что нихуя..... чё как.
Аноним 07/07/16 Чтв 08:41:35 #262 №374167 
>>374165
У газов всегда есть какое-то давление, оно вызвано тем, что молекулы газа соударяются с поверхностью. И давят они не "вниз", а "перпендикулярно поверхности, давление на которую мы измеряем". Гравитация лишь препятствует разлёту атмосферы в космос и приводит к тому, что давление(и плотность) зависит от высоты.
Аноним 07/07/16 Чтв 09:02:31 #263 №374169 
>>374167
Ну, а причём тут столб воздуха? Вот стою я на улице. Если меня посадить в герметическую коробку то как изменится давление? во прямо она захлопнется вокруг меня и вместе со мной захавает и воздух, давление упадёт? плотность же ведь не упадёт.
Аноним 07/07/16 Чтв 09:10:27 #264 №374170 
>>374169
>Ну, а причём тут столб воздуха?
Не при чём. Просто так уж договорились, называть весом силу, с которой тело давит на опору. В данном случае тело-столб воздуха, а опора-поверхность Земли, вот и говорят, что давление-вес столба воздуха, и они правы просто по определению.
Аноним 07/07/16 Чтв 10:00:03 #265 №374174 
>>374143
То есть не известно?
Аноним 07/07/16 Чтв 11:12:09 #266 №374178 
>>374152
Скорость удаления фронтов света друг от друга и так 2с. Тут нет никаких противоречий, т.к. каждый фонарик светит со скоростью света.
Аноним 07/07/16 Чтв 11:14:16 #267 №374179 
>>374174
Известно. Вокруг одна чернота, ты как будто у негра в жопе.
Аноним 07/07/16 Чтв 11:51:10 #268 №374184 
>>374178
Кстати, тут важно понимать, что "скорость удаления фотонов друг от друга" с точки зрения стороннего наблюдателя и скорость удаления одного фотона от второго с точки зрения одного из фотонов - разные вещи на самом деле.
Аноним 07/07/16 Чтв 12:43:22 #269 №374191 
Что за опыт с2 щелями и как он решается?

Есть гипотетическая ЧД и некая падающая на него материя, ввиду неких обстоятельств ЧД испустила грав. волну, и в период от А до Б материя отдалилась от ЧД ввиду этого. В этот период ЧД получила тягу?
Аноним 07/07/16 Чтв 14:05:50 #270 №374212 
>>374191
>с2 щелями
>щелями
Аноним 07/07/16 Чтв 14:10:25 #271 №374214 
>>372443 (OP)
Хочу скачать учебнгик по физике. Нахожу два разных учебника с подписью на одном полный курс и на другом лекции по физике. В чем разница между ними?
Аноним 07/07/16 Чтв 14:42:57 #272 №374222 
>>374191
Ты про это?
https://ru.wikipedia.org/wiki/Опыт_Юнга

Насчёт чёрной дыры-если ЧД пустила волну, которая оттолкнула падающее тело, значит в момент испускания этой волны сама ЧД получила импульс в противоположном направлении.
Аноним 07/07/16 Чтв 16:59:24 #273 №374249 
>>374161
капли тяжелые, плотные, высокое облако, издалека ебошат. так думаю
Аноним 07/07/16 Чтв 17:28:40 #274 №374253 
14679017202770.png
>>374164
У тебя неправильная формула. Настоящая формула: V=$^81(&cv-uv)@.@=2c. Proof me wrong.
Ну вод допустим, два поезда (1 и 2) с околосветовой проходят друг мимо друга. Они отмечают друг дружкины начала и концы относительно фиксированой точки на поезде (пусть это будет начало поезда, машинист в голове отмечает начало и конец другого поезда) - так измеряют скорость проходящего мимо поезда. Длина поездов в покое относительно покоя - L, длина поезда при околосветовой относительно покоя l=L
k , где k=sqrt(1-v^2/c^2), время по нулевой скорости будет T, то же время в поезде на околосветовой - t=T/k >> T.
Поезда головой встречаются около оси, которая находится на голове третьего поезда.
При равной длине поездов и скоростях от момента встречи голов на оси до выравнивания концов с головами свидетель увидит, что будет пройдено расстояние l/2 для поездов 1 и 2. Скорость ~с, свидетель замеряет время - T1 = T2 = l/2 / c = Lk / 2c.
Теперь мы преобразовываем это время в систему поезда номер 1: время прохождения от оси до расстояния l/2, которое при измерении относительно первого поезда (в этой системе измерения отмечаю апострофом) окажется равным l'/2 = l/2
k:
t'1 = l'/2 / c = l/2 k / c - поезд номер один замерил скорость поезда номер два длиной lk равной
v'2 = l'1/t'1 = lk / (l/2 k / c) = 2c.
Расчитаем скорость поезда номер три относительно того же поезда номер один. Для "свидетеля" мы получаем время прохода поезда номер один головой от начала до конца поезда номер три T3 = L/c.
Теперь в системе поезда номер один:
L'3 = L3k = Lk
t'3 = L'3/c = Lk/c
Скорость поезда номер три, измеренная первым поездом
v'3 = L'3 / t'3 = (L
k) / (L*k/c) = c.
Чмафки всем в этом чяте.
Аноним 07/07/16 Чтв 17:34:49 #275 №374255 
>>374253
Сууука, я забыл, что тут звездочка - служебный символ. Исправляю поебанный пост. часть:
Ну вод допустим, два поезда (1 и 2) с околосветовой проходят друг мимо друга. Они отмечают друг дружкины начала и концы относительно фиксированой точки на поезде (пусть это будет начало поезда, машинист в голове отмечает начало и конец другого поезда) - так измеряют скорость проходящего мимо поезда. Длина поездов в покое относительно покоя - L, длина поезда при околосветовой относительно покоя l=Lxk , где k=sqrt(1-v^2/c^2), время по нулевой скорости будет T, то же время в поезде на околосветовой - t=T/k >> T.
Поезда головой встречаются около оси, которая находится на голове третьего поезда.
При равной длине поездов и скоростях от момента встречи голов на оси до выравнивания концов с головами свидетель увидит, что будет пройдено расстояние l/2 для поездов 1 и 2. Скорость ~с, свидетель замеряет время - T1 = T2 = l/2 / c = Lxk / 2c.
Теперь мы преобразовываем это время в систему поезда номер 1: время прохождения от оси до расстояния l/2, которое при измерении относительно первого поезда (в этой системе измерения отмечаю апострофом) окажется равным l'/2 = (l/2) x k:
t'1 = l'/2 / c = (l/2) x k / c - поезд номер один замерил скорость поезда номер два длиной lk равной
v'2 = l'1/t'1 = lxk / ((l/2) x k / c) = 2c.
Расчитаем скорость поезда номер три относительно того же поезда номер один. Для "свидетеля" мы получаем время прохода поезда номер один головой от начала до конца поезда номер три T3 = L/c.
Теперь в системе поезда номер один:
L'3 = L3xk = Lxk
t'3 = L'3/c = Lxk/c
Скорость поезда номер три, измеренная первым поездом
v'3 = L'3 / t'3 = (Lxk) / (Lxk/c) = c.
Аноним 07/07/16 Чтв 17:54:36 #276 №374258 
>>374253
У меня как раз правильная формула, которая несложно выводится из преобразований Лоренца.
Пусть частица 1 летит вправо со скоростью v, а частица 2-влево со скоростью V. Возьмём момент, когда частицы были в одной точке, за t=0, а саму эту точку-за начало отсчёта x=0. Тогда координата второй частицы будет равна x=-Vt. Теперь перейдём в систему, связанную с первой частицей(β=v/c):
V'=dx'/dt'=d(γx-γβct)/d(γt-γβx/c)=- (-dx+vdt)/(dt-v/c^2 dx)=-(Vdt+vdt)/(dt+vV/c^2 dt)=-(V+v)/(1+vV/c^2)
Знак "-" появляется, потому что частица 2 движется в отрицательном направлении.

У тебя ошибка в начале перехода в систему поезда 1: ты считаешь время прохождения половины l, но при этом в расчёте скорости ты делишь полную l на это время.
И вобще, ты бы так неаккуратно не обращался со скоростями света, для них твоё k=0.
Аноним 07/07/16 Чтв 18:06:12 #277 №374262 
>>374161
https://www.youtube.com/watch?v=9KzKw8RkHr4
Не благодари.
Аноним 07/07/16 Чтв 18:26:16 #278 №374264 
>>374255
>поезд номер один замерил скорость поезда номер два длиной lk
Коэффициент k связывает неподвижную систему отсчёта с системой отсчёта поезда 1. Его нельзя применять, чтобы определить видимую длину поезда 2 для поезда 1.
Поезд 2 будет казаться поезду 1 более сплющенным, чем поезд 3. В два раза.
Аноним 07/07/16 Чтв 18:29:23 #279 №374265 
>>374264
На самом деле можно. Как раз из-за того, что в обоих случаях относительная скорость(1-3 и 1-2) одинакова и равна c.
Аноним 07/07/16 Чтв 18:31:05 #280 №374266 
>>374265
Нет нельзя. И она не равна с, иначе ты делишь на 0.
Аноним 07/07/16 Чтв 18:34:25 #281 №374268 
>>374266
>она не равна с
Почему?
Аноним 07/07/16 Чтв 18:36:03 #282 №374269 
>>374268
Для здоровья вредно.
Аноним 07/07/16 Чтв 18:39:01 #283 №374271 
>>374269
Аргументированно
Аноним 07/07/16 Чтв 19:13:59 #284 №374273 
14679080397010.jpg
Там гугл какую-то мёртвую пизду пиарит. Поясните кто она по масти. Нетти Стивенс
Аноним 07/07/16 Чтв 19:14:20 #285 №374275 
14679080611470.png
>>372443 (OP)
Такая задача. Нужно порассуждать. Имеется 3 неподвижные точки пространства. О, A, B. в точках A и B помещены яркие лампы( чтобы свет можно было увидеть в точке 0). расстояние между 0 и A L, а между
A и B L'. В начальный момент времени зажигается свет( мгновенная вспышка) и частица со скоростью u начинает двигаться из A в B. в тот момент когда частица приходит в B, снова вспыхивает лампа(мгновенно). Время между вспышками для частицы очевидно равно
t=L'/u так как за это время частица пройдет расстояние L'. Для наблюдателя находящегося
в точке O,промежуток времени между двумя вспышками равен
t'=(L+L')/c+L'/u-L/c=L'/u-L'/c
теперь возьмем скорость u=c/2
тогда t'=2L'/c-L'/c=L'/c
а t=2L'/c, выходит что для наблюдателя в 0 промежуток времени между вспышками в 2 раза короче.
Аноним 07/07/16 Чтв 19:18:17 #286 №374276 
>>374179
В смысле, блять? Какая чернота если в центре звезда? Или свет уже не от звезд исходит?
Аноним 07/07/16 Чтв 19:18:17 #287 №374277 
>>374275
а если взять u=c, то t'=0, т.е вспышки произойдут в одновременно. кек

Аноним 07/07/16 Чтв 19:23:30 #288 №374278 
>>374275
Не "для наблюдателя О промежуток времени короче", а "наблюдатель О увидит вспышки с меньшим интервалом"
И нет, этот интервал посчитан неправильно. Первая вспышка будет наблюдаться через L/c после начала, вторая-через L'/u+(L+L')/c, в итоге разность Δt= L'/u+(L+L')/c-L/c= L'/u+L'/c.
Так что в итоге вспышки будут наблюдаться с большим интервалом.
Аноним 07/07/16 Чтв 19:24:22 #289 №374280 
>>374264
Откуда два? Почему не три?
>>374258
>У тебя ошибка в начале перехода в систему поезда 1: ты считаешь время прохождения половины l, но при этом в расчёте скорости ты делишь полную l на это время.
Нет ошипки. Это наблюдение со стороны свидетеля прохождения мимо наблюдаемых свидетелем точек, поскольку второй поезд не неподвижен, k=sqrt(1-v^2/c^2) не имеет смысла для v>c, как ты уже заметил (помашем ручкой мнимым числам).
В преобразовании лоренца имеет место нечто, отсутствующее в моем уравнении t' = t/sqrt(1-v^2/c^2), а именно, оно вдоль одной оси принимает вид t' = (t - (v/c^2)x) / sqrt(1-v^2/c^2), что совпадает с моим разве что при x -> 0. Время замедляется по мере удаления от оси? Бг-г-г.
Теперь возьмем твой расчет V'=-(V+v)/(1+vV/c^2) и β=v/c -> v = βc, который при V=v принимает форму
V' = -(2βc)/(1+β^2)
Теперь сравниваем со скоростью неподвижной точки v = -βc :
-(2βc)/(1+β^2) <> -βc
-2 / (1+β^2) <> -1
Для β < 1 получаем β^2 < 1; (1+β^2) < 2; 2/(1+β^2) < 1, тогда модуль
[V'] < v
то есть, движущийся навстречу поезд движется медленее, чем покоящийся на месте.
Аноним 07/07/16 Чтв 19:26:51 #290 №374281 
>>374258
>2/(1+β^2) < 1
Пардон, обосрамус. Конечно же 2/(1+β^2) > 1, тогда движущийся навстречу поезд движется на крайне малую величину быстрее, чем покоящийся.
Аноним 07/07/16 Чтв 19:33:27 #291 №374282 
>>374278
Действительно больше, тогда если взять u=c, то t'=2t
Аноним 07/07/16 Чтв 19:38:14 #292 №374285 
>>374280
>Нет ошипки.
t1=l/2 /c
v=l/t1
Взято из твоего же поста, штрихи опущены.
>В преобразовании лоренца имеет место нечто, отсутствующее в моем уравнении t' = t/sqrt(1-v^2/c^2)
Очень плохо, что у тебя этого нет
>Время замедляется по мере удаления от оси? Бг-г-г.
Увы, но так и есть. Глупо использовать половину формул сто, и игнорировать вторую половину
>Теперь возьмем твой расчет V'=-(V+v)/(1+vV/c^2) и β=v/c -> v = βc, который при V=v принимает форму
>V' = -(2βc)/(1+β^2)
Да, так и есть
>Теперь сравниваем со скоростью неподвижной точки
Ну давай. Подставь V=0. Получишь V'=-v, относительно движущейся частицы частица, которая покоилась в лабораторной СО, будет двигаться со скоростью v в противоположном направлении.
А ещё открою тебе секрет: вся СТО основана на том, что скорость света-инвариант, и если в одной системе у частицы скорость равна с, то и во всех остальных системах она тоже равна с, и если ты получил другой результат, пользуясь формулами СТО-значит у тебя ошибка.
Аноним 07/07/16 Чтв 19:47:24 #293 №374286 
И добавлю
>>374281
>2/(1+β^2) > 1, тогда движущийся навстречу поезд движется на крайне малую величину быстрее, чем покоящийся.
Почему на крайне малую? Разность скоростей (2-1-β^2)/(1+β^2)=(1-β^2)/(1+β^2), и это ещё умножить на βc, то есть на скорость.
Аноним 07/07/16 Чтв 20:10:50 #294 №374289 
>>374286
(1-β^2)/(1+β^2)>>374264
>Коэффициент k связывает неподвижную систему отсчёта с системой отсчёта поезда 1. Его нельзя применять, чтобы определить видимую длину поезда 2 для поезда 1.
Здесь у нас возникают ПРОБЛЕМКИ. Почему свидетель может видеть поезда проходящими мимо указанными мной опорными точками, точками, имеющими нулевую скорость, эти точки расположены на нуле и l/2 для поезда два, и 0 и L для поезда 3? Поезд номер 2 может находиться своими частями в один момент времени в разных точках пространства?
>>374285
>вся СТО основана на том, что скорость света-инвариант, и если в одной системе у частицы скорость равна с, то и во всех остальных системах она тоже равна с, и если ты получил другой результат, пользуясь формулами СТО-значит у тебя ошибка
Не ошибка, а противоречие. Мы об этом и ведем речь.
Аноним 07/07/16 Чтв 20:24:06 #295 №374291 
>>374289
Так нет никаких противоречий. Формула релятивитского сложения скоростей даёт c, преобразования Лоренца дают формулу сложения скоростей, всё сходится, только какой-то аноним, отбрасывающий преобразование времени потому что "Бг-г-г", и получающий при этом другой результат, жалуется на противоречия.
Аноним 07/07/16 Чтв 20:24:50 #296 №374292 
>>374184
Не важно. Фотоны друг друга не увидят при всем желании.

А если считать гипотетическки в системах отсчета фотонов, то их скорость будет совсем не световой.
За 14 лярдов лет для фотона прошло около 3 секунд по его часам. Значит замедление времени для него 14 лярдов • 365,24 • 24 • 3600 / 3 = 147264768000000000 раз.
Значит видимая для фотона скорость другого фотона, с учетом замедления времени, составит 147264768000000000 световых: за одну секунду по часам фотона второй фотон от него сьебал на 14/3 лярдов световых лет, нихуя ж себе прыть, да?
Аноним 07/07/16 Чтв 20:32:39 #297 №374293 
>>374276
Пиздец долбоеп.
Определись уже, что смотреть собрался, звезду или сферу.
Звезду ты увидишь. Но сферу - нет, она идеально черная, ты даже не поймешь что там что-то есть, даже радары не вернут сигнал, просто космос без звезд, черный-черный, а позади, в центре сферы, висит одна-единственная звезда.
Но если попытаешься сьебать от звезды - со всего размаху мордой долбанешься об поверхность сферы.

Ночью в комнате без света если тебе переебут по щщам, ты даже не увидишь кулак - только почувствуешь удар. Так и со сферой дайсона - ты ее не увидишь, пока не долбанешься об нее - только на ощупь можно найти ее поверхность. И это не смотря на то, что со спины тебе подсвечивает нехилых размеров фонарик в виде звезды.
Аноним 07/07/16 Чтв 20:34:14 #298 №374294 
>>374178
Но скоростей больше с не существует, как гласит школьная программа. Вот в чем соль.
>>374164 >>374253 >>374255 >>374258 >>374280 >>374281 >>374285 >>374289
Что непонятного во фразе "для совсем тупых"? Я рад за вас двоих, что нашли хорошую возможность умственно поупражняться, но я из этого так ничего толком и не понял.
Что за формулы? Поезд А едет на север со скоростью 50 км/ч, а Б - от него на юг со скоростью 60 км/ч => скорость их удаления равна, тадам, 50+60=110 км/ч. Почему со светом не так?
>>374184
Ну вот смотри, если я сижу на одном пучке света (именно пучке, а не единичном фотоне, который от наблюдения и взаимодействия сам меняется как-то квантово), достаю супер-ментовской-радар, то я вижу, что второй пучок (или его наездник, хз) улепетывает от меня с 2с?
Ну ладно, такого радара быть не может, так как его волна тоже максимумом скорости имеет с, поэтому я направляю его на фронт (или как там назвать это) второго пучка и... не вижу его? То есть, я не могу напрямую (не по моему охлождению или нагреву окрестностей) регистрировать ЭМ излучение, которое исходит от меня? Какой-то брет.
Аноним 07/07/16 Чтв 20:35:29 #299 №374295 
>>374291
Описываемая мной проблема заключалась в том, что точки пространства оказываются ВНЕЗАПНО искаженными на какие-то величины, отличные от обычного L' = γL и t' = t / γ. Мы так не договаривались. То есть, проход мимо неподвижных 0 и l/2 каким-то образом становится отличным от прохода мимо околосветового поезда, который проходит через эти же точки в эти же моменты времени.
Аноним 07/07/16 Чтв 20:42:31 #300 №374296 
>>374294
>Что за формулы? Поезд А едет на север со скоростью 50 км/ч, а Б - от него на юг со скоростью 60 км/ч => скорость их удаления равна, тадам, 50+60=110 км/ч. Почему со светом не так?
Потому что на самом деле не V'=v+V, а V'=(v+V)/(1+vV/c^2). И для поездов тоже именно эта формула верна. Другое дело, что множитель 1/(1+vV/c^2) для скоростей поездов и вобще для всех скоростей, с которыми мы встречаемся в жизни, неотличим от единицы.
Аноним 07/07/16 Чтв 20:43:45 #301 №374297 
14679134258350.png
>>374294
Со светом не так, потому что он движется на скоростях. где даже на глаз заметно замедление времени.
Пик - уже на 0,2 световых начинает сильно играть замедление времени, а после 0.9 световых - вообще пиздец начинается, разница в скорости в доли процента, а во времени - сотни раз.
Аноним 07/07/16 Чтв 21:02:06 #302 №374298 
Может кто-то популярно объяснить за пространство-время? Что это?
Аноним 07/07/16 Чтв 21:06:38 #303 №374301 
>>374295
Давай я посчитаю за тебя, а ты скажешь, что тебе не нравится.
Итак, поезд 1-направо, поезд 2-налево, скорости обоих v.
Для второго поезда можно написать зависимость координаты от времени: x=-vt+x0
Перейдём к системе первого поезда:
x'=y(x-bct)=y(-vt+x0-vt)=y(x0-2vt)
t'=y(t-bx/c)=y(t+v^2/c^2 t)
Ну и скорость V'=dx'/dt'=-2v/(1+v^2/c^2).
Где тут должно быть противоречие?
Аноним 07/07/16 Чтв 21:08:58 #304 №374303 
>>374298
Никто не знает... Но оно есть, и часть свойств известна, и понятно что пространство и время - это одна и та же хрень.
Аноним 07/07/16 Чтв 21:18:02 #305 №374305 
>время прохождения от оси до расстояния l/2, которое при измерении относительно первого поезда (в этой системе измерения отмечаю апострофом) окажется равным l'/2 = (l/2) x k:
Поезд 1 считает, что сам он длины L, а ему навстречу со скоростью v12 = (v1+v2)/(1+v1v2/c^2)
едет коротенький поезд. Перрон несётся назад, короткий поезд надвигается ещё быстрее. К моменту когда машинист поезда 1 видит конец сплющенного поезда 2 перрон успевает отъехать на l/2.
На пике сверху картина глазами 3-го поезда, ниже глазами первого (он красный).
Аноним 07/07/16 Чтв 21:18:38 #306 №374306 
14679155188140.png
>>374305
Присовокупляю пик
Аноним 07/07/16 Чтв 21:32:21 #307 №374308 
>>374289
>Здесь у нас возникают ПРОБЛЕМКИ.
Изложи суть проблем чётче. Падежи не сходятся, трудно вникать.
Аноним 07/07/16 Чтв 21:48:08 #308 №374309 
>>374296
Хорошо, пусть формула будет таковой (хотя не вижу логически ни одной причины не использовать тупо просто сумму, и не понимаю, зачем этот довесок со скоростью света, когда нет речи про макромасштабы).
Прошу тогда подсказать, где доступно написано, почему формула именно такова. Я же обыватель - вон у меня до сих пор полыхает от того, что я не учитываю скорость света, когда на тротуаре обгоняю другого человека, но просто сравниваю скорости.
>>374297
Я готов с этим согласиться, но мне очень трудно это понять. Тоже хочу попросить статейку (не уверен, что мой моск осилит целую книгу без подготовки).
>>374306
Вот да, причину этого явления бы узнать. Я так понял, оно заключается в том, что с изменением точки зрения перемещается равноудаленная от улетевших пучков света точка?

Выражаю благодарность анонимным собеседникам - благодаря вам почувствовал зайчатки желания прикоснуться к хардкорному знанию.
Аноним 07/07/16 Чтв 22:01:25 #309 №374314 
>>374309
Короче вот тебе краткий гайд по СТО.
Первое-принцип эквивалентности-все физические явления во всех инерциальных системах отсчёта(ИСО) проходят по одинаковым законам. Этот принцип не выводится ниоткуда и проверяется экспериментально, пока что нарушений не обнаружено.
Из этого принципа следует, что скорость света, измеренная любой ИСО, одинакова. То есть например измеряя скорость света, выпущенного фарами автомобиля, который едет мимо нас, мы должны получить ту же скорость, что и для неподвижной фары.
Из постоянства скорости света и однородности/изотропности пространства выводится инвариантный интервал между двумя событиями s: s^2=x^2+y^2+z^2-(ct)^2, где x, y, z-разности координат событий, а t-разность между их временами. При изменении системы отсчёта интервал не меняется.
Из постоянства интервала выводятся преобразования Лоренца, которые показывают, как меняются координаты и время при смене системы отсчёта.
И уже из этих преобразований выводится формула сложения скоростей, замедление времени и сокращение длины.
Аноним 07/07/16 Чтв 23:21:21 #310 №374325 
>>374222
Да спасибо.
Аноним 08/07/16 Птн 00:20:59 #311 №374331 
>>374293
А вопрос про хабитат ты опускаешь? К тому же с каких пор сфера это абсолютно черное тело?
Аноним 08/07/16 Птн 02:46:03 #312 №374337 
>>374331
Это же сфера дайсона - ее задача поглощать максимум энергии звезды. Для этого она должна быть идеально черной. Очевидно же.

Сфера дайсона строится, когда цивилизации надоело играть с карманными термоядерными реакторами, вот звезду и превращают в ядро оче большого реактора, обкладывая ее со всех сторон энергопоглотителями.

Задача звезды - просто производить энергию, ее свет никого не интересует.
Потом собранную энергию, вместо того чтобы рассеивать по космосу, концентрируют в узкий пучок и отправляют куда требуется цивилизации - на какую-нибудь научную базу, требующую оче много энергии, или жилую планету.

Расстояние потребителя от сферы дайсона значения особо не имеет - концентрированный поток энергии дойдет до любой сколь угодно удаленной точки, просто спустя какое-то время.
А время для цивилизации подобного уровня значения не имеет - там уже оперируют стабильными процессами, стратегия развития рассчитывается на века вперед.
Аноним 08/07/16 Птн 03:20:51 #313 №374338 
>>374309
Скорость света вытекает из оче простой идеи: энергия не может существовать сама по себе.

Т.е. фотоны - это радиоволны, а радиоволны в чем-то движутся. Неявно подразумевается некий эфир - среда распространения волн. Мы называем эту среду пространством.

Мы так и не поняли, что такое пространство - его не пощупать, ни набрать в карман.
Сперва вообще думали что это просто пустота.
Но потом начали замечать проявления взаимодействия материи с пространством. И выяснили некоторые его свойства.

Во первых, самое известное свойство пространства - оно постоянно расширяется.
Без модели пространства по иному обьяснить разлет галактик трудно.
Так что пришлось признать, что пространство существует, и даже играет существенную роль в жизни и устройстве мира.

Далее обнаружили еще одну забавную штуку - оказывается вакуум не вакуум! На мелких уровнях там откуда-то постоянный энергошум.
Это свойство известно как эффект казимира: две пластинки, сведенные близко, притягиваются, не смотря на отсутствие внешних сил.

Потом, с приходом ТО, обнаружили еще одну забавную штуку: оказывается пространство и время связаны так, как будто это два параметра одного обьекта, находящеся в прямой зависимости друг от друга. В итоге пространство стали называть пространством-временем, ибо все это одна на деле и та же хрень.

Ну и сам эфир как среда распространения радиоволн подозревался с самого начала. Даже до сих пор в радиолюбительстве используется вместо слова "пространство" именно "эфир" - это осталось с 19 века, научники пошли в одну сторону, и запилили более подробную ТО, технари - в другую, они ТО не учитывают, они практики, и там ТО пока не проявляется - не те расстояния, так что не стали забивать голову новыми тонкими теориями: эфир и эфир - это понятно всем, свойства крайне простые.


Итак, в итоге мы пришли к некой субстанции, пространству. Оно имеет некий набор свойств, т.е. существует обьективно, и хоть напрямую его замерить нельзя, но косвенно, по влиянию на материю - можно. Так вообще можно рассчитать полный набор его свойств, но пока никто этого не делал.
Пространство материально, не смотря на то, что мы его пощупать не можем - оно взаимодействует с материей,и мы это наблюдаем, а как известно если что-то взаимодействует с материей - оно обязано быть материальным.

Мы нашли еще несколько похожих на пространство субстанций, материальных, но которые тоже нельзя пощупать - мы назвали их полями.
Поля - это поле взаимодействия материи и энергии. Т.е. вне полей такого взаимодействия нет, оно есть только в полях, причем параметры взаимодействия строго определяются свойствами источника поля.
Т.е. поле - это как арена со своими законами, она имеет свод правил, имеет размеры, имеет управляющий центр, задающий эти правила, и на ней взаимодействуют материальные участники, обмениваясь энергией.
У разных полей свои наборы свойств.
Так что пространство можно рассматривать как некое поле.

В то же время у пространства есть одно свойство, которое у полей пока не обнаружено - это скорость взаимодействия.
Возможно у всех полей она одинакова или близка, поэтому мы не можем отделить его от пространства.
А может у полей действительно нет такого свойства, и все они используют базовое свойство пространства - тогда пространство это некое суперполе, внутри которого формируются локальные поля разных видов.

Но это все не существенно. Главное, что ограниченная скорость взаимодействия подразумевает, что пространство в какой-то степени дискретно, т.е. волна передается как некая цепь мелких-мелких взаимодействий, именно это и ограничивает скорость волны - большое число взаимодействий.
Очевидно, что скорость волны в среде с подобными свойствами очень сильно зависит от параметров этих взаимодействий: замедление взаимодействий всего на чуть-чуть резко уменьшает скорость волны.
И это действительно так - свет в разных средах имеет разную скорость, т.к. параметры сред различны.
Свет в одной среде тоже имеет разную скорость, т.к. среда неоднородна, либо подвергается воздействиям. Т.е. влияя на кристалл, через который пропускается свет, меняются параметры кристалла, от чего резко меняются параметры света - такие кристаллы сейчас используются широко.

Это же свойство распространения волн, а именно зависимость от параметров среды, позволяют существовать эффекту допплера (отраженный сигнал модулируется параметрами отражателя), позволяет существовать замедлению времени (от скорости движения обьекта меняется частота взаимодействия с пространством).

Это же свойство наглядно видно на простом водоеме: мы знаем как распространяются волны в воде, даже школьники это знают, это вбивают в голову всем.
Свет - это волны среды, для сравнения представь вместо света волны на воде, например на море: с какой бы ты скоростью не двигался относительно воды, твои волны двигаются со скоростью относительно воды, а не тебя, также и свет движется относительно среды а не тебя, и на твою скорость ему начхать.
Также на этой аналогии отлично видно эффект доплера:
1 - представь два неподвижных острова, если начать бить по воде с частотой 1Гц на одном острове, волны с такой же частотой докатятся и до другого, и будут приняты на частоте 1Гц
2 - представь себе неподвижный остров, и движущийся корабль. Если начать бить на острове по воде с частотой 1Гц, эти волны будут восприниматься на корабле в зависимости от его радиальной (относительно острова) скорости движения: если корабль движется к острову - он будет наскакивать на волны чаще, чем они идут, и принимаемая частота будет больше 1Гц, если корабль удаляется - он будет убегать от волн, и принимаемая частота будет ниже 1Гц, если корабль превысит скорость волн в воде - на нем волны вообще не будут приняты, их частота для корабля будет равна 0Гц, но если корабль превысит скорость волн по направлению к острову - частота принимаемых волн будет кратна превышению скорости, но волны будут приниматься всегда, т.к. даже если бы корабль двигался с бесконечной скоростью, частота просто стала бы бесконечной, но никак не нулевой.
3 - представь себе два подвижных корабля, если начать бить по воде на одном корабле, частота приема на другом будет зависеть от радиальной относительной скорости этих кораблей друг к другу. Т.е. частота будет изменена дважды: мы бьем с частотой 1Гц на одном корабле, кораблю наскакивает или убегает от воды, и частота излученных волн (которая будет приниматься на неподвижном острове) будет выше или ниже 1Гц, но никак не 1Гц, а при приеме на втором корабле, частота еще раз будет искажена - корабль вновь наскакивет или убегает от волн, и еще раз увеличивает/уменьшает принимаемую частоту.
Вот как-то так это и работает.
Аноним 08/07/16 Птн 05:50:01 #314 №374344 

>>374303
А какие свойства-то известны?
Аноним 08/07/16 Птн 06:58:21 #315 №374345 
Как получить жидкость с трупным запахом (запахом тухлятины) для отпугивания гопоты, которая облюбовала лавочку у подъезда?

Желательно чтобы запах держался несколько часов и бы не очень сильным (чтобы не тянуло в окна первого этажа)
Аноним 08/07/16 Птн 08:07:52 #316 №374346 
>>374338
>Без модели пространства по иному обьяснить разлет галактик трудно
Если СТО ложно, то объяснять может оказаться нечего. Может получиться, что красное смещение вызвано не ускорением. Дальше, под сомнением может оказаться теория большого взрыва.
Аноним 08/07/16 Птн 10:29:25 #317 №374373 
>>374346
>Если СТО ложно
Ты хотел сказать ОТО?
>Может получиться, что красное смещение вызвано не ускорением.
Эйнштейн поправит лямбда-член и пойдёт дальше.
Аноним 08/07/16 Птн 10:44:23 #318 №374375 
Почему я чихаю, когда смотрю на солнце?
Аноним 08/07/16 Птн 10:55:15 #319 №374376 
>>374373
>Ты хотел сказать ОТО?
Поясни, каким образом ОТО объясняет красное смещение? Это ведь основной аргумент в поддержку расширения вселенной, не так ли?
Аноним 08/07/16 Птн 11:00:21 #320 №374378 
>>374375
Типичный солнцефоб. Таких как ты надо изолировать от общества.
Аноним 08/07/16 Птн 11:10:42 #321 №374380 
>>374376
"Образование космологического красного смещения можно представить так: рассмотрим свет — электромагнитную волну, идущую от далёкой галактики. В то время как свет летит через космос, пространство расширяется. Вместе с ним расширяется и волновой пакет. Соответственно, изменяется и длина волны. Если за время полёта света пространство расширилось в два раза, то и длина волны и волновой пакет увеличивается в два раза."
Аноним 08/07/16 Птн 11:12:46 #322 №374381 
>>374380
Откуда ты взял, что пространство расширяется, скажи мне? Я почему и говорил про большой взрыв.
Аноним 08/07/16 Птн 11:23:01 #323 №374383 
>>374381
Наблюдается красное смещение излучения далёких галактик. Если ОТО верна, то это красное смещение может иметь одно из 3 обьяснений: гравитационное, допплеровское(связанное с удалением галактик от нас) и связанное с расширением пространства.
Первый вариант не подходит, поскольку красное смещение не зависит от массы галактики, второй вариант означал бы, что по какой-то случайности все далёкие галактики улетают от нас со скоростями, пропорциональными расстоянию, и третий вариант говорит, что пространство расширяется.
Теперь, если написать уравнения ОТО для Вселенной, и выбросить ламбда-член, то эти уравнения как раз и дадут расширение пространства, и логично отождествить это расширение с причиной красного смещения.
Аноним 08/07/16 Птн 12:03:05 #324 №374384 
>>374383
>третий вариант говорит, что пространство расширяется
То есть, эфир расширяется - тела остаются на месте, так?
Аноним 08/07/16 Птн 12:45:16 #325 №374389 
>>374384
>эфир
Нет
Аноним 08/07/16 Птн 13:11:05 #326 №374395 
>>374389
Ну ты же говоришь
>пространство расширяется
что это значит? У боженьки на суперкомпьютере оси мира постоянно масштабируются?
Аноним 08/07/16 Птн 13:28:07 #327 №374396 
>>374395
Это значит, что в сопутствующей системе координат(привязанной к усреднённому однородному и изотропному веществу в каждой точке) расстояние между двумя точками с заданными координатами растёт со временем. Также, в случае замкнутой модели Вселенной, растёт её полный обьём.
Аноним 08/07/16 Птн 13:47:56 #328 №374398 
Насколько херово на потомство влияет тот факт, что их родители сводные брат и сестра?
Аноним 08/07/16 Птн 13:48:51 #329 №374399 
Почему работает частотное разбеление?
Почему сигналы разных частот не мешают друг другу?
Как это обосновывается, если мыслить логически?
Аноним 08/07/16 Птн 13:57:13 #330 №374400 
Пацаны, есть кто может в опыт Аббеля-Порте растолковать, а то не могу нормально его расписанным найти. Ну и ещё про дифракционный предел оптики, только суть этого дерьма, а не математический аппарат вывода.
Аноним 08/07/16 Птн 14:42:18 #331 №374405 
>>374399
А почему вдруг они должны мешать? Приёмник ловит узкий диапазон частот, сигнал, частота которого входит в этот диапазон, улавливается приёмником, остальные-нет. Или ты о чём-то другом?
>>374400
Можешь кинуть ссылку на этот опыт? Гугл молчит.
По поводу дифракционного предела: есть дифракция-явление огибания светом препятствий. Это приводит к тому, что луч света-на самом деле расходящийся пучок, и сфокусировать его в точку невозможно.
Аноним 08/07/16 Птн 14:48:36 #332 №374406 
>>374405
Это да, в гугле трудно найти. Ах да, я ещё и даун, неправильно написал, лол.
http://mash-xxl.info/info/192000/
Аноним 08/07/16 Птн 15:26:04 #333 №374409 
>>374406
www.webpoliteh.ru/subj/optika/361-6-1-osnovnye-ponyatiya-fure-optiki.html
Тут я нашёл более подробное описание опыта и теории, которая его обьясняет.
Насколько я понял, суть такова: мы освещаем сетку когерентным светом, а за сеткой ставим линзу. В фокусной плоскости линзы будет система пятен, которая на больших расстояниях переходит в изображение самой решётки(как и в геометрической оптике). При этом, перекрывая все пятна, кроме горизонтальной полоски, мы получим изображение без горизонтальных линий, и наоборот.
Почему так происходит-потому что оставшаяся горизонтальная полоса пятен такая же, какая бы получилась, если бы на входе у нас была решётка без горизонтальных линий, значит и изображение, полученное из них, будет без этих линий.
Аноним 08/07/16 Птн 15:30:50 #334 №374410 
>>374405
Но что такое частота? Как именно приёмник различает сигналы различных частот и отлеляет один от другого?
Аноним 08/07/16 Птн 15:36:10 #335 №374411 
>>374409
Спасибо, анон. Почитал, грубо говоря, этот опыт доказывает, что фурье-преобразование это не выдумка против костылей, а так и есть.
Аноним 08/07/16 Птн 15:39:11 #336 №374412 
>>374410
Это уже радиоэлектроника, да? Ну вся фишка в том, что приёмник настраивается на нужную частоту, где в итоге она у нас резанирует с нужной частотой. Регулируется параметрами катушки и конденсатора, грубо говоря.
Аноним 08/07/16 Птн 15:46:17 #337 №374415 
>>374398
Если биологического родства нет - можешь жениться.
Аноним 08/07/16 Птн 15:52:11 #338 №374416 
>>374410
За счет резонанса.
Он просто содержит деталь, в которой не могут существовать волны других частот, кроме той, на которую деталь настроена.

Наглядно принцип можешь пронаблюдать на примере обычного карандаша - попробуй им быстро помахать туда-сюда. И смещай точку, за которую его держишь.
Легче всего махать только в одной точке - в точке резонанса. Чем дальше точка от точки резонанса - тем труднее им быстро махать.
Аноним 08/07/16 Птн 16:12:12 #339 №374418 
>>373329
Допустим тебе надо смоделировать некоторый объект который состоит из 100 атомов, значит компьютер должен хранить как минимум информацию о 100 частицах. Пусть у частиц будет только один параметр(их насамом деле гораздо больше) и о принимает значение 0 или 1, тогда компьютеру необходимо для описания этого объекта иметь минимум 100 бит памяти, а 1 бит памати реализуется технически огромным количеством атомов. Значит для описания хотя бы земли, нужен будет компьютер размом с галактику(может больше может меньше, но точно больше чем солнечная система).
Аноним 08/07/16 Птн 16:28:18 #340 №374422 
14679844987550.jpg
>>374416
Аноним 08/07/16 Птн 17:10:00 #341 №374423 
>>374418
Это называется энтропией.

Энтропия - количество состояний какой-либо системы.
Каждый бит удваивает энтропию.

Так можно посчитать минимально необходимый размер структуры, описывающей некую систему или обьект.

Но важен не столько размер, сколько затраты энергии: для изменения каждого бита требуется затратить определенное количество энергии.

Таким образом энтропия дает представление о минимальном количестве памяти и минимальной энергетической мощности, требуемым для просчета какого-либо обьекта или системы.
Аноним 08/07/16 Птн 17:28:43 #342 №374424 
>>374415
Лол, но биологическое родство как раз есть у всех нас. Ну разве что между медной и железной жизнью оно дальше чем между тобой и туфелькой, но всё равно родня.
Аноним 08/07/16 Птн 18:18:56 #343 №374429 
>>374424
Этому больше не наливать
Аноним 08/07/16 Птн 21:17:21 #344 №374472 
>>374338
Спасибо за текст. Содержательным для меня стала часть про волны на воде - понимается проще.
Аноним 08/07/16 Птн 22:15:59 #345 №374493 
>>373344
Какой охуенный пост. Спасибо анон.
Аноним 08/07/16 Птн 23:42:57 #346 №374506 
На чём строятся теория струн и теория петлевой квантовой гравитации, если переносчик гравитационных взаимодействий, гравитон, до сих пор не найден?
Аноним 09/07/16 Суб 02:27:58 #347 №374526 
>>374493
это из футурамы сюжетик
Аноним 09/07/16 Суб 03:29:29 #348 №374530 
>>372443 (OP)
Мы пьем воду ради того чтобы насытить себя водородом?
Ну а хуле атом кислорода и 2 водорода. Кислород мы и так получаем, получается мы пьем ради водорода?
Аноним 09/07/16 Суб 05:50:24 #349 №374541 
>>374530
Нет. Атомы и молекулы - пример синергии.
Молекулы имеют свойства, сильно отличающиеся от исходных атомов.

Вода например - это отличный растворитель.
Ни кислород ни водород такими свойствами не обладают.
А еще вода несжимаема, умеет в лед, имеет точку кипения в 100 градусов. Кислород и водород так не умеют.

Так что мы пьем не ради водорода, а ради в оды - у нас вся биохимия замешана на воде, без воды биохимия стопорится, и мы дохнем.

Из-за этого мы жестко завязаны на концентрацию и состав воды.
Мы на 80% состоим из воды, но потеря всего 10% (или 20?) смертельна - настолько тонкая биохимия.
Также любой загрязнитель в воде также грозит смертью - очень многие вещества растворяются в воде, и многие из них разрушают биохимию.
Электронике это знакомо лучше - там даже присутствие малейших следов солей в воде разрушает электронику.
Аноним 09/07/16 Суб 07:16:15 #350 №374543 
>>374541
а еще там диполи какие-то возникают, неравное мерное расположение + - в атоме, типа как яйца знаете обычно по обе стороны, а иногда оба сползают влево и приходится их поправлять. и типа из-за этих диполей дохуя важных свойств. я заметил, что когда начальную химию изучаешь, там все просто, молекулы-атомы, выпил h выпил o, а потом оказывается все как непросто!
Аноним 09/07/16 Суб 12:16:53 #351 №374569 
С точки зрения молекулярной физики объясняются химические реакции?
Аноним 09/07/16 Суб 14:35:11 #352 №374595 
Какой толщины и на чём лучше делать две прорези (красный лазер) в опыте Юнга
Аноним 09/07/16 Суб 15:55:37 #353 №374606 
>>372443 (OP)
Прочитал про Фейнманиум и у меня возникла пара вопросов:
I. Уменьшается ли радиус/плотность вероятность 1s-оболочки с увеличением порядкового номера атома из-за:
1) В следствии увеличения числа протонов, и тем самым увеличивается заряд ядра и притяжение двух 1s-электронов становиться сильнее?
2) В следствии увеличения числа электронов на верху, которые создают почти такой же(разница в 2 е) заряд как у ядра, только отрицательный и заставляя два 1s-электрона отталкиваться от них ближе к ядру, т.е. верхние электроны вдавливают два 1s-электрона ближе к ядру?
II. "Скорость" электрона(если она у него есть в режиме атомный орбитали) зависит от заряда ядра или от расстояния электрона до ядра?
Аноним 09/07/16 Суб 16:29:54 #354 №374614 
>>374606
>I. Уменьшается ли радиус/плотность вероятность 1s-оболочки с увеличением порядкового номера атома из-за:
>1) В следствии увеличения числа протонов, и тем самым увеличивается заряд ядра и притяжение двух 1s-электронов становиться сильнее?
Да, этот овет правильный. Второй ответ не подходит, поскольку "верхние электроны" не находятся "вверху", а размазаны по поверхности вокруг атома. Ещё учти, что само ядро при росте атомного номера растёт
>II. "Скорость" электрона(если она у него есть в режиме атомный орбитали) зависит от заряда ядра или от расстояния электрона до ядра?
В модели Бора скорость зависит от радиуса и заряда ядра(как (Z/r)^(1/2)), но для фиксированного уровня энергии радиус сам зависит от заряда(как 1/Z), в итоге скорость электрона на данном уровне энергии зависит лишь от заряда. В частности, для наинизшего уровня v=Zαc. В квантовой механике скорости нет, но зависимость энергии низшего уровня от заряда тоже немного ломается при достаточно больших Z.
Аноним 09/07/16 Суб 18:51:53 #355 №374626 
>>374338
На связи снова не могущий в сверхсветовую скорость.
Первый штраф за превышение скорости был выписан, когда чел ехал 13 км/ч вместо 3. Так, по предложенной аналогии с волнами на воде, имея крайне ограниченную технологию скоростного радара, возможно не зарегистрировать и один поезд, отъезжающий от другого (>>374294, их-то относительная скорость - 110 км/ч, а наш несовершенный радар меряет только, допустим, до 100 км/ч).
Так вот, тот факт, что мы с одного фронта света не можем зарегистрировать второй фронт, так как наше средство регистрации не дотягивает до относительной скорости разбежки фронтов, по-моему, никак не отменят объективной, не померенной кем-то, скорости в две световых. Явление, так сказать, а не феномен. Так же как и скорость поездов мы можем не измерять, а логически (не эмпирически, а абстрактно) сложить два сведения, полученных звонками от машинистов.
Совсем я того, похоже :S
Аноним 09/07/16 Суб 19:32:07 #356 №374634 
>>374626
Померять скорость можно и другими способами, не обязательно радаром. Вот один из вариантов:
Берём огромное количество часов и ставим их на огромной длинной линейке с интервалом в 1 метр.
Настраиваем их так, что при прохождении мимо часов поезда они отправляют тебе сигнал с текущим временем.
Теперь берёшь эту линейку с часами, и вместе с ней разгоняешься на первом поезде, а в противоположную сторону запускаешь другой поезд.
Синхронизируешь часы
Теперь остаётся только сидеть и ловить сигналы. Так, если вначале пришёл сигнал "10 секунд", а затем пришёл сигнал "12 секунд", значит относительная скорость будет 0.5 м/с.
Только неважно как измерять относительную скорость, она всегда будет ≤c.
Аноним 09/07/16 Суб 19:49:18 #357 №374641 
>>374626
Естественно не отменяет.
Тут все зависит от точки зрения.

Для внешнего неподвижного наблюдателя время течет нормально - он видит, что пучки разлетаются друг от друга на двух световых.
Тут все норм - волны от обоих фронтов, двигаясь на световой, рано или поздно достигают наблюдателя, и он видит движение обоих фронтов.

Для самого пучка происходит тоже самое - он тоже видит, что второй пучок улетает от него на двух световых.
Но есть две проблемы:
1 - т.к. волны фронта движутся на световой, а пучки удаляются друг от друга на сверхсветовой, фронт первого пучка никогда не достигнет второго пучка - пучки не смогут друг друга пронаблюдать, физически.
2 - если учесть замедление времени, наблюдаемая скорость удаления одного пучка от другого превысит световую в дохуллиард раз. Это окончательно ставит точку в вопросе возможности наблюдения - доххулиард процентов на ошибку никак не списать, при всем желании.

На пучки замедление времени действует, а на наблюдателя - нет. В этом и разница.
Аноним 09/07/16 Суб 19:52:19 #358 №374644 
>>374641
>1 - т.к. волны фронта движутся на световой, а пучки удаляются друг от друга на сверхсветовой, фронт первого пучка никогда не достигнет второго пучка - пучки не смогут друг друга пронаблюдать, физически.
А если они сближаются, а не разлетаются?
Аноним 09/07/16 Суб 22:03:45 #359 №374663 
>>374644
Тогда ситуация другая: пучки друг друга не видят до самой встречи, т.к. пучки движутся со скоростью фронта, вместе с фронтом.
После встречи они тоже друг друга не видят - уже по причине сверхсветовой скорости удаления.
Видят друг друга они только в момент встречи, всего миг.
И при этом видят с сильным смещением спектра, из-за замедления времени. Мы например даже самой крутой электроникой не смогли бы засечь пучок, двигаясь ему на встречу на световой.
Аноним 09/07/16 Суб 22:22:53 #360 №374664 
Поясните, почему скорость света вообще есть?
Аноним 09/07/16 Суб 23:38:52 #361 №374669 
>>374664
Потому что свет в чем-то должен распространяться.
Как звук распространяется в воздухе, со скоростью относительно воздуха, независимо от скорости источника, породившего звук.
Так и свет распространяется в пространстве, со скоростью относительно пространства, независимо от скорости источника, породившего свет.
С момента излучения света/звука они дальше живут самостоятельно, и никак не зависят от источника.
Аноним 10/07/16 Вск 14:52:06 #362 №374715 
Зачем нужна теория категорий? Кружочки и стрелочки между ними прекрасно описываются теорией графов.
Аноним 10/07/16 Вск 15:34:22 #363 №374716 
>>374715
Тебя же не удивляет, что мат анализ не считается разделом теории множеств?
Аноним 10/07/16 Вск 15:46:28 #364 №374717 
14681547881850.jpg
>>374669
>Как звук распространяется в воздухе, со скоростью относительно воздуха, независимо от скорости источника, породившего звук.
Аноним 10/07/16 Вск 16:05:58 #365 №374721 
>>374715
>Зачем нкжна физика? Электроны и сонолюминесценсия прекрасно описывается божьей волей.
Аноним 10/07/16 Вск 18:48:09 #366 №374774 
>>372443 (OP)
В чем разница между полиморфными/аллотропными превращениями под действием давления и под действием гравитационным полем(например другая планета с огромной силой приятжения)?
Аноним 10/07/16 Вск 20:07:23 #367 №374795 
>>374774
Переформулируй. Суть не понятна.
sageАноним 10/07/16 Вск 20:12:34 #368 №374798 
Напомните, почему если до нашей земли каждую тысячу лет долетает свет от новых звезд, почему ночь еще не выглядит как огромный пласт светящихся точек, ярче даже солнца?
Аноним 10/07/16 Вск 20:16:51 #369 №374799 
>>374798
Так плотность очень низкая, плюс рассеяние.
Аноним 10/07/16 Вск 20:18:31 #370 №374800 
>>374799
звезд же бесконечно много
Аноним 10/07/16 Вск 20:19:59 #371 №374802 
>>374800
Не бесконечно. И между ними огромные расстояния, а волновые пакеты расплываются.
Аноним 10/07/16 Вск 20:28:43 #372 №374805 
>>374716
>>374721
>ответили вопросом на вопрос
Таки, давайте без евrейской дискуссии, а?
Еще раз, чем отличается граф от категории? Просто, понятно, по-рабоче-крестьянски.
Аноним 10/07/16 Вск 20:31:56 #373 №374806 
>>374795
полиморфные/аллотропные превращения - это изменение строение кристаллической решетки под действием давления и температуры. Банальный пример превращение графита в алмаз под действием высокого давления, высокой температуры и катализатора(который не обязателен)
Я спрашиваю будет ли разница, если вместо повышения давления использовать повышение силы тяжести. Например при повышение давления вещество А перестраивает кристаллическую решетку, превращаясь в вещество Б, а если бросить вещество А на планету, у которой нет атмосферы, но есть огромная сила притяжения, превратится ли вещество А в вещество Б, в таком случае. Если превращается, то эквивалентны ли сила притяжения и давление или есть разница, например некоторые полиморфные модификации можно получить только путем изменения силы притяжения, и как бы ты не менял давление, при постоянной силе тяжести эти полиморфные модификации ты не получишь и наоборот.
Аноним 10/07/16 Вск 20:34:49 #374 №374808 
>>374806
Говоришь всякие умные слова а уровень знаний 7 класса средней школы, лол.
Аноним 10/07/16 Вск 20:45:39 #375 №374814 
>>374806
В теории гравитация может заменить давление. На этом основана концепция существования металлического водорода в ядрах газовых гигантов. Однако при другой гравитации изменится и поверхность энергии Гиббса, поэтому возможно смещение локальных энергетических минимумов. Т е модификации будут не такие же, что и на Земле.
Аноним 10/07/16 Вск 20:53:29 #376 №374818 
>>374814
Спасибо за ответ
>>374808
Ну бля, я большее по химии и материаловедению упарываюсь, чем по физике(Хотя в глубокой теоретической химии, уже только одна физика)
Аноним 10/07/16 Вск 20:59:12 #377 №374820 
>>374814
А можно ли заменить гравитационное притяжение ускорением? Вроде видел как будущих космонавтов сажают в герметическую кабину и начинают вращать по кругу с разными ускорениями создавая перегрузки в разные g.
Аноним 10/07/16 Вск 21:37:01 #378 №374830 
>>374820
Можно.
Аноним 10/07/16 Вск 23:11:07 #379 №374850 
Кто как думает? Допустим просыпаетесь утро и видите в небе корабли инопланетян. С какой целью они могут прилететь сюда? Они мирные или все-таки война?
Аноним 10/07/16 Вск 23:40:33 #380 №374856 
>>374850
В твоём случае они прилетели, не предупредив никаким сигналом, и люди их не смогли засечь, то есть они прилетали скрытно-налицо злой умысел.
Аноним 10/07/16 Вск 23:48:20 #381 №374857 
>>374850
>мирные или все-таки война?
Освобождать будут, не иначе. Лучше уж сразу пустить себе пулю в лоб.
Аноним 11/07/16 Пнд 00:10:12 #382 №374861 
>>374850
Субъект не поймет субъекта. Даже зависнув над городами невозможно точно сказать мир или войну они несут.
Аноним 11/07/16 Пнд 00:53:11 #383 №374873 
>>374850
По любому мирные.
Чтобы смочь в такие технологии, цивилизация должна сохранять стабильность веками, т.е. вообще без войн.
Это неизбежно приводит к столь высокой морали, что представители подобной цивилизации искренне удивятся, если им поведаем, не доверяем им, что нацелили на них тысячу боеголовок и держим палец над кнопкой.
С их точке зрения подобное поведение варварство каменного века, когда все бегали голожопыми с дубинами, били друг друга по голове и воровали самок. Подобное поведение непродуктивно, их же цивилизация намного продуктивнее нас, намного эффективнее, так что подобное нашему мышление у них осталось за бортом многие века назад.

В общем подобные технологии гарантируют высокую степень морали, высокую социолизированность - их члены общества имеют намного более тесные связи, чем мы, можно сказать у них все друг другу братья.

Опасаться нечего. Наоборот - нужно активно перенимать новый уровень цивилизованности, стараться ему соответствовать, постараться влиться.
И с порога предупредить, что мы пока за себя отвечать не умеем - так что пусть игрушки придержат до лучших времен, иначе мы бахнем ими, от недостатка ума. Дураков у нас хватает.
А у них возможно давно не осталось полицейских служб - один дурак, пусти его к ним жить, натворит много бед, пока они хоть что-то смогут предпринять. Это как свинью пустить в хату - и в грязи изваляется, и все полы загадит, и на кровать насрет, заебешься устранять последствия столь грубого и глупого поведения. Тонкую систему легче сломать.

>>374856
Во первых они могут искренне ожидать от нас того, что сами ждали бы от пришельцев - распростертых обьятьев, а никак не ракеты.
Уровень мышления выше.
Во вторых мы и астероид не можем засечь, пока он не ебнет по челябинску, а ты о маленьких корабликах, которые могут выныривать из пространства прямо над атмосферой - при всем желании ни сигнала, ни раннего предупреждения не дождешься.
Есть и третья причина - если они не пользуются прыжками, а юзают досветовые двигатель, и летают на оптимальной временной скорости, т.е. не слишком медленно и не слишком быстро, ни они нам сигнал не смогут послать, ни мы их радарами не сможем засечь - слишком высокое смещение по частоте, у нас и оборудования на такие частоты нет. Откуда им знать, что мы их не видим и не слышим?
На таких скоростях летать - считай почти вслепую, до самого конца полета ты почти полностью изолирован от вселенной.
Аноним 11/07/16 Пнд 02:32:37 #384 №374888 
>>374873
Это не ты там тред про мир духов создал, лол?
Аноним 11/07/16 Пнд 03:47:14 #385 №374895 
>>374888
Нет
Аноним 11/07/16 Пнд 04:38:03 #386 №374897 
>>374895
Твой пост - паста?
Аноним 11/07/16 Пнд 05:51:10 #387 №374899 
>>374873
Когда-то давно одноклеточные создания начали образовывать колонии, которые в итоге развились в многоклеточные организмы.
Сейчас процесс повторяется в более крупном масштабе. Мы, люди, стали кучковаться в сообщества, и у нас есть амбиции создать единую цивилизацию.
Такая аналогия не подтверждает твоих предположений. Например
>Опасаться нечего. Наоборот - нужно активно перенимать новый уровень цивилизованности, стараться ему соответствовать, постараться влиться.
Скажи, ты часто задумываешься о судьбах колоний бактерий, которые тебя окружают? Если одна из них захотела бы "влиться" в тебя, то ты бы обрадовался, стал бы ей покровительствовать и помогать развиваться? Или может пшикнул бы на неё дезинфектантом?
>у них возможно давно не осталось полицейских служб
В нашем организме полицейских служб полно. И если они обнаруживают, что какая-то клетка переродилась, то её оперативно уничтожают. Существует даже целый класс аутоимунных заболеваний, когда имунная система атакует здоровые клетки тела. Охота на ведьм была придумана природой задолго до инквизиции.
>Во первых они могут искренне ожидать от нас того, что сами ждали бы от пришельцев - распростертых обьятьев, а никак не ракеты.
В мире многоклеточных происходят те же процессы что и у одноклеточных - вечно идёт борьба, сильный ест слабого. Непонятно, почему сделав ещё один шаг к глобализации развитая цивилизация должна вдруг стать по-детски наивной.


Я не разделяю твоего оптимизма в отношении глобализации. В хрупком мире высокоразвитых технологий личные интересы индивида неизбежно будут конфликтовать с общественными. Ты предполагаешь, что эту проблему будут преодолевать воспитанием высокой морали. Мне видится иначе.

Если тебе надо чтобы слуга не портил твоих наложниц ты отрубаешь ему яйца, а не ведёшь с ним беседы о нравственности и пользе целомудрия. Не потому что ты садист, просто отрубание яиц работает, а болтовня нет.

Чтобы человек всегда делал выбор в пользу общественных интересов, и не было даже малейшей возможности что сделает эгоистичный выбор, надо отрубить ему эго. Пчела не задумываясь отдаёт свою жизнь чтобы защитить улей. У людей это ещё не отработано, но мы обязательно этому научимся. Не знаю как, возможно методами биоинженерии или фармакологии. Способы сделать это появяться у нас раньше, чем возможность улететь к другим звёздам.

Будущее за тоталитаризмом, он неизбежен. Обычно это слово воспринимают негативно, но что собственно плохого в тотальном контроле?
Аноним 11/07/16 Пнд 07:17:35 #388 №374903 
>>374899
> Такая аналогия не подтверждает твоих предположений.
Аналогия не доказывает ничего.

Между солнцем и футбольным мячом тоже есть аналогия. А в футбол солнцем почему-то играть не получается.
 Аноним 11/07/16 Пнд 08:52:53 #389 №374904 
>>372443 (OP)
Sup, аноны! Все мы знаем, что существуют переносные радиостанции, с помощью которых общаются дальнобойщики, военные и всякие радиолюбители, если еще остались. Подобное занятие было популярно в 80-е годы.
У меня такой вопрос: А существуют ли подобные видеостанции, чтобы транслировать свой видео сигнал?
Аноним 11/07/16 Пнд 10:59:24 #390 №374909 
>>374899
Ты тоже считаешь, что прилетят аналоги людей? Нельзя вот так сразу утверждать, что они прилетели с миром или с войной. Как и аналогия с бактериями неправильная - ты описываешь взаимодействие всей цивилизации (организма) разом, а не её отдельных ячеек.
Пока нам не известна психология пришельцев, предсказывать столь глобальные события невозможно. Может жизнь и развивается по строгим принципам, но психология и мышление у каждого свое.
Аноним 11/07/16 Пнд 11:06:00 #391 №374910 
>>372443 (OP)
То, что Ландау якобы сказал, что атомный взрыв невозможен, это ведь пиздеж? Или нет?
Аноним 11/07/16 Пнд 13:35:02 #392 №374916 
>>374903
>Аналогия не доказывает ничего.
Да, я экстраполирую из имеющихся данных, не имея возможности доказать что это корректный переход.
Но мои предположения по крайней мере опираются на что-то, а вот у >>374873 они парят в атмосфере плотного манямирка.

>>374909
>Ты тоже считаешь, что прилетят аналоги людей? Нельзя вот так сразу утверждать, что они прилетели с миром или с войной.
Да я и не утверждаю этого. Я предполагаю, что они будут рассматривать нас как материал. Микробиолог, нашедший колонию бактерий, может очень обрадоваться, посадить её на питательную среду и смотреть в микроскоп, как она растёт. А может и дезинфектантом...

>Пока нам не известна психология пришельцев, предсказывать столь глобальные события невозможно. Может жизнь и развивается по строгим принципам, но психология и мышление у каждого свое.
Но теория игр на всех одна. Значит понятия "мир", "война", "общественный договор", "альтруизм" и "эгоизм" наверняка будут им знакомы.
Аноним 11/07/16 Пнд 13:37:56 #393 №374917 
>>374904
видео сигнал? лолчто? можно видео передать по радиосигналу.
сагу забыл убрать?
Аноним 11/07/16 Пнд 13:42:52 #394 №374918 
>>374917
Может он про видеофоны спрашивает?
Аноним 11/07/16 Пнд 14:45:54 #395 №374926 
>>372443 (OP)
проиграл с каждого вопроса в этом итт in this thread треде
Аноним 11/07/16 Пнд 16:10:09 #396 №374943 
>>374916
Теория игорь не работает в одну харю. А играть во вселенной, сам понимаешь, не с кем.

Законы развития тащемто тоже одни на всех.
Промоделируй развитие нескольких цивилизаций, которое приведет к технологиям межзвездных путешествий. И увидишь, что вариантов нет - все цивилизации в итоге пошли одним и тем же путем.

Выходит, развитие закономерно. Эволюция любых систем идет по общим правилам, и мы - не исключение.

Подобные технологии гарантируют общественный строй соответствующего уровня. По иному никак не выходит.
Более низкоуровневое общество просто не сможет осилить разработку столь тонких технологий - оно само очень нестабильно, слишком нестабильно, а для технологий подобного уровня требуется высокая стабильность общества и на довольно долгом временном отрезке - будь там войны, для такой цивилизации все накрылось бы медным тазом.

Т.е. для достижения более высокого уровня технологий должно измениться само общество.
Или по иному - мораль ограничивает общество в технологиях.
Можно утверждать, что дикари обладают низким уровнем технологий, а высокоцивилизованные люди - высоким.
Можно утверждать и обратное: обладатели низких технологий - дикари, обладатели высоких - высокоцивилизованные люди.
Истории про захват дикарями высоких технологий - детские сказки. Технологии не существуют в вакууме - это часть инфраструктуры. Тем же движкам нужен целый бекграунд в виде запчастей, обслуживания, управления.
Да взять хотя бы те же истребители - представь что группа папуасов перехватила корабль с сотней истребителей. И что, это автоматически означает, что у папуасов есть современная авиация?
Нет. Они не знают как летать, у них нет подготовленных для них аэродромов, системы регулирования воздушного траффика, связи, сети радаров, спутниковой разведсети, нет топливного производства, нет производства запчастей и оружия, боеприпасов.
Эти истребители у них сгниют. Они из них себе жилища сделают. Но никак не смогут конкурировать с армиями, которые имеют те же истребители на вооружении плюс весь бекграунд к ним.
Для низкоуровневых цивилизаций высокие технологии - это экспонат, а не рабочая модель.
Без той же ТО например невозможно запилить gps - цивилизация не может в спутниковую навигацию, даже получив производство спутников и ракет, но не получив ТО. Она просто не сможет воспользоваться спутниковой технологией, ей придется самостоятельно развить всю ветку технологий до ТО - а это сотни лет.
В общем подобное несбалансированное развитие тоже невозможно - цивилизация не может использовать чужие технологии, не имея своих, которые их дополняют. Нельзя заиметь технологию межзвездных двигателей, не имея полный стэк технологий с ней связанных.
Верно и обратное: если цивилизация имеет все необходимые низкоуровневые технологии, то ей нет смысла красть высокоуровневую технологию, т.к. она сама на пороге ее открытия.
В итоге каждый развивается независимо, и вынужден так или иначе, но добывать все технологии самостоятельно.
Даже если нам кто-то захочет помочь с развитием, подкинув на халяву технологий - из этого ничего не получится, т.к. мы их просто не поймем, не сможем использовать осознанно, и в итоге все-равно окажемся зависимы от доброго самаритянина - он вынужден будет управлять своей технологией по нашим запросам.

Высокий уровень цивилизации автоматически исключает любые войны.
Убивают потому, что не видят иного выхода. А не видят его потому, что глупы, чтобы увидеть.
Цивилизация высокого уровня видит, как можно использовать врага и без его убийства.
От убийства пользы никакой - враг это тоже людской ресурс, и его можно использовать. Надо просто грамотно подобрать последовательность воздействий.
Т.е. решение конфликта - это интеллектуальная задача. Когда мозгов не хватает - хватаются за оружие.
Так что в подобной цивилизации скорее всего полицейских сил и тюрем нет как класса - есть более эффективные процессы регуляции общества, чем насилие и принуждение. И наука дает к ним доступ. А с наукой у межзвездной цивилизации проблем явно нет.
Мы уже на пороге этих изменений - на настоящий момент в принципе уже все поняли, что тюрьмы и полиция бесполезны. Просто пока не знают чем их заменить.

У цивилизации нашего типа например нет шанса в межзвездные технологии на данном этапе. Мы слишком примитивны и нестабильны для столь тонких процессов. Нам нужны новые более тонкие подсистемы в обществе, а их создание в текущих условиях невозможно, из-за постоянных конфликтов и политической нестабильности.
Сначала нужно реорганизовать общество, иначе ничего не взлетает.
А реорганизовав общество, мы фактически становимся другой цивилизацией - более высокоуровневой. И вот она в звездолеты может.
Но до этого еще долгий путь - сперва нужно устранить причину конфликтов, ближайший век-полтора будем заниматься именно этим, ни о каких звездолетах пока можно не мечтать.
Потом нам предстоит еще век-два непосредственно эволюции, уже в новом обществе.
И только потом станет возможным получить доступ к столь тонким научным процессам, чтобы заиметь технологию межзвездных путешествий.
Аноним 11/07/16 Пнд 16:47:05 #397 №374946 
Пздц. О чем вы, аноны? Как можно судить о морали и технологиях даже не просто другого биологического вида, а существ, которых возможно даже живыми в земном понимании назвать нельзя? Откуда представление, что у них вообще есть мораль, или что у них не считается высокоморальным очистить вселенную от живой материи? Даже среди людей, у каждого своя мораль и свои мировоззренческие особенности.

>>374943
>будь там войны, для такой цивилизации все накрылось бы медным тазом
Технологии человеческой цивилизации, кстати, совершают качественные скачки именно во время войн. В то же время, в теократических обществах (что в исламских странах, что в средневековой христианской Европе), где за моралью не просто следят, а беспощадно наказывают любых отступников, уровень технологического развития сильно упал по сравнению даже с античностью. Так что не надо тут про высокодуховные общевселенские ценности.
Аноним 11/07/16 Пнд 17:56:11 #398 №374954 
14682489718800.jpg
Спрошу здесь. Подскажите, пожалуйста, годных книг о морской живности, от беспозвоночных до млекопитающих, и экологии морских глубин. Чем больше, тем лучше.
Аноним 11/07/16 Пнд 18:37:01 #399 №374956 
>>374943
>Теория игорь не работает в одну харю.
У этой одной хари обязательно есть история.
Если у неё вдруг случилась амнезия и история забылась, то вокруг куча примитивных живых организмов, которые живой ум непременно станет изучать.
Если других организмов вокруг нет, то можно изучать собственное строение.
Если и это по каким-то причинам невозможно, то теория игр будет в кратчайшие сроки разработана после того, как произойдёт столкновение с другим живым существом. Мы всё-таки о высокоинтеллектуальном создании говорим.

>Можно утверждать, что дикари обладают низким уровнем технологий, а высокоцивилизованные люди - высоким.
Ок.

>Убивают потому, что не видят иного выхода. А не видят его потому, что глупы, чтобы увидеть.
Или потому что на всех объективно не хватает ресурсов.

>Т.е. решение конфликта - это интеллектуальная задача. Когда мозгов не хватает - хватаются за оружие.
Отчасти соглашусь. Если бы все были беспристрастными интеллектуалами, то конфликты бы разрешались без оружия.
Например научное исследование однозначно показывает, что одна раса имеет более низкий потенциал по сравнению с другой. Ретарды читают статью и говорят:
- Ок, мы всё понимаем, мы перестанем размножаться. Заселяйте, пожалуйста, наши города, забирайте наше имущество. Мы покажем где что лежит, и будем делать для вас грязную работу. А если надо ускорить процесс заселения, то мы будем выпиливаться.
Полное торжество разума над животными инстинктами.

>Цивилизация высокого уровня видит, как можно использовать врага и без его убийства.
Она также видит какую выгоду можно извлечь из уничтожения врага.
Представь, что на участке, где ты хочешь построить дом обнаружился муравейник. Ты знаешь, что муравьёв можно использовать без убийства - если засунуть в муравейник палочку, то они будут атаковать её, и она станет кисленькой.
Но едва ли это достаточная причина чтобы отменить строительство дома.

>Мы уже на пороге этих изменений - на настоящий момент в принципе уже все поняли, что тюрьмы и полиция бесполезны. Просто пока не знают чем их заменить.
Лоботомией.

>Нам нужны новые более тонкие подсистемы в обществе, а их создание в текущих условиях невозможно, из-за постоянных конфликтов и политической нестабильности.
Не только поэтому. Мы тратим кучу ресурсов обслуживая свой гедонизм.
От многих пережитков нам предстоит избавиться. В идеале сознательные, дисциплинированные граждане должны тратить ресурсы только на продуктивную деятельность.
Личный транспорт - бессмыслица, надо пользоваться общественным. Казармы с двухъярусными кроватями намного эффективнее личного жилья, а столовые эффективнее индивидуальных кухонь.
Вообще частная собственность стоит на пути прогресса. Система должна давать человеку всё нужное, а он должен отдавать ей всё что производит.
Внешний вид одежды не важен, продуктивнее шить одинаковые модели, отличающиеся только размером и назначением.
Вместо того, чтобы тратить время на сложные причёски проще и быстрее бриться налысо.
Кстати клеить обои на стены совершенно бесполезно.

Современный человек недостаточно сознателен, чтобы от всего этого отказаться. Но человек будущего будет сконструирован так, что для него такие условия будут полностью удовлетворительными. Он сможет трудиться на благо общества с высокой эффективностью, и будет счастлив. А мысли о каких-то личных интересах у него просто рождаться не будут.
Аноним 11/07/16 Пнд 18:48:41 #400 №374957 
>>374910
Ну раз атомные взрывы уже были, и не раз-вывод очевиден. А вообще источник этой фразы неплохо бы предоставить
Аноним 11/07/16 Пнд 19:00:18 #401 №374958 
>>374956
Эту хуйню веками втирают, а человек всё не меняется и не меняется.
Аноним 11/07/16 Пнд 23:52:59 #402 №374993 
Вот допустим на планете похожей на землю, но не тронутой человеком, оказывается группа людей. Как они будут действовать?
Аноним 12/07/16 Втр 00:53:38 #403 №374999 
>>374993
https://www.youtube.com/watch?v=dnX-3NebyOw
В /fs/, быдло.
Аноним 12/07/16 Втр 10:27:41 #404 №375022 
>>374999
Так меня интересует самый оптимальный план действий на такой случай.
Аноним 12/07/16 Втр 10:47:55 #405 №375023 
14683096758980.jpg
Рождаются ли новые частицы во вселенной, или их количество фиксированно? Синтезировать можно что угодно, но из каких то основных материалов. Количество этого материала - фиксированное? Убывает ли оно, или оно, количество, может расти?
пик рандом
Аноним 12/07/16 Втр 11:08:19 #406 №375025 
>>375023
хуй его знает, квантовая физика пока еще работает над этим, всякие субатомные частицы новые открывает
Аноним 12/07/16 Втр 11:12:33 #407 №375026 
>>375023
Если под "новые" ты имеешь в виду, что их количество увеличилось-да, без проблем, такое даже ускоритель может устроить.
Аноним 12/07/16 Втр 11:18:16 #408 №375027 
>>375026
Раскрой подробнее свою фразу. Ускоритель может сделать из 10 протонов сто?
Аноним 12/07/16 Втр 11:27:24 #409 №375028 
>>375027
Формально может.
Берём 2 протона, разгоняем их и сталкиваем. При достаточной энергии может произойти реакция
p+p->p+p+p+(-p)
, в которой рождается пара протон-антипротон. Повторить 100 раз, не допуская аннигиляции-и вот у нас 100 лишних протонов и столько же антипротонов
Аноним 12/07/16 Втр 11:30:17 #410 №375029 
>>375028
>p+(-p)
Мне вспоминаются виртуальные частицы. Твой пример с ними связан?

Если да -- флуктуации в вакумме это единственный источник частиц, или можно ещё найти примеры, когда из ничего и энергии появляются частицы?
Аноним 12/07/16 Втр 11:30:21 #411 №375030 
>>375028
>>375027
Но зачем, если можно создавать ёбафотоны?
Аноним 12/07/16 Втр 11:33:01 #412 №375031 
>>375029
Нет, получившиеся частицы вполне реальны и существуют дольше, чем h/E. И берутся они из энергии столкновения
Аноним 12/07/16 Втр 11:42:02 #413 №375032 
>>375031
На самом деле почти нет. Смотря что понимать под реальностью.
Аноним 12/07/16 Втр 11:46:09 #414 №375033 
>>375032
Виртуальные частицы не могут существовать дольше, чем h/E. Если частица существует дольше этого времени-она не виртуальная. Никаких "почти" тут нет
Аноним 12/07/16 Втр 12:08:34 #415 №375035 
>>375033
Смотри, так называемые "виртуальные частицы" - это в большинстве случаев, если говорить более понятно, визуализация той же частицы из прошлого и будущего.
Аноним 12/07/16 Втр 12:24:47 #416 №375036 
>>375035
Нет, это не так. Хуйню несёшь. Как минимум потому что протон может испустить виртуальный пи-мезон, ты же не скажешь, что этот протон был/будет пи-мезоном? Даже больше, виртуальные частицы могут рождаться и в вакууме, например под действием электромагнитного поля
Аноним 12/07/16 Втр 13:26:36 #417 №375047 
>>375036
Нет, это так. Хуйню не несу. Восприятие наблюдателя в тебе засело.
Аноним 12/07/16 Втр 13:44:55 #418 №375051 
>>375047
А какое ещё у меня должно быть восприятие?
И потом, я тебе предоставил 2 факта: первый-частица может излучать виртуальные частицы другого вида, и второй-виртуальные частицы могут образовываться в вакууме. Либо твоя теория про "виртуальная частица-это реальная частица с прошлого" обьясняет эти факты(в таком случае с тебя обьяснение, или хотя бы ссылка на него), либо не обьясняет(а значит она не верна, и можешь её уносить туда, откуда принёс).
Аноним 12/07/16 Втр 16:12:51 #419 №375060 
>>372443 (OP)
Хочу книгу по двигателям внешнего сгорания, чтобы можно было построить себе такой без регистрации и смс.
А что кто может сказать про цикл эдвардса? Вики грит что он круче карно, но это не подтвержденно другими ученым подкопчеными.
Аноним 12/07/16 Втр 16:44:59 #420 №375071 
>>375051
Абстрагированное.
Сначала докажи свои "факты", или так и будешь наблюдателем.
Аноним 12/07/16 Втр 17:15:37 #421 №375083 
>>375060
Ну я открыл вики, нашёл первую формулу кпд, пересчитал-так и есть. Сравнил с кпд Карно, оказалось, что Карно лучше для любых температур(при T1=T2 они оба дают 0), как и ожидалось
Аноним 12/07/16 Втр 17:17:53 #422 №375085 
>>375071
Обменное взаимодействие нуклонов(если бы нуклоны обменивались реальными частицами-они не притягивались бы)
Поляризация вакуума(эффект Казимира)
Вот тебе экспериментальные факты.
Аноним 12/07/16 Втр 17:32:45 #423 №375087 
>>375022
Самый оптимальный - двухходовочка.
Сперва строим временный лагерь, и кушаем что принесли, а в это время ведем максимально широкую разведку на местности.
Опосля выбираются варианты размещения базового лагеря, с учетом близости природных источников воды, пищи, топлива, и собственно начинается обживание планеты.
Других вариантов и нет. Информация - залог успеха.
Аноним 12/07/16 Втр 17:36:12 #424 №375088 
>>375023
Количество энергии во вселенной непрерывно растет.
Материя - одна из форм энергии.

Так что не вижу проблем, почему в каких-то процессах лишняя энергия не может превращаться в материю.
Аноним 12/07/16 Втр 17:46:00 #425 №375089 
>>375036
Тащемто и без электромагнитного поля вакуум постоянно штормит
https://ru.wikipedia.org/wiki/%D0%AD%D1%84%D1%84%D0%B5%D0%BA%D1%82_%D0%9A%D0%B0%D0%B7%D0%B8%D0%BC%D0%B8%D1%80%D0%B0

Колеблющиеся пластины вообще выбивают из вакуума целый поток материи. Т.е. при приложении внешней энергии виртуальные частицы превращаются во вполне себе реальные.
Аноним 12/07/16 Втр 17:49:19 #426 №375090 
>>375089
Да, то я спутал с рождением реальных пар в сильном поле.
Аноним 12/07/16 Втр 17:51:41 #427 №375093 
>>375085
Ты не понимаешь.
Аноним 12/07/16 Втр 18:00:01 #428 №375095 
>>375093
>вот моя теория
>нет, это не так, потому что факт1 и факт2
>а я говорю, что это так
>нет, вот же факты
>где доказательства?
>вот, всё экспериментально проверено
>ты не понимаешь
Думаю на этом можно и закончить
Аноним 12/07/16 Втр 18:13:52 #429 №375096 
Не знаю, почему мод удалил тред, попробую спросить тут.
Есть ли симуляторы молекулярных и атомных взаимодействий, достаточно точные и мощные для того, чтобы симулировать на молекулярном уровне жизнь одной бактерии?

Т.е. чтобы все процессы типа поглощения питательных веществ, транскрипции генов и т.д. симулировались на уровне физики отдельных атомов и соединений?
Аноним 12/07/16 Втр 18:17:50 #430 №375098 
>>375096
Нет. Не в последнюю очередь потому, что топовые суперкомпьютеры единственный белок-то со скрипом моделируют.
Аноним 12/07/16 Втр 18:44:50 #431 №375103 
>>375098
Сколько там в этих белках может быть атомов, что их поведение симулируется "со скирпом"?
Аноним 12/07/16 Втр 18:48:39 #432 №375104 
>>375103
>>375098
И ещё. Объясните мне, дилетанту, на что там тратятся основные ресурсы?
Если на обычных видюхах однотипные вычисления уже идут с охуительными скоростями
Если бы один анон не сделал эту тему своеобразным пугалом для более-менее цивилизованного анона, на эту тему дискуссии были бы куда интереснее
Аноним 12/07/16 Втр 19:20:29 #433 №375110 
>>375104
>пугалом для более-менее цивилизованного анона
Любопытное у тебя определение цивилизованности. На стадность очень смахивает.
>симулировались на уровне физики отдельных атомов и соединений?
Смешно.
Смотри: мы с помощью атомных взаимодействий симулируем атомные взаимодействия. Симулируем на чуждой реальности архитектуре, чуждыми дубовыми бинарными командами, какую производительность ты хочешь из этого получить?
Аноним 12/07/16 Втр 19:25:28 #434 №375112 
>>375088
>Количество энергии во вселенной непрерывно растет.
Это такая местная шутка? Помоему, наоборот, убывает.
Если не шутка - то откуда энергия новая берётся?
Аноним 12/07/16 Втр 19:49:54 #435 №375116 
>>372443 (OP)
Хочу сделать себе из пластиковых бутылок бочку для воды.
Что для этого надо, кроме формы, печки и бутылок? Инертную атмосферу? Давление? Пластик бутылок вообще может в переработку или только фтопку его?
План переплавки таков, создаю из стальных листов форму бочки, запихиваю в форму кучу мелкого измельченного бутылочного пластика, закрываю форму, засовываю форму в костер и жду пока мне будет бочка. Все правильно или нет? С костром сильно сосну?
Аноним 12/07/16 Втр 19:51:45 #436 №375118 
>>375112
Синтез гелия из четырех протонов
Аноним 12/07/16 Втр 19:52:55 #437 №375120 
>>375112
Количество свободно энергии убывает. Именно свободной.

Мимо-анон
Аноним 12/07/16 Втр 21:00:46 #438 №375125 
>>375110
>Симулируем на чуждой реальности архитектуре, чуждыми дубовыми бинарными командами, какую производительность ты хочешь из этого получить?
Поведение атомов вполне себе описывается обычными уравнениями, и даже точность не проблема - достаточно обеспечить её больше Планковского порога.
А числа из этих уравнений ничем не отличаются от чисел в графики в GTA 5.
Аноним 12/07/16 Втр 21:03:42 #439 №375126 
>>375125
>Поведение атомов вполне себе описывается обычными уравнениями
Тупые ебланы строят адронные коллайдеры, когда можно просто взять калькулятор и посчитать.
Аноним 12/07/16 Втр 21:10:52 #440 №375128 
>>375126
>>Поведение атомов вполне себе описывается обычными уравнениями
Да, ведь в бактериях каждый день происходят высокоэнергетические ядерные реакции, поэтому без коллайдера тут никуда.

Специально указал - на уровне физической химии. Целые атомы, электронные облака и тому подобное. Без кварков и внутриядерных сил.
Аноним 12/07/16 Втр 21:15:48 #441 №375129 
>>375112
Нет, именно растет.

Есть такая штука как закон сохранения энергии.
Он гласит, что в замкнутой системе количество энергии постоянно.

Но для вселенной он не работает.
Спутники недавно подтвердили, что вселенная расширяется. Это факт.
Если бы она расширялась равномерно - все было бы ок, закон сохранения импульса такое позволяет.
Но она расширяется с ускорением - что-то постоянно затрачивает силу на ускорение материи и пространства, т.е. постоянно вкачивает во вселенную энергию.
Таким образом количество энергии во вселенной постоянно увеличивается.

Такой НЕХ не происходит в закрытых системах - значит вселенная система открытая, и контактирует с чем-то извне, обмениваясь энергией, а возможно и материей.
Аноним 12/07/16 Втр 21:28:13 #442 №375130 
>>375116
С таким подходом соснешь 146%. Дешевле купить.
Пластик нельзя перегревать - он разлагается с потерей свойств. Источник стабильной температуры стоит много денег - на один раз смысла брать нет.
Пластик нельзя плавить - он пузырится, будет дырявая бочка. На производстве его дегазируют и выплавляют болванки, из которых потом давлением вытягивают бутылки любых форм и размеров в ближайшем подвале.

Вывод: дешевле купить готовую бочку.
На вторичном рынке есть бочки из-под химикатов с заводов - большие, прочные, дешевые. Промыть - и можно юзать.
Аноним 12/07/16 Втр 21:30:07 #443 №375131 
>>375125
Принцип Гейзенберга делит на ноль всю точность.
Он работает и на атомном масштабе.
Аноним 12/07/16 Втр 21:48:45 #444 №375132 
>>375131
>Принцип Гейзенберга делит на ноль всю точность.
>Он работает и на атомном масштабе.
Т.е. ты считаешь, что на атомном масштабе, заюзав функцию Rnd(), нельзя симулировать примерное поведение атомов?
Аноним 12/07/16 Втр 22:12:33 #445 №375135 
>>375125
> точность не проблема - достаточно обеспечить её больше Планковского порога.
Ты постоянную Планка видел вообще? Там впереди нули идут, а другие числа где-то за горизонтом начинаются.
Аноним 12/07/16 Втр 22:22:57 #446 №375137 
>>372443 (OP)
Почему за открытие способа восстановить изначальную структуру денатурированного белка дают шнобелевскую премию? Это же годно в отличие от остального, не?
Аноним 12/07/16 Втр 22:27:54 #447 №375138 
>>375137
Ну смешно же звучит. "Мы научились делать из варёного сырое"
Аноним 12/07/16 Втр 22:32:51 #448 №375139 
>>375130
А хотя бы труб наделать из пластика?
Аноним 12/07/16 Втр 22:37:13 #449 №375140 
>>375132
>заюзав функцию Rnd()
Унесите отсюда этого дебила, пока я ему не уебал.
Аноним 12/07/16 Втр 22:40:06 #450 №375141 
Почему говорят, что нового химического элемента во всей огромной вселенной не найти? Мы же даже за пределы нашей солнечной системы не вышли, почему ученые так уверены в том, что на протяжении миллиардов световых лет от нас разумная форма жизни будет всегда возникать лишь из углерода? Разве это не противоречит логике?
Аноним 12/07/16 Втр 22:45:14 #451 №375142 
>>375132
А зачем вообще считать? Давайте просто заюзаем функцию Rnd(). Профит!
Аноним 12/07/16 Втр 22:47:59 #452 №375143 
>>375141
При чём тут жизнь? Сверхтяжёлые химические элементы долго не живут, даже если они и образуются в небольших количествах.при взрыве сверхновой.
Аноним 12/07/16 Втр 22:50:55 #453 №375144 
>>375139
Без стабильной температуры ты получишь либо дырявые либо хрупкие трубы.
Сможешь в стабильную температуру - можно думать над способом разливки. Но там тоже есть свои сложности.
Аноним 12/07/16 Втр 23:39:12 #454 №375147 
тупой вопрос - нужна ли запятая в "вопросы тупые, и еще тупее" и почему
Аноним 13/07/16 Срд 00:17:45 #455 №375151 
>>375030
>Но зачем, если можно создавать ёбафотоны?
фотоны не имеют массы.
Аноним 13/07/16 Срд 00:21:23 #456 №375152 
>>375088
>>375112
Закон сохранения энергии и массы
Аноним 13/07/16 Срд 00:43:55 #457 №375153 
Как вывели первообразные для элементарных функций, восстанавливая по известным правилам взятия производных в обратную сторону или ещё каким способом?
Аноним 13/07/16 Срд 01:28:36 #458 №375156 
>>375153
>восстанавливая по известным правилам взятия производных в обратную сторону
Да.
Аноним 13/07/16 Срд 05:38:10 #459 №375167 
>>375153
Я уже точно не помню, но sqrt(sinx - x) кажется как-то через полярные координаты ищется.
Аноним 13/07/16 Срд 12:43:36 #460 №375184 
>>375144
Схема кипятильника обыкновенного, например? Тонкий проводок затолкать в инертную среду и сунуть два конца в розетку и обмотать им трубу с формой трубы.
Аноним 13/07/16 Срд 12:49:15 #461 №375185 
>>375184
Кипятильник стабилизирует вода.
Сам по себе он перегревается и сгорает.
Аноним 13/07/16 Срд 13:29:40 #462 №375187 
>>375185
Медленно нагревать.
Как на заводах тогда делают трубы пластиковые?
Аноним 13/07/16 Срд 14:11:09 #463 №375191 
Посоны, я в школе клал болт на физику, а теперь жалею и хочу наверстать. С чего начать?
Аноним 13/07/16 Срд 14:21:12 #464 №375194 
14684088728230.jpg
>>374899
Почитай немного про интегральную теорию развития общества и общественных ценностей.
Аноним 13/07/16 Срд 14:26:50 #465 №375195 
>>375191
Начни со школьного курса. Ну и было бы неплохо знать математику на уровне решения задач
Аноним 13/07/16 Срд 14:32:06 #466 №375198 
>>375167
Это должно быть сложная функция уже.
>>375156
В Википедии ещё про какой-то предел n-ной производной написано было. Это альтернативное определение?
Аноним 13/07/16 Срд 14:44:07 #467 №375201 
>>375194
В воздухе запахло жидкими фантазиями идеалистов?
Аноним 13/07/16 Срд 14:47:08 #468 №375203 
>>375201
Прочитай про спиральную динамику. сначала, перед тем как вынести об этом свое мнение, это же верный подход, да?
Аноним 13/07/16 Срд 14:52:02 #469 №375205 
>>375203
Зачем? Это все - необоснованные теории психологов и социологов с философами. Никаких аргументов в свое подтверждение кроме "Я ЩИТАЮ" они предоставить не в состоянии. Блядь, это все-равно что с пеной у рта доказывать истинность теории струн типа ii a. Эта доска про науку, не квазинаучную демагогию.
Аноним 13/07/16 Срд 14:57:08 #470 №375206 
14684110286550.jpg
>>375205
>Никаких аргументов в свое подтверждение кроме "Я ЩИТАЮ" они предоставить не в состоянии.
Грейвз описал спиральную динамику еще в 80х годах, и дальнейшее развитие цивилизации только подтвердило указанные в теории предположения. как я и сказал, хотя бы ознакомся перед составлением мнения об этом А вообще исключительно-излишне рационально-материалистический подход склонен к редуцированию взглядов к изучению реальности вплодь до того, что феноменология или даже психология сводится чуть ли не к магии.
Аноним 13/07/16 Срд 15:02:57 #471 №375210 
>>375206
Рациональный подход - единственно допустимый в научной сфере. Любые прогнозы, не основанные на естественно-научном знании, в расчет приниматься не должны. Астрологи тоже говорят, что развитие цивилизации только подтверждает их концепции о влиянии Сатурна на историю людей.
Аноним 13/07/16 Срд 15:10:55 #472 №375220 
>>375210
Исследования астрологии создателем интегрального подходакеном уилбером, выдали абсолютный провал в 300 из 300 (изначально в эксперименте учавствовало около 500 человек, из которых были отобранны посредством опроса максимально приверженные астрологическому подходу люди, для точности эксперимента) случаях составления статей старающихся хоть как-то применить астрологический метод в реальной жизни и разумеется не были включены в интегральную теорию. Так что опять же несешь хуйню с потолка. А вообще доводы зеркально похожи на доводы верящих в мифы как в описание реальной жизни.
>Любые прогнозы, не основанные на естественно-научном знании, в расчет приниматься не должны.
Ты хотел сказать на "империческом измерении материи", но решил не создавать себе проблем? Измерение активности мозга во время сна человека не покажут на приборе, что бодрствование более реально чем сон, разве что просто составят различные описания каждых из состояний. И подобного рода вопросы можно изучить только из перспективы первого лица. системность и структурированность психологии не является изучением от первого лица, это чтобы ты вдруг не начал по новой
Аноним 13/07/16 Срд 15:26:58 #473 №375225 
>>375206
>Грейвз описал спиральную динамику еще в 80х годах, и дальнейшее развитие цивилизации только подтвердило указанные в теории предположения.
Однажды выигранный джекпот не делает человека экспертом в покупке лотерейных билетов.
Аноним 13/07/16 Срд 15:32:54 #474 №375228 
>>375225
Ну хорошо, тогда давай так
>Это все - необоснованные
Обоснованы, за самим текстом проследуйте к специальной литературе.
>Никаких аргументов в свое подтверждение кроме "Я ЩИТАЮ" они предоставить не в состоянии.
В состоянии, за самим текстом проследуйте к специальной литературе.
>Любые прогнозы, не основанные на естественно-научном знании, в расчет приниматься не должны.
Тогда либо измени свое представление об естественно-научном, либо опиши свое представление о мире и изучаемом материале другими словами, поскольку прогнозы и описания теорий как раз таки являются естественно-научными, основаны на наблюдениях и опыте.
Специальная литература:
https://www.amazon.com/Sex-Ecology-Spirituality-Spirit-Evolution/dp/1570627444
www.ozon.ru/context/detail/id/5519486/
И еще много остального по тем же авторам, но тебе в любом случае похуй, оставляю здесь просто так
Пересказывать 1500 страниц текста человеку, котороый на любое утверждение отвечает - "совпадение, ниправда, ну и что" не собираюсь. да и в любом случае не стал бы
Аноним 13/07/16 Срд 15:41:16 #475 №375231 
>>375220
Еще раз для тупых. Все, что не основано на фундаментальных законах природы и их экспериментальном проверке к науке не относится. Поэтому социология, психология, история, философия, математика и прочие подобные области знаний - не наука в ее классическом понимании, а следовательно любые гипотезы, выдвинутые в их рамках априори не фальсифицируемы, и не могут служить объективным основанием для описания окружающего мира и построения прогнозов. Считай = астрология, по своей аргументационной базе.
Аноним 13/07/16 Срд 15:47:27 #476 №375233 
>>375228
>как раз таки являются естественно-научными
Лол, ни один эксперимент с участием абстрактно мыслящих существ в качестве объектов не объективен по своей природе. Естественно-научными могут считаться только физические и химические эксперименты, как единственно основанные на неизменных объективных законах вселенной.
Аноним 13/07/16 Срд 15:47:42 #477 №375234 
>>375231
Пиздец, сначала ты свалил в одну кучу психология, история, математика, а теперь еще оказывается что психология и социология не фальсифицируема.
>основано на фундаментальных законах природы и их экспериментальном проверке
Ну так ровно это и описано в этих теориях.
А вообще какой смысл критически обсуждать то, о чем ты вообще не имеешь никакого представления, если для тебя это по прежнему аргументации на уровне отсылок к библии и астрологии. Это ты еще по заглавию не пробежался, как только дошел бы до духовности наверное позвоночник из меня вырвал бы. Но предрекая такой исход событий скажу, что для начала тебе стоило бы разобраться в вопросах терминологии. А то мы в этом обсуждении только на ней уже и погорели.
Аноним 13/07/16 Срд 15:50:09 #478 №375235 
>>375233
>на неизменных объективных законах вселенной
А значит должны быть немедленно отброшены как несущест... как не представляющие внимания, или точнее, как не нравящиеся мне, или если быть до конца точным, как неприятная часть реальности для меня. Ведь частью реальности следует считать только то, что хочу я, а все остальное должно быть немедленно отброшено. Отличная позиция, такой придерживался один веселый, интересный парень с усиками под носом.
Аноним 13/07/16 Срд 15:53:59 #479 №375238 
>>375234
Хватит манявров. В науке правило простое: есть физический закон - есть научное знание, нет физического закона - есть гуманитарно-философская херота. Ни социологи, ни психология, ни история на физических законах не основаны => они не естественные науки.

>>375235
Иди в раздел религии свою хуйню проповедовать. В науке уже давно устоялись критерии, в соответствии с которыми все, что невозможно достоверно установить экспериментально, сведя к фундаментальным законам физики, к науке не относится и доверия не вызывает.
Аноним 13/07/16 Срд 16:00:21 #480 №375243 
>>375238
>установить экспериментально
Это все устанавливается экспериментально, феноменологически или на исследованиях, если критерием истинности считать опыт - то на опыте все изучения и основанны, в зависимости от результатов экспериментов и принимаются дальнейшие решения или поправки в теориях и моделях. У тебя какой-то уже граничащий с дегенеративным тип мышления. Я бы написал еще, но учитывая твой подход, буду только разрушать твою нервную систему. Но на последок пару дров в твою топку все же оставлю:
Шредингер Эрвин - Материя и разум - описание переживания Сатори, да еще и кем... ммм... да да я подтраливаю уже, потому что знаю как ты сейчас мыслишь обо всем этом, путая до/над рациональные воззрения. Ухожу ухожу, надеюсь успею убежать от взрывной волны.
Аноним 13/07/16 Срд 16:01:26 #481 №375244 
>>375140
>>375142
То есть ты утверждаешь, что построение симулятора химической активности невозможно?
Аноним 13/07/16 Срд 16:03:47 #482 №375247 
>>375195
Посоветуйте учебники годные, ибо без понятия куда смотреть
Аноним 13/07/16 Срд 16:09:40 #483 №375249 
>>375243
Ты идиот? Что непонятного в правиле: научный эксперимент - научное знание? Все остальное - влажные домыслы. Кончай свои манявры.
Аноним 13/07/16 Срд 16:18:37 #484 №375252 
>>375249
Грубые примеры
Область психологии: У 25% детей, чьи родители в раннем детстве покончили с собой выражена склонность к депрессии - влажные домыслы.
У 94% людей занимающихся медитацией на протяжении 3-4х лет выявлены продвижения по уровням развития на 1-2 уровня по этой схеме >>375194 все изменения начинались только с нижних уровней, и заканчивались только уровнями выше - влажные домыслы.
Присутсвие осознанности в состояниях бодрствования, сна и глубокого сна у людей занимающихся медитацией больше 10 лет - влажные домыслы.
Ну да, я так вообще что угодно могу отвергать. Твоего сознания тоже не существует, если его нельзы вылит в ведро.
Вопрос изначально был о более прогрессивном отношении к представителям более равитых цивилизаций, которое вряд ли соответсвует действительности, и причины этого описани в спиральной динамике и интегральной теории. Но читать я не буду, лучше буду нести хуйню основываясь на некой "науке", и что в конкретно твоем понимании представляет это твое определение "науки" мне не совсем понятно, что-то на уровне аллаха.
Аноним 13/07/16 Срд 16:29:33 #485 №375253 
>>375252
Лол, ты точно ущербный. Я тебе в четвертый раз должен определение научного знания говорить? Иди-ка ты и правда в религач, чакры (и перышки) прочищать.
Аноним 13/07/16 Срд 16:29:37 #486 №375254 
>>375228
>Ты неправ, даже объяснять лень. Купи книжку на 1500 страниц, там всё написано.
Уважаемый Кен Уилбер, я не буду покупать вашу книжку. Я повидал достаточно нью-эйджевый пиздоболов, чтобы безошибочно определять их по внешним признакам.
Называемся последователем восточной философской школы, чтобы заинтриговать западных хомячков.
Дальше пишем эклектичные тексты, полные слов "мистический", "эзотерический", "божественный", "духовный", "метафизический", можно придумать что-то своё, например "постметафизика". Главное использовать модные слова и избегать немодных.
Смело отождествляем понятия - "Я вам вот что скажу: Шамбала, Китеж-град, Атлантида и Эльдорадо - одно и то же! Точно так же, как дух, пассионарность, эйдос, монада, эгрегор, атман и мын. Вот видите, как чётко пазл складывается?".
Приправляем это всё инфографикой, чтобы создать у читателя иллюзию того, что он чему-то научился, например детектировать у собеседника уровень "божественности". Не важно, что конкретных практических предсказаний он не сможет сделать. Предсказания - удел Учителя, а ты - ничтожный червь.
Всё это было уже у Блаватской.
Аноним 13/07/16 Срд 16:36:37 #487 №375257 
>375254
>Не важно, что конкретных практических предсказаний он не сможет сделать.
Вот так ты свел важность изучения всех секторов реальности для дальнейшего развития к предсказаниям, ну молодец.
>Я повидал достаточно нью-эйджевый пиздоболов
Аргумент не в твою пользу, кстати говоря.
>>375253
>Я тебе в четвертый раз должен определение научного знания говорить
Ну тогда и не надо нести какую-то хуйню о том, что теории развития человечесских ценностей неверна, что психологии несуществует, потому что к ней нельзя применить критерии истинности материалистического мира. Это ты начал нести это дерьмо, а не я. И в ответ на указание несопоставимости имперического материального измерения, ты начал доказывать несостоятельность психологии, потому что она неизмерима линейкой. Ну что за хуйню ты несешь в таком случае?
Аноним 13/07/16 Срд 16:40:55 #488 №375259 
>>375254
Ну и добавлю, что это не Нью Эйдж.
и
>Не важно, что конкретных практических предсказаний он не сможет сделать.
Это только твое мнение ничем не подтвержденное))))
Аноним 13/07/16 Срд 16:43:33 #489 №375260 
>>375257
Ты напоминаешь евангелиста, который пришёл на конференцию по эволюции, чтобы сообщить им что они неправы, а когда его спросили в каком месте они неправы и где аргументы, многозначительно закатил глаза и сказал, чтоб они прочитали Священное Писание.

Без конкретной тезисной критики можешь идти наслажаться каникулами.
Аноним 13/07/16 Срд 16:46:48 #490 №375263 
>>375257
Лол, психологии не существует? Когда это я такое говорил? Ты шоколадным глазом читаешь, видать. Какие такие несопоставимости ты нашел никому кроме тебя не ведомо, а твое написание слова "эмпирический" просто за гранью добра и зла. Наука основана только на фундаментальных закономерностях. Это общественный консенсус, принятый с подачи Ньютона и Поппера. Все твои так называемые теории и исследования - не более чем статистические спекуляции. Кончай уже свои манявры.
Аноним 13/07/16 Срд 16:51:06 #491 №375265 
>>375260
Пушка, это же ты говоришь, что я ну вообще даже не я в конкретном случае, а спиральная динамика к примеру не прав. И я что, должен спорить с твоим отсутсвием обвинения даже? Как можно оспорить что-то, что не имеет в себе никакой идеи? Проще говоря, я должен доказывать, что ты говоришь неправду, при том что эти твои утверждения даже утверждениями не являются? Шизофрения. И пример-то какой привел, ах красавец, только ситуация у нас полностью противоположная.

В сокращенном варианте и по пунктам:
- Ты сказал, что не права спиральная динамика, а прочие вещи в числе которых психология вообще не существуют.
- Просишь меня тезисно критиковать твое утверждение о твоих же словах. Т.е. выходит что критику тут должен приводить ты, а не я. (если уж на то пошло, книги я тебе уже привел, описания теорий дал, то что их нельзя потрогать, облизать или запихнуть себе в задницу в отличии от микроскопа - это конечно весомый аргумент, против такого я поспорить не смогу)
Аноним 13/07/16 Срд 16:52:18 #492 №375266 
>>375253
может это радиопидор?
Аноним 13/07/16 Срд 16:52:23 #493 №375267 
>>375263
>психологии не существует? Когда это я такое говорил?
>Все твои так называемые теории и исследования - не более чем статистические спекуляции.
Аноним 13/07/16 Срд 16:58:41 #494 №375269 
>>375267
Лел, то есть для тебя все, что не является наукой не существует? Ты хуже червя-пидора, честное слово. Есть устоявшееся представление о науке и научном знании, нет, блядь, ты свою хуйню продолжаешь толкать. Опросы и исследования поведения людей, которые могут сознательно вести себя неестественным образом у тебя являются объективными научными данными. Несопоставимости какие-то придумал, ни одной из которых так и не назвал. Чет я с тебя проигрывать начинаю.
Аноним 13/07/16 Срд 17:07:15 #495 №375272 
>>375269
>Есть устоявшееся представление о науке и научном знании
И ты применяешь ее к тем областям, в которых подобные критерии в принципе не примеными, на этом строишь свое опровержение и начинаешь за него держаться.
>Несопоставимости
Состояние сатори например, невозможно доказать измерительным прибором. Или скорее измерить переживание первого лица. Конечно таких людей можно считать шизофренниками, но к несчастью никакие критерии шизофринии у них более не выявляются. Ну или они настолько же сумасшедшие как Шредингер к примеру, который в своей же собственной книге описывал это состояние.
Так же невозможно к примеру описать переживание измененных состояний сознаний измерениями 3-его лица. А значит и подход к измерению этих областей не может быть тем же, что применяется к материальной части мира.
>Опросы и исследования поведения людей, которые могут сознательно вести себя неестественным образом у тебя являются объективными научными данными
Если эти результаты системны, и работают для колосального количества людей, это служит важным аргументом, в пользу определенной теории, описывающей эти явления.
Более не хочу продолжать эту бессмысленную демагогию, началось всё с твоего утверждения о том, что спиральная динамика - несостоятельная теория, но описание того, почему она не верна ты даже не собираешься приводить, разумеется потому что не знаешь в чем она заключается. Но тогда и говорить не о чем, точно так же как и твоих доводах о том, что психология история и астрология это одно и то же.
Аноним 13/07/16 Срд 17:10:34 #496 №375275 
>>375265
>Ты сказал, что не права спиральная динамика, а прочие вещи в числе которых психология вообще не существуют.
Нас тут более двух. Я - это >>374899 >>374916 >>374956 >>375225 >>375254
Началось с моего поста >>374899 содержащего целый ряд тезисов:
>Сейчас процесс повторяется в более крупном масштабе.
>В хрупком мире высокоразвитых технологий личные интересы индивида неизбежно будут конфликтовать с общественными.
>Чтобы человек всегда делал выбор в пользу общественных интересов, и не было даже малейшей возможности что сделает эгоистичный выбор, надо отрубить ему эго.
>Способы сделать это появяться у нас раньше, чем возможность улететь к другим звёздам.
>Будущее за тоталитаризмом, он неизбежен.
Ты предложил мне ответ ознакомиться с неким писанием - >>375194
Я не счёл этот пост интересным и никак не ответил. Другой анон начал сраться с тобой. Я подключился с ремаркой про джекпот. Затем написал про нью-эйджевость этого всего.

Так что твоё утверждение, что
>Пушка, это же ты говоришь, что я не прав.
>И я что, должен спорить с твоим отсутсвием обвинения даже?
неверно. Я такого не говорил. На данный момент между мной и тобой спора нет. Ты не оспорил моих аргументов, и не привёл своих.
Аноним 13/07/16 Срд 17:11:00 #497 №375276 
>>375254
>Ты неправ, даже объяснять лень. Купи книжку по физике, там все написано
Аноним 13/07/16 Срд 17:15:26 #498 №375277 
>>375276
>Ты неправ
Я не говорил такого. >>375275
Аноним 13/07/16 Срд 17:17:25 #499 №375278 
>>375272
>И ты применяешь ее к тем областям, в которых подобные критерии в принципе не примеными
Оно применимо ко всему, что претендует на научность. No exceptions.

>>375272
>Состояние сатори например, невозможно доказать измерительным прибором
"Следовательно о нем не имеет смысла говорить в рамках научной дискуссии" - бритва Ньютона.

>Если эти результаты системны, и работают для колосального количества людей, это служит важным аргументом, в пользу определенной теории, описывающей эти явления.
Статистика не рассчитана на свободу воли. Без фундаментального закона протекания явления ее интерпретация - голословная спекуляция.

>психология история и астрология это одно и то же
У тебя явные проблемы с восприятием текста, лол.

Как этот хуесос вообще в научном разделе оказался?
Аноним 13/07/16 Срд 17:27:16 #500 №375282 
14684200370170.jpg
>>375275
Ах, ну вот в чем дело. Окей:
>Сейчас процесс повторяется в более крупном масштабе. Мы, люди, стали кучковаться в сообщества, и у нас есть амбиции создать единую цивилизацию.
Это-то как раз и описывается Уилбером и Спиральной динамикой, внезапно. И как раз там более глубоко описываются эти этапы.
>Чтобы человек всегда делал выбор в пользу общественных интересов, и не было даже малейшей возможности что сделает эгоистичный выбор, надо отрубить ему эго.
И это тоже, но дело как раз в том, что в той схеме представлен уровень развития личностного сознавания до Эго-осознающего погугли значение термина, здесь нет никакой магии, чего достигают многие люди посредством различных практик, одной из самых полных и систематизированных из которых можно считать интегральную практику.
>В хрупком мире высокоразвитых технологий личные интересы индивида неизбежно будут конфликтовать с общественными.
И эта проблема так же отходит на другой план, по мере преодоления этого уровня с развитием системы моральных ценностей личности. Они описаны в книге.
>Будущее за тоталитаризмом, он неизбежен.
И по мере развития общества, все идет к тому, что подобные этапы все больше и больше отходят на второй план, к примеру это показывает описание уровней развития человека с архаического - 500 000 лет до н.э. до конвенционального 150-200 лет тому назад. Все больше и больше начинает преобладать стремление к плюрализму, и с каждым поколением все больше и больше этот плюралистический "центр тяжести" ценности общества достигается все быстрее и полноценнее. Но и он не конечный и не единственный.

А вот тут у тебя наступает путанница из-за насистематизированности знаний, вот смотри:
>Пчела не задумываясь отдаёт свою жизнь чтобы защитить улей.
Потому что у нее вообще нет никакого эго, или как минимум оно находятся на самом раннем этапе формирования, который человеческий вид прошел уже очень давно. И путанница возникает здесь из-за попытки уравнивания этого ДО-эгоцентрического состояния индивида к ПОСТ-эгоцентрическому развитию личности и Эго просветленных буддистов к примерю, или ЭГО-осознающего состояния сознания. Все это более подробно описано в книгах. Те же ошибки к примеру делал Карл Юнг описывая ТРАНСсрациональные мистические погугли значение термина, здесь нет никакой магии переживания и сопостовляя их с ДОрациональными состояниями у детей.

И именно поэтому, вероятнее всего высокоразвитые формы жизни попросту не дожили бы то времени развития таких технологий, и вот тут описание спиральной динамики - потому что как только они прошли бы все этапы спирали, начался бы новый виток, когда выживание с уровня дубинок, тупизны и палок поднялось бы к выживанию в мире с террористами и ядерным оружием - сильное несоответсвие когнитивного и морального уровней развития общества.

Я думаю теперь понятно с какой глубиной воззрений и ума подходит ко всему этому Уилбер? Все более подробное уже в конкретных статьях и книгах, потому что описать это в 2х словах будет трудно.
Аноним 13/07/16 Срд 17:36:49 #501 №375286 
>>375275
>Затем написал про нью-эйджевость этого всего.
И да, это конечно немного заставило меня немного взволноваться, потому что я лично знаю человека, знакомого с Уилбером. И он на своем примере явно демонстрирует полезность интегрального подхода.один из ведущих деятелей исскуства в польше
Аноним 13/07/16 Срд 17:45:41 #502 №375287 
Почему обычная бумага под высоким давлением взрывается?

Что взрывается в обычной бумаге? Газ? Вода?

https://www.youtube.com/watch?v=PmvKlnhMjUw
https://www.youtube.com/watch?v=KuG_CeEZV6w
Аноним 13/07/16 Срд 18:25:51 #503 №375292 
>>375187
Ты серьезно?
Нахуя тебе самодельная бочка?
Если от недостатка бабла - поверь, купить дешевле. Тебе потребуется ради одной бочки пилить печь, форму, и мини-лабораторию по измерению свойств пластика. Оно того не стоит.
Если ради фана - расскажу процесс подробнее.
Аноним 13/07/16 Срд 18:46:07 #504 №375295 
14684247678270.jpg
и ТАК. Уже как дохуя лет назад открыли базон хигза. Миллиарды долларов ушли на это. Почему я не чувствую изменений в материальном плане? Где обещаные профиты? Где технологии меняющие жизни людей? А эти пидоры ведь новый говнорагонятель за 25 лярдов строить собрались. МОжет нахуй их?
Аноним 13/07/16 Срд 18:51:03 #505 №375296 
>>375231
Проиграл. У меня от тебя ТЕМНАЯ ЭНЕРГИЯ.
Социология, психология, история, философия, математика и прочие абстрактные и не очень области - они работают с вещами, которые всегда здесь и сейчас, которые легко проверить экспериментально, и для этого не нужно строить новые йоба-ускорители или лететь на альфу центавра. Для подтверждения справедливости достаточно натурного эксперимента, чаще всего над группой людей - людей у нас хватает.
Так что эти области знаний наиболее актуальны из научных. А вот остальная наука - древнее неактуальное говнецо, поросшее мифами, отставшее от жизни, потому что не поддается проверке без полета к альфе центавра и йоба-ускорителей, которые подгонят только к 30-му веку.


И вообще вот эта вот ограниченность выглядит очень смешно.
Типа мы ограниченные и не признаем ничего нового и непроверенного (даже если оно работает) - значит мы занимаемся наукой. Хуита же.
Наука - это в первую очередь познание, любознательность, неуёмный полет фантазии, невероятные теории и сумасшедшие эксперименты. А не консерватизм головного мозга, скука и уныние.

Ты не сможешь увидеть то, что не способен себе вообразить.
Поэтому большинство открытий - это именно слабая корреляция наблюдаемого с ожидаемым.
В любых других случаях наблюдение было бы отброшено, и суть упущена, но ученый уже ожидал того, что наблюдает, так что для него эта слабая корреляция - как мигающий красный прожектор с громкой сиреной: "вот оно я, ты меня открыл, я существую, твои предположения подтвердились". И наблюдение защитывается, открытие оформляется.
Собственно открытие произошло намного раньше - когда ученый ПРИДУМАЛ некую концепцию.
Концепцию, обьясняющую что-то непонятное, или лучше обьясняющую уже знакомое, или предсказывающую что-то, что еще не встречалось.
Подтверждение ее реальности - лишь формальность.

А ты значит в абсолют возводишь именно эксперименты.
Что проверять то собрался, если ничего придумать не способен, если консерватор без фантазии до мозга костей?
Такие ученые максимум на что способны - считать и проверять за другими, это фактически лаборанты, науку они двигать не могут - у них нет никакого потенциала для этого, мозги то не работают. Ходячие калькуляторы.
Но компьютер то все-равно считает быстрее, зачем тогда они живут? Обслуживать компьютеры?
Аноним 13/07/16 Срд 19:21:07 #506 №375299 
>>375282
В природе экспоненциально нарастающие процессы не могут продолжаться вечно. Рано или поздно какой-то ресурс заканчивается и правила игры резко меняются. "Ресурс" тут может быть как чем-то материальным, так и некоторой сложной мерой.
Наша способность разрушать в последнее время росла экспоненциально. Наконец мы стали способны себя уничтожить. Свобода безнаказанно делать что угодно закончилась. Значит пришла пора меняться правилам игры.

Новые правила жестоки, в них нет места для плюрализма мнений. На вопрос "а может жахнем?" ответ всегда и у всех должен быть один - "нет". Либо идеальная дисциплина навечно, либо смерть. Смерть конечно более вероятна, не факт что технологический прогресс вообще можно пережить.

Возможно мы действительно двигались по спирали в каком-то смысле. Но что толку от этого знания, если эта траектория ведёт нас к обрыву?

>>375286
А что демонстрирует нам Марк Гафни?
Аноним 13/07/16 Срд 19:53:05 #507 №375302 
>>375088
>Материя - одна из форм энергии.
Сука как же мне хочется уебать за такую хуйню. Энергия это блядь свойство материи - мера движения материи и взаимодействия материи между собой, энергии без материи не бывает, запомни это тварь/пидор/сука/скотина/чмо ебаное/шакал ебучий/гандон/хуй простой/даун
Аноним 13/07/16 Срд 20:02:27 #508 №375303 
>>375087
Нет, это понятно. Я пытаюсь узнать вот что - в каком порядке стоит использовать свои знания? Ну вот например с едой, в виде грибов, ягод и пары птичек и водой проблем нет, что надо делать после? Просто исследовать планету до бесконечности?
Аноним 13/07/16 Срд 20:46:43 #509 №375308 
>>375296
Сначала я подумал, что ты зеленый. После прочтения всей твоей простыни уже и не уверен, может ты просто диванный. У науки, как это ни странно, есть консенсуальное определение, все, что невозможно проверить экспериментально к науке не относится по общенаучной договоренности. Причем эксперимент должен быть объективным, то есть построен на полностью понятных и неизменных помимо твоего желания основаниях. Социологическое наблюдение, например, объективным считаться не может. Человек - животное разумное и способное к умышленному срыву эксперимента, либо к нестандартному поведению с мотивацией "просто так". Ты можешь считать наукой все что угодно, хоть ландшафтный дизайн, хоть умозрительные эксперименты, хоть продажу апельсинов на рынке. От этого установленное определение науки не поменяется.
Аноним 13/07/16 Срд 21:04:30 #510 №375311 
Поясните за теорию вероятностей. Почему её часто используют для пояснения того, что такое то событие точно не произошло. Ну тип вероятность такого то события 0.0001 знач оно не происходило. Это корректно? МОжно же вроде рассчитать только вероятность повторения события? как можно рассчитать вероятность повтора события которое не происходило еще? А разве время или число "попыток" за которое указывается вероятность не надо указывать? На вики чёт сложна. Да и вообще. Теория вероятностей же применяется к событиям будущего, почему её применяют к событиям прошлого?
Аноним 13/07/16 Срд 21:32:26 #511 №375317 
>>375311
Обыкновенное упрощение. Если вероятность события достаточно мала, то ты его скорее всего не встретишь. Как нетрудно догадаться в подавляющем большинстве случаев такой прогноз оказывается верным. И почтенные ученые мужи экономят свое бесценное время и ресурсы, не пердолясь с учитыванием миллиардных долей процентов в каждом расчете. Простой пример, существует ненулевая вероятность прохождения тебя сквозь бетонную стену метровой толщины, однако сквозь стену ты не пройдешь никогда. Парадокс? Нет, просто всего времени жизни вселенной не хватит, чтобы приблизить вероятность этого события хотя бы к 1 проценту. Поэтому нужно не ебать себе и окружающим мозги и смело приписывать этому событию нулевую вероятность, помня при этом, что на самом деле она таки не ноль.
Аноним 13/07/16 Срд 21:35:05 #512 №375318 
>>375317
>хотя бы к 1 проценту
Хотя бы к одной удачной попытке

самофикс
Аноним 13/07/16 Срд 23:01:55 #513 №375348 
>>375317
Квантовая механика допускает вероятность нарушение второго начала термодинамики? Например кинул лёд в огонь, а он вместо того чтобы растаять и охладить огонь, охладился сам и подогрел огонь.
другой анон
Аноним 13/07/16 Срд 23:05:13 #514 №375350 
>>375348
Да. Второй закон термодинамики-лишь статистический, его нарушения возможны, просто маловероятны.
Аноним 14/07/16 Чтв 00:20:36 #515 №375365 
>>375350
Возможны нарушения в микросостояниях и только в них.
Аноним 14/07/16 Чтв 01:03:05 #516 №375371 
>>375299
>Возможно мы действительно двигались по спирали в каком-то смысле. Но что толку от этого знания, если эта траектория ведёт нас к обрыву?
Эта теория описывает именно новые витки в виде смены парадигм, а не маятниковый принцип к примеру. Ты правильно написал, правила игры и меняются, именно это и происходило все 6 этапов развития человечества до сегодняшнего дня, и сейчас в рамках описания этой модели , мы находимся на 7 уровне, и будут и дальше 8, 9 и т.д. И как раз таки, для чего все это прездназначено, нет никакой гарантии, что мы таки не уничтожим себя, т.е. разумеется безусловно общество очень сильно скакнуло со времен аборигенов, если дать им сейчас возможность управлять атомными бомбами, это приведет к неминуемой гибели, из-за несоответствия уровня развитий технологий с уровнями морального развития. И несмотря на все это, отставание по прежнему может наблюдаться и поныне, и в наших силах приложить усилия по недопущению этого. Модель описывает человеческую природу основываясь на исследованиях, а не на чьи-то предпочтения, проще говоря то, что модели не соответствовало, просто убиралось из описания или подвергалось критике или пересмотру. Никаких иллюзий.
Аноним 14/07/16 Чтв 01:14:32 #517 №375373 
>>375308
Это писал другой анон
>Социологическое наблюдение, например, объективным считаться не может
Но вот с этим ты реально уже заебал. Вот к примеру Джейн Левинджер и ее теория стадий развития моральных ценностей у женщин, был проведен эксперимент - опросили множество женщин, и задали вопрос: Аборт, можно ли его делать? И было 3 варианта ответов на выбор: Да можно, моё тело, делаю что хочу./Нет, определенно нельзя./Можно, но только в определенных случаях, когда это взвешенное решение и оно необходимо по каким-либо причинам. В абсолютном большинстве стадии ответов продвигались именно в таком порядке, в зависимости от времени проведение эксперимента с интервалом примерно в 3-4 года. И никогда эти стадии не двигались в обратном направлении, застопоренность имела место, но редукция - нет. И это один из примеров, очень сухих и почти без теоретического описания. Можешь просто открыть AQAL карту и посмотреть, перепрыгивал ли ты моментально от одного уровня по одной из линий развития к другой или же все таки нет. И не согласен я в первую очередь с тобой именно по тому, и все наше с тобой последнее обсуждение я все вел к тому, что эти методы не являются просто "твоего желания", а точно так же экспериментально проверяются. К примеру если бы те же самые женщины в этих ответах в абсолютном большинстве случаев отвечали бы бессистемно, то и теории об этом просто бы не возникло, или скажем описание тогда как раз таки и было бы другим, описывающим бессистемность и не структурированность развития.
Аноним 14/07/16 Чтв 01:34:35 #518 №375375 
>>375371
Ты мне скажи, вот это вот
https://www.integrallife.com/member/corey-devos/blog/narrative-guruji-ken-wilber
>"Blessed" crops ("blessed" is the word Mr. Trivedi uses to indicate the action of his intentionality in consciousness to bring about the desired change in the blessed object) show yields up to an astonishing 500% higher than a field of the same (but unblessed) crops lying right next to it. Seeing photos of these fields is actually rather hard to believe. The blessed half of the field (in this case, a mango field) looks like the Amazon Rainforest, with lush green vegetation everywhere; the other half of the field (the unblessed half) looks straggly, thin, and pale.
что за хуйня? Помолились - урожай на 500% вырос. Охуительная история просто. Это на таких-то исследованиях модель основывается?
Хотя мне наверное не понять, потому что уровень сознания у меня низкий, мозги промыты официальной наукой, и вообще я close-minded и потому не могу мыслить холистически.
https://www.youtube.com/watch?v=qUxWdIQVT_c
Аноним 14/07/16 Чтв 01:55:58 #519 №375377 
>>375302
Что такое гравитационное поле?
Аноним 14/07/16 Чтв 01:59:46 #520 №375379 
>>375303
Нет. Просто жить, развиваться.
Исследования сами собой пойдут - на них нет смысла фокусироваться специально.
Важнее наладить стабильный быт, общество.

Основная задача на первое время - минимизация рисков. Т.е. максимальное воспроизводство и получение стабильного источника пищи.
Аноним 14/07/16 Чтв 02:00:45 #521 №375381 
>>375311
Все, что тебе нужно знать про вероятности - они ничего не гарантируют.
Аноним 14/07/16 Чтв 02:03:38 #522 №375382 
>>375375
Хм, не знаю, но опять же как он сам и написал, если они просто провели тесты, и попросту озвучили их результаты, то им нужно либо верить, либо игнорировать их, да и тут еще обсуждение этой статьи, как я понимаю оно вызвало много споров https://www.integrallife.com/member/kimberella/blog/thoughts-and-concerns-about-guruji-endorsement. И там еще в статье говориться о как-то типе жизненной энергии. Об этом я вообще ничего не знаю и не в курсе. Такого он не описывал в своих книгах. Последняя их которых была переведена на русский вообще 12 лет назад. Не знаю на что ссылается статья, внизу указан только сайт, жаль не открывается, сам сайт в числе источников нигде не указан. Вот пример хода мыслей Уилбера, тут не 1500 страниц, и может это даст тебе представление о нем http://www.integralworld.net/ru/paulson-ru.html

Не знаю что тут сказать если честно, абсолютно всегда в его рассуждениях я отчетливо отслеживал критерии истинности, как рассуждать в этом случае не знаю, особой достоверности эта статья у меня не вызывает, ровно как и отношение уилбера к этому.

Конкретно этот приведенный пример наверное первый, котоырй вызвал у меня сомнения, буду обсуждать с одним человеком, который занимается переводом его книг. Это конечно еще нисколько не отрицает всей проделанной им работы. не потому что я так хочу или типа того, я описал уже это выше
Аноним 14/07/16 Чтв 02:05:31 #523 №375383 
>>375382
>>375375
Ах да, обязательно отпишусь тебе позже, если тред смоет, то в другом треде подобного рода.
Аноним 14/07/16 Чтв 02:23:07 #524 №375385 
>>375375
Ну даже вот к примеру, это очень точное описание хода его работ и мыслей, и о чем он всегда говорит -
>Уилбер высмеивает представление о том, что наш ум может изменить физическую реальность, он отвергает книги Нью-Эйдж и фильмы, наподобие «Дао Физики» и «Что мы об этом знаем», хотя его часто описывают как мыслителя Нью-Эйдж. Он также стремится показать, что «транс-рациональные» состояния сознания реальны, сравнивая научных материалистов, сомневающихся в этом с «плоскоземцами».
Так что не знаю, опять же ни сайт не работает ни других ссылок нет.
Аноним 14/07/16 Чтв 02:35:56 #525 №375388 
>>375385
Вебархив помнит. http://web.archive.org/web/20120111154136/http://www.kenwilber.com/blog/show/637
Аноним 14/07/16 Чтв 09:08:28 #526 №375416 
>>375377
Искажение пространства-времени массой, из-за чего искажаются мировые линии вот и всё.
Аноним 14/07/16 Чтв 09:09:47 #527 №375418 
Ребят бамплимит до 1000 у этого треда?
Аноним 14/07/16 Чтв 09:25:20 #528 №375420 
>>375418
1500
Аноним 14/07/16 Чтв 09:44:55 #529 №375421 
>>375416
>из-за чего искажаются мировые линии
подробней объясни
Аноним 14/07/16 Чтв 10:12:01 #530 №375427 
>>375421
Берём пробную частицу А и наблюдаем за её полётом с другой пробной частицы Л. В плоском пространстве их относительная скорость была бы постоянной, и для наблюдателя в Л движение А было бы прямолинейным равномерным, но в искривлённом это не так, из-за кривизны появляется относительное ускорение и наблюдаемое движение уже не прямолинейное.
Аналогия с Землёй: ты и твой друг отправляетесь с 2 разных точек экватора на север, при этом оба пытаетесь идти прямо с постоянной скоростью и не сворачивать. В начальный момент ваши скорости паралельны, и в случае плоской Земли вы бы шли по 2 паралельным прямым. Но Земля не плоская, и, наблюдая за другом, ты будешь замечать, что его скорость поворачивается в твою сторону, а на полюсе его траектория вообще пересекает твою.
Аноним 14/07/16 Чтв 13:43:03 #531 №375461 
Перекат в этот новый итт тред
>>375459 (OP)
comments powered by Disqus

Отзывы и предложения